Review of Psychiatry (Praveen Tripathi) - 1st Ed. (2016) PDF

Download as pdf or txt
Download as pdf or txt
You are on page 1of 142

j wl*t ' a/ .

J » y

:, visit our website www.jaypeebrothers.com, for detailed information on psychiatry books, visit our website www.jaypeebrothers.com
ÿ

led information on fi

Thoroughly Updated with all Recent Questions

Fully Updated with DSM-5 Guidelines

Free Online Exam Support WWW.QCCGSSpQ 17166,COtTl

Based on Kaplan's and Sadock's Comprehensive Textbook of Psychiatry, 9th edition


and Short Oxford Textbook of Psychiatry, 6th edition

Explained Referenced Answers


All Recent Questions (2015-1997)
All India (2012-1997)
AIIMS (Nov 2015-1997)
DNB (2012-1997)
AP, JIPMER, Bihar, UPSC CMS, PGMCET (2015) t
Other State Exams (2015-1997) i ,

A must-buy book for All India, AIIMS, PGI, JIPMER, DNB, FMGE & State entrance exams
Review of Psychiatry
Review of Psychiatry

Praveen Tripathi mbbs, md


Consultant, Psychiatry
Kailash Hospital and Research Institute
Noida, Uttar Pradesh, India

Foreword
Kailash Kedia mbbs, md

The Health Sciences Publisher


New Delhi | London | Panama | Philadelphia
Qayi'ee) Jaypee Brothers Medical Publishers (P) Ltd

Headquarters
Jaypee Brothers Medical Publishers (P) Ltd
4838/24, Ansari Road, Daryaganj
New Delhi 110 002, India
Phone: +91-11-43574357
Fax: +91-11-43574314
Email: jaypee@jaypeebrothers.com

Overseas Offices
J.P. Medical Ltd Jaypee-Highlights Medical Publishers Inc Jaypee Medical Inc
83 Victoria Street, London City of Knowledge, Bid. 235, 2nd Floor, Clayton 325 Chestnut Street
SW1H OHW (UK) Panama City, Panama Suite 412, Philadelphia, PA 19106, USA
Phone: +44 20 3170 8910 Phone: +1 507-301-0496 Phone: +1 267-519-9789
Fax: +44 (0)20 3008 6180 Fax: +1 507-301-0499 Email: support@jpmedus.com
Email: info@jpmedpub.com Email: cservice@jphmedical.com

Jaypee Brothers Medical Publishers (P) Ltd Jaypee Brothers Medical Publishers (P) Ltd
17/1-B Babar Road, Block-B, Shaymali Bhotahity, Kathmandu, Nepal
Mohammadpur, Dhaka-1207 Phone +977-9741283608
Bangladesh Email: kathmandu@jaypeebrothers.com
Mobile: +08801912003485
Email: jaypeedhaka@gmail.com

Website: www.jaypeebrothers.com
Website: www.jaypeedigital.com

© 2016, Jaypee Brothers Medical Publishers


The views and opinions expressed in this book are solely those of the original contributor(s)/author(s) and do not necessarily represent those
of editor(s) of the book.
All rights reserved. No part of this publication may be reproduced, stored or transmitted in any form or by any means, electronic, mechanical,
photocopying, recording or otherwise, without the prior permission in writing of the publishers.
All brand names and product names used in this book are trade names, service marks, trademarks or registered trademarks of their respective
owners. The publisher is not associated with any product or vendor mentioned in this book.
Medical knowledge and practice change constantly. This book is designed to provide accurate, authoritative information about the subject
matter in question. However, readers are advised to check the most current information available on procedures included and check
information from the manufacturer of each product to be administered, to verify the recommended dose, formula, method and duration of
administration, adverse effects and contraindications. It is the responsibility of the practitioner to take all appropriate safety precautions.
Neither the publisher nor the author(s)/editor(s) assume any liability for any injury and/or damage to persons or property arising from or
related to use of material in this book.
This book is sold on the understanding that the publisher is not engaged in providing professional medical services. If such advice or
services are required, the services of a competent medical professional should be sought.
Every effort has been made where necessary to contact holders of copyright to obtain permission to reproduce copyright material. If any
have been inadvertently overlooked, the publisher will be pleased to make the necessary arrangements at the first opportunity.

Inquiries for bulk sales may be solicited at: jaypee@jaypeebrothers.com


Review of Psychiatry

First Edition: 2016


ISBN 978-93-85999-52-9

Printed at Sanat Printers


Foreword
Psychiatry is quite different from mainstream medical specialties and poses unique challenges when the novice
medical graduate is attempting to understand these concepts. Psychiatry is also a fast evolving science and the recent
introduction of DSM-5 has led to several diagnostic revisions. Most of the textbooks on psychiatry are fairly exhaustive
and can be difficult to read for students preparing for entrance exams who are hard-pressed for time.
Keeping these aspects in mind Dr Tripathi has made enthusiastic efforts to compile the exhaustive literature on
mental health into a simple format that is highly readable and easy to understand. He has also included MCQs from
past examinations for practice and to adapt to the exam questions. Irecommend this book as a powerful and time
efficient tool to prepare for psychiatry section of postgraduate entrance examinations.
Iwish all the readers good luck and congratulate Dr Tripathi for his efforts in writing this book.

Kailash Kedia MBBS, MD


Staff Specialist
Princess Alexandra Hospital
Woolloongabba, Queensland-4102
Associate Lecturer
University of Queensland, Australia
Preface
Psychiatry is a complex subject and students have minimal exposure to psychiatric disorders during their MBBS
training. The terminology used in psychiatry is quite different from other medical specialties and makes the subject
tough to understand. Most of the students resort to rote memorization and struggle with the conceptual aspects. In
this book, an attempt has been made to explain the concepts in a simple language and without using the psychiatry
jargons. A large number of examples have been included in the text to explain the concepts and help in learning.
Another important aspect of this book is that it has been fully updated with DSM-5. In DSM-5, a large number of
new diagnoses have been introduced and diagnostic criterions of many existing disorders have been changed. All
these changes have been incorporated in the book.
This book has been written keeping in mind the needs of students preparing for various postgraduate entrance
examinations and MCI screening test. Nowadays, mastery over short subjects has become a key to get a good rank.
In most of the exams (including AIIMS, PGI and NEET), at least 5-6 questions are being asked from psychiatry. If
students can spare 5-6 days for psychiatry, they would be easily be able to get those questions correct and that will
make a real difference in the final ranks achieved.
Finally, a word of advise for the students. If you can keep yourself motivated for the entire duration of preparation,
cracking the entrance becomes a child's play. You should remain in regular touch with your seniors and take both tips
and inspiration from them. Appearing regularly for mock tests and discussion with peers is a good way of assessing
your strengths and weaknesses, it also motivates you to work harder and get better results next time. Remember you
need to win many small batties, before you can win a war.
So buckle up, get ready to bring your best to the table, work so hard that you surprise even yourself and achieve
what you righdy deserve.
My best wishes and blessings are always with you.

April, 2016
Praveen Tripathi mbbs, md
Consultant, Psychiatry
Kailash Hospital and Research Institute
Noida, Uttar Pradesh, India
info@ drpraveentrip athi.com
www.facebook.com/drpraveentripathipsychiatrist
Contents
1. Basics 1

2. Schizophrenia Spectrum and Other Psychotic Disorders 10

3. Mood Disorders 27

4. Neurotic, Stress Related and Somatoform Disorders 43

5. Substance Related and Addictive Disorders 63

6. Organic Mental Disorders 78

7. Personality Disorders 89

8. Eating Disorders 95

9. Sleep Disorders 98

10. Sexual Disorders 103

11. Child Psychiatry 107

12. Psychoanalysis 115

13. Miscellaneous 122


see
; ->
. -V.

Chapter

1 Basics

Psychiatry is the branch of medicine which deals with b. Depressed mood: Excessive sadness of mood,
morbid psychological processes. To establish diagnosis which is usually seen in depression.
of a psychiatric disorder both history and clinical exami¬ • Fluctuations: It refers to the changes in mood/affect.
nation are required. The clinical examination in psychia¬ The common disturbances of fluctuations are as
try, wherein the clinician records the psychiatric signs follows:
and symptoms, is known as Mental Status Examination a. Labile mood: Excessive variations in mood with¬
(MSE)Q. out any apparent reason. It is also known as
emotional lability0. For example, a man starts
Mental Status Examination * crying and then starts laughing without any
apparent reason. It is usually seen in mania.
In mental status examination, following areas of mental
b. Affective flattening: Absence of changes in mood
functioning are assessed:
irrespective of the situation. In this condition,
A. General appearance and behavior: The appearance of
patient doesn't experience any emotions hence
the patient is described along with any gross abnor¬
his affect remains the same. For example, a
malities (such as abnormalities of dressing etc).
B. Speech: Various aspects of speech such as rate, tone,
schizophrenic patient would not look happy
volume, spontaneity of speech are described. during festivals and did not appear sad when
C. Mood and affect The terms "affect" and "mood" are his mother died. His mood remained the same
both used to describe the emotions or emotional irrespective of the situation.
state. "Affect"Q is the cross sectional emotional state • Appropriateness and congruency: Appropriateness
whereas "mood" is the sustained or longitudinal of affect is described in relation to the social situa¬
emotional state. For example, if an individual who tion. For example, in a funeral, the expected emo¬
was extremely sad for last one month, gets extremely tional state is sadness. Hence, being sad in a funeral
and unusually happy for a moment; it can be said is an appropriate affect. If a man starts laughing
that his affect is happy (euphoric), whereas his mood and looks extremely happy in a funeral, it would
is depressed. The term affect and mood are at times be diagnosed as inappropriate affect. Congruency
used interchangeably. Affect and mood are further of mood is described in relation to the thought con¬
described under the following three subheads: tent of the person. Congruency describes whether
• Quality: It refers to the predominant affective (or the emotional state of person is in sync with his
mood) state. There can be various disturbances in thought/ speech or not. For example, if a man is
the quality of mood, common ones include: thinking about or talking about the events which
a. Euphoric mood (elevation of mood): Euphoria led to his mother's death, he is expected to be sad.
refers to a state of excessive happiness, without Hence, appearing sad while talking about mother's
any reason. It is usually seen in mania or hypo- death is a congruent affect. If a person, looks very
mania. happy and smiles while describing his mothers
2 Review of Psychiatry

death, it would be considered as incongruent voices are coming from the wall or from outside
mood. It must be stressed that while "appropri¬ the house. (Pseudohallucinations*3 are experi¬
ateness" of affect is described after comparing the enced in the inner subjective space, or originat¬
current affect with the expected affect in the given ing from within the mind. For example, a patient
social situation, the congruence is described after with auditory pseudohallucinations will report
comparing the current affect with the expected that the voices are originating within his mind
affect in the context of the patients thoughts. and not from outside).
Few other important disturbances of emotions d. Hallucinations are not under the willful con¬
include: trol*3 of the patient. It means that the patient can
a. Alexithymia: It refers to the inability to understand neither start the hallucinations nor can he stop
emotions of others and inability to express emo¬ them.
tions of self. Although alexithymia is closely related Hallucinations can occur in any modality. The
to affective flattening, alexithymiaQ is "lack of most common hallucinations in psychiatric disorders
words to describe emotions" rather than absence are auditory hallucinations*3. The most common
of emotions. hallucinations in organic psychiatric disorders
b. Anhedonia : It refers to the loss of capacity to expe¬ (such as delirium) are visual hallucinations*3. In
rience pleasure. The patient is unable to enjoy any¬ patients with temporal lobe epilepsy*3 all kinds of
thing in the life. hallucinations can be present including olfactory and
Neuroanatomicalsubstrate ofemotions: Limbic system*3 tactile hallucinations. Tactile hallucinations are also a
(which includes hippocampus, amygdala, hypothalamus, typical feature of cocaine intoxication.
cingulate gyrus and related thalamic and cortical areas) Few specific hallucinations:
is the neural substrate for the emotional experiences. The a. Hypnagogic hallucinations13: These hallucina¬
regulation of emotions is a function of frontal ldbe*3. tions occur while falling asleep or while going
to sleep. Since hypnagogic has the word "go" in
D. Perception: Perception is the receiving of information
it, hence its easy to remember that they occur
using one of the sensory modalities (i.e. auditory, vis¬
while "going" to sleep. Hypnagogic hallucina¬
ual, tactile, olfactory and gustatory). Two most impor¬
tions are seen in narcolepsy.
tant disturbances of perception are:
b. Hypnopompic hallucinations'3: These halluci¬
• Illusions'*: Illusion is false perception of a real nations occur while getting up from the sleep.
object. For example, a man mistakes a rope for
c. Reflex hallucinations (Synesthesia:QJ: In reflex
snake in night.
hallucinations, stimulus in one sensory moda¬
• Hallucinations: Hallucination is a false perception lity produces hallucinations in another sensory
in the absence of any object or stimulus. For exam¬
modality. For example, a patient reports that
ple, a patient of delirium reported seeing snakes on whenever he sees a white bulb (stimulus in
the ground of his room, when in reality there was
visual modality), he starts hearing voices of god
nothing there. Hallucinations have the following (hallucination in auditory modality). Reflex hal¬
properties and all these properties must be present lucinations are a feature of cannabis and LSD*3
to diagnose a perception as hallucination.
(and other hallucinogens) intoxication.
a. Hallucinations occur in the absence of any sen¬ d. Functional hallucination: Here, stimulus in one
sory or perceptual stimulus.
sensory modality, produces hallucinations in the
b. Hallucinations are as vivid (clear or detailed) same sensory modality. For example, a patient
as true perceptions. It means that the person reported that whenever he heard the sound of
who experiences hallucinations is able to give a a ticking clock (stimulus in auditory modality),
detailed description of what he is experiencing. he would also start hearing voices of god (hal¬
c. Hallucinations are experienced in outer objec¬ lucinations in auditory modality).
tive space*3. It means that patients experiences E. Thought (Cognition): The terms "thought" and
that the source of hallucinations is in the outer "cognition"*3 are at times used interchangeably, how¬
world. For example, a patient who is having ever in a stricter sense cognition is the mental process
auditory hallucinations will report that the of acquiring knowledge which includes thoughts but
Basics 3 |
also experiences and sensations. The thought distur¬ d. Circumstantiality**: It is a pattern of speech
bances are primary in many psychiatric disorders like which progresses with inclusion of lots of
schizophrenia. Thought and its disturbances can be unnecessary details and goes round and round
described under the following subheads. before reaching the final goal. For example, a
• Stream (Flow of thought): It refers to the speed with medical student was asked about his preferred
which thoughts follow each other. The disturbances branch in postgraduation and he replied by say¬
of stream includes: ing "Sir, in the first year iwas very interested in
a. Flight of ideas**: Here, the thoughts follow each physiology, however in the second year istarted
other very rapidly, and connection between dif¬ liking pathology. In the third year, istarted liking
ferent thoughts appears to be due to chance fac¬ ophthalmology however in the final year i rea¬

... ...
tors or rhyming. It is usually seen in mania. For
lized that i have a lot of liking for orthopedics
example, a manic patient when asked about his too and i liked putting casts and working with
hometown said "I live in Delhi...my cat has a big POP. Ialso think that after MBBS one should
belly i like to eat Jelly lilly lilly lilly" Some
get married as soon as possible and that noone
authors describe "flight of ideas" as an abnor¬ should have more than two kids...Well..you see
mality of form of thought. ilike pediatrics as a subject and want to do my
b. Inhibition of thinking: Here thoughts come in
postgraduation in the pediatrics" In this exam¬
mind very slowly and thought progresses with
ple the thought process progressed with inclu¬
a slow rate.
sion of lots of irrelevant details however in the
• Form of thought: The form refers to the "organi¬ end, the goal was reached as student said that
zation" of thought or the "association" between
he wants to become a pediatrician.
the consecutive thoughts. Normally, the thoughts
e. Tangentiality**: In tangentiality, the answer
are well organized and there is a connection
is related to the question in some distant way
between various components of a single thought
and the goal of thought is never reached. For
and between the consecutive thoughts. In formal
example, a patient was asked about his favorite
thought disorders, there are disturbance in the
bollywood actor and he replied "Well, you see
organization, associations and connections of the
the hindi movies are mostly hero centric and
thoughts. The important formal thought disorders
usually deal with the relationship issues whereas
include:
the hollywood movies have lots of action and
a. Derailment: In derailment, the association bet¬
science fiction. Ithink the Hindi Film Industry
ween two successive thoughts is disturbed. For
is growing rapidly and its a good medium for
example, a patient said Jawahar Lai Nehru was
entertainment of masses". In this example, the
the first prime minister of India and he was a
patients answer was distantly related to ques¬
congress leader. Sachin Tendulkar scored 100
tion, however the exact answer was never given.
international hundreds" In this example, there
f. Neologism: A neologismQ is coining of a new
is no link between the first thought about Nehru
word, whose derivation cannot be understood.
and second thought about Tendulkar.
For example, a patient would use the word
b. Loosening of association**: Here, the connection
is lost between components of a single thought. "tintintapa" for a pen. Neologism is highly sug¬
For example, a patient says "I thought that it will gestive of schizophrenia.
rain today, Modi is the current prime minister g. Word approximations (metonyms): Here, old
of India" In this example the phrase before the words are used in a new or unconventional way.
comma is totally disconnected from the phrase The meaning will be easily evident, though the
after the comma and hence this represents loos¬ word in itself might appear strange. For exam¬
ening of association. ple, a patient would us the world "time vessel"
c. Incoherence: It is the total lack of organization so for watch, and use the word "handshoes" for
that the thought is incomprehensible and does gloves.
not make any sense. For example, a patient says h. Perseveration: It is repetition of the same res¬
"India me churchgate pulses cricket computer" ponse, beyond the point of relevance. For
4 Review of Psychiatry

example, a patient was asked the following may claim that they have committed unpardon¬
questions. Q: What is your name. Ans. Mahesh able sins. It is usually seen in severe depression.
kumar....Q: Where do you live. Ans: Mahesh Bizarre Vs Nonbizarre Delusions
Kumar.....Q: How many children do you have... Bizarre delusions: The term bizarre is used for
A: Mahesh Kumar. delusions which are scientifically impossible
It must be noted that the perseveration is in and culturally implausible (ununderstandable).
response to a question and is not spontaneous. For example, if a patient says that aliens have
• Content of thought: It refers to what person is actu¬ stolen his heart, it would be an example of
ally thinking about. Delusion is a disorder of con¬ bizarre delusion.
tent of thought. It is defined as a false, unshakeable Nonbizarre delusions: These are delusions which
belief that cannot be explained on the basis of per¬ are false but are possible, i.e. they can happen.
sons social and cultural background. The following For example, if a patient develops a delusion
are the types of delusion: that his family members wants to take away his
a. Delusion of persecution: It is the most common property, it would be an example of nonbizarre
type of delusion.The patient believes that some¬ delusion, since it is not impossible for a family
one wants to harm him. For example, a patient member to take away property of another family
claimed that Indian police along with CBI is member.
hatching a conspiracy to kill him. • Possession of thought. Normally one experiences that
b. Delusion of reference: The patient believes that their thoughts belong to themselves and no one else
events happening around him are somehow can influence their thinking process, also there is a
related to him. For example, a patient claimed sense of control over one's thought. In disturbances
of possession of thought either the patients experi¬
that the tube light of his apartment was flicker¬
ences that others are tampering with their thoughts
ing as there was a camera fitted inside through
or that they have lost control over their thoughts. The
which his movements are being recorded.
disorders of possession include the following:
c. Delusion of grandeur or grandiosity: The patient
a. ObsessionsQ: Here, a thought comes repeatedly
believes that he has some exceptional identity or
into the mind of patient against his will. The
power. For example, a patient claimed that he is
patient recognizes the thought as his own, how¬
the reincarnation of Lord Hanuman and that he
ever is distressed by the repetitive and intrusive
can carry the mountains on his shoulders.
nature of the thought. The patient feels that he
d. Delusion of love (erotomania9, fantasy lover syn¬ has lost control over his thoughts.
drome): Patient may have false belief that some¬ b. Thought alienation: Here, the patient feels that
one is in love with them. It is also known as de their thoughts are under control of an outside
Clerambault syndrome. For example, a rickshaw agency or that others are interfering with their
puller claimed that Katrina Kaif is in love with thought process. Thought alienation pheno¬
him though he admitted that he has never met menon is of following types:
her. - Thought insertion: Patient feels that some
e. Nihilistic delusion (delusion of negation, Cotard's external agency is inserting foreign thoughts
syndromeQ): Here, the patient may deny exis¬ into their mind.
tence of their body, their mind, or the world in - Thought withdrawal: Patient experiences
general. They may claim that everybody is dead, that his thoughts are being withdrawn from
the world has stopped, etc. The basic theme of their mind by an external agency.
delusion is the "end of existence" - Thought broadcast: Patient experiences that
f. Delusion of infidelity (delusion ofjealousy): The thoughts are escaping from their minds and
patient has a false belief that his partner/spouse other people are able to access them.
is having an affair. It is also known as morbid F. Higher mental functions: In this component of MSE,
jealousy or Othello syndromeQ. various higher mental functions like attention, con¬
g. Delusion of guilt: Here, the patient may develop centration, memory, judgement, abstract thinking and
a delusion that they are bad or evil person and insight are assessed.
Basics 5 (
CLASSIFICATION _ are known to affect brain parenchyma) For example,
delirium, dementia.
At present, there are two major classificatory systems in B. Functional (Nonorganic) mental disorders: These dis¬
psychiatry. orders do not have any demonstrable disturbance
1.ICD-10 (International classification of diseases, 10th of brain parenchyma. For example, schizophrenia,
edition): It is published by WHO and provides classi¬ mania, etc.
fication for all medical disorders (including psychia¬ This classification is at best arbitrary, since with the
tric disorders). The psychiatric disorders have been advent of science its possible to demonstrate brain
classified in the chapter-V (F)Q of ICD-10. parenchyma disturbances even in so called "func¬
2. DSM-5 (Diagnostic and statistical manual of mental
tional" mental disorders.
disorders): It is published by American Psychiatric
Association. The fifth edition of DSM was published
Psychoses vs neuroses: The functional disorders can be
in 2013. further classified into psychotic disorders (psychoses)
and neurotic disorders (neuroses).
Psychiatric disorders have been classified in multiple A. Psychoses: Psychotic disorders are characterized by
ways. The most important classifications includes organic lack of awareness of illness (also known as lack of
vs functional psychiatric disorders and psychosis vs neu¬ insight)Q and impaired reality testing (i.e. the patients
rosis. loses contact with reality and start living in a fantasy
Organic vs Functional (Nonorganic) mental disorders: world created by their ill minds). For example, schizo¬
This was the first major classification of psychiatric/men¬ phrenia, bipolar disorder. Delusions and hallucina¬
tal disorders. tions are the prototype psychotic symptoms.
A. Organic mental disorders: These disorders are caused B. Neuroses: Neurotic disorders are characterized by aware¬
by demonstrable disturbances of brain (primary ness of the illness (insight is present) and reality contact
brain disturbances or systemic disturbances which is also intact. For example, anxiety disorders, depression.

QUESTIONS AND ANSWERS

QUESTIONS _ D. Inability to recognize and describe feelings


E. Inappropriate mood
.
1 Which of the following are sections of Mental State
Examination? (DNB NEET 2014-15) 4. A person who laughs at one minute and cries the
A. Mood and affect B. Speechandlanguage next minute without any clear stimulus is said to

C. Cognition D. All of the above have: (AIIMS Nov 2005)


A. Incongruent affect
B. Euphoria
Affect and Mood
C. Labile affect
2. A 25-year-old woman complaints of intense D. Split personality
depressed moodfor last 6 months. She also reports
5. Emotionis controlled by: (PGI1997)
inabilityto enjoypreviously pleasurable activities.
A. Limbic system B. Frontallobe
This symptom is known as: (AIIMS Nov 2005)
C. Temporal lobe D. Occipital lobe
A. Anhedonia B. Avolition
C. Apathy D. Amotivation Perception
3. Alexithymia is: (Kerala 2000, DNB 2004) 6. Phantom limb is an example of disorder of:
A. A feeling of intense rapture (DNB NEET 2104-15)
B. Pathological sadness A. Thought B. Perception
C. Affective flattening C. Cognition D. None of the above
I 6 Review of Psychiatry

7. A patient wanting to scratch for itching in his A. At the beginning of the sleep
amputated limb is an example of: B. At the end of sleep, while getting up
(DNBNEET 2014-15) C. After head trauma
A. Illusion D. After convulsions
B. Pseudohallucination
15. Hallucinations which occur at the "start" of sleep
C. Phantom limb hallucination
are known as: (JIPMER 2002, DNB 2005)
D. Autoscopic hallucination
A. Hypnagogic hallucinations
8. A patient sees a rope and gets afraid that it is a
B. Hypnopompic hallucinations
snake. This sign is known as:
C. Jactatio capitis nocturna
(DNBNEET 2014-15, PGI2002)
D. Extracampine hallucinations
A. Illusion B. Hallucination
C. Delusion D. Depersonalization 16. Hallucinations are seen in all except:
E. Derealization (MP 1999, DNB 2001)
A. Schizophrenia
9. A 8-year-old child after a tonsillectomy sees a bear
B. Seizures due to intracerebral space occupying
in her room. She screams in fright. A nurse who
lesions
rushes on switching the light, finds a rugwrapped
C. Lysergic acid diethyl amide intoxication (LSD
on an armchair. What child experiences is best
intoxication)
described as? (DNB 2006, Kerala 1997)
D. Anxiety
A. Illusion B. Hallucination
C. Delusion D. Depersonalization 17. Olfactory hallucinations are seen in:
(PGIMay 2011)
10. Which statement is nottrue about hallucinations?
A. Schizophrenia
(AIIMS 2009)
B. Alzheimer's disease
A. It is as vivid as a real perception
C. Mesial temporal sclerosis
B. It occurs in inner subjective space
D. Body dysmorphic disorder
C. It is independent of will of observer
E. Temporal lobe epilepsy
D. It occurs in the absence of any perceptual
stimulus 18. Visual hallucinations are seen in: (PGIlun 2009)
A. Hebephrenic schizophrenia
11. All of the following are features of hallucinations,
B. Residual schizophrenia
except: (AI2003)
C. Simple schizophrenia
A. It is independent of wall of observer
D. Delirium
B. Sensory organs are not involved E. Temporal lobe epilepsy
C. It is as vivid as a real perception
D. It occurs in the absence of any perceptual 19. Reflex hallucinations is a morbid variety of:
stimulus (AIIMS May 2009, 2011)
A. Kinesthesia B. Paresthesia
12. Formedvisual hallucinations are seen in lesions C. Hyperesthesia D. Synesthesia
of: (PGI2006, 2000)
A. Frontal lobe B. Temporal lobe Thought
C. Occipital lobe D. Parietal lobe
20. The term "cognition" is used to imply about:
13. The following is suggestive of an organic cause of (AI 1997, Jharkhand 2003, DNB 1998)
behavioral symptoms: (AI2002) A. Affect B. Perception
A. Formal thought disorder C. Thought D. Speech
B. Auditory hallucinations
21. True about thought is all except: (PGIFeb 2007)
C. Delusion of guilt
A. Perseveration is out of context repetition
D. Prominent visual hallucinations
B. Circumstantiality is over inclusion of irrelevant
14. When is hypnopompic phenomenon experienced? details while eventually getting back to the origi¬
(Bihar 2006, DNB 2002) nal point
Basics 7

C. Verbigeration is senseless repetition 29. A false belief which is unexplained by reality and
D. Vorbeireden is skirting around the end point but is shared by a number of people is:
never reaching it (AIIMS 2003, 2004 lipmer 1998)
E. Loosening of association is logically connected A. Illusion B. Delusion
thoughts with loss of goal. C. Obsession D. Superstition

22. Perseveration is: (AI2005) 30. The primary delusions are disorder of: (AI 1999)
A. Persistent and inappropriate repetition of the A. Flow of thought
same thoughts B. Form of thought
B. Feeling of distress in a patient with schizophrenia C. Content of thought
C. Characteristic of schizophrenia D. Possession of thought
D. Characteristic of obsessive compulsive disorder
31. Delusions are not likely to be seen in: (AI2012)
23. Inschizophrenia, characteristic feature is: A. Dementia B. Depression
(PGI1997) C. Schizophrenia D. Conversion disorder
A. Formal thought disorder
32. Delusions can be seen inall of the following except:
B. Delusion (SGPGI2002, DNB 2001)
C. Hallucination B. Depression
A. OCD
D. Apathy
C. Mania D. Schizophrenia
24. Loosening of association is an example of:
33. Delusion of persecution can be seen in:
(AI2006) (PGIJun 2009)
A. Formal thought disorder
A. Schizophrenia
B. Schneider's first rank symptoms B. Delusional disorder
C. Perseveration Manic episode
C.
D. Concrete thinking Melancholic depression
D.
25. Not a disorder of form of thought is:
34. Delusion of grandiosity can be seen in:
(AIIMS May 2012)
PGINov 2010, May 2011)
A. Tangentiality A. Hypomania
B. Derailment B. Paranoid schizophrenia
C. Thought block C. Schizoaffective disorder
D. Loosening of association D. Kleptomania/Pyromania
26. Which of the following is/are thought disorder? E. Cyclothymia
(DNBNEET 2014-15) 35. Nihilistic ideas are seen in: (PGIDec 2008)
A. Circumstantiality B. Tangentiality A. Simple schizophrenia
C. Prolixity D. All of the above B. Paranoid schizophrenia
27. Schizophrenia and depression both have the fol¬ C. Cotard's syndrome
lowing features except: (PGI2002) D. Depression
A. Formal thought disorder E. Body dysmorphic disorder
B. Social withdrawal 36. A 25-year-old university student had a fight with
C. Poor personal care
the neighbouring boy. On the next day while out,
D. Decreased interest in sex
he started feeling that two men in police uniform
E. Suicidal tendency
were observing his movements. When he reached
28. Delusion is a disorder of: home in the evening he was frightened and told
(DNB NEET 2014-15, AIIMS Nov 2006, AI2007) his family members that police was after him and
A. Perception B. Thought would arrest him. Despite reassurances by family
C. Insight D. Affect members, he remained afraid that he is about to
8 Review of Psychiatry

be arrested. The history is suggestive of which 6. B. Perception. In phantom limb, the patient feels
psychiatric sign/symptom: (AIIMS Nov 2003) sensations in the amputated limb. Hence, its a
A. Delusion of persecution disorder of perception.
B. Delusion of reference 7. C. Phantomlimb hallucination. Since, patient expe¬
C. Somatic passivity riences sensation inthe absence of any stimulus,
D. Thought insertion it is a hallucination. In autoscopic hallucination,
37. A man had a fight with his neighbor. The next day patient sees himself in the mirror and feels that
he started feeling that police is following him and "he" is the "image" i.e. what he is seeing is not
his brain is being controlled by radio waves by only an image but him.
his neighbor. The history is suggestive of which 8. A. Illusion.
psychiatric sign/ symptom: (AIIMS 1999) 9. A. Illusion.
A. Thought insertion Illusion is false perception of a real object.
B. Somatic passivity 10. B. It occurs ininner subjective space. Hallucinations
C. Delusion of persecution occur in outer and objective space; pseudohal-
D. Obsession lucinations occur in inner and subjective space.
11. None>B.
38. Healthy thinking includes all of the following
All the statements are correct. However, if one has
except: (AIIMS2011) to chose, the best answer would be B (sensory
A. Continuity B. Constancy
organs are not involved) as rest three options
C. Organization D. Clarity
form the criterion of hallucinations.
12. B. Temporal lobe. The lesions of temporal lobe can
Insight cause all types of hallucinations and formed
39. The awareness regarding the disease in mental visual hallucinations (elaborate visual hallucina¬
status examination is known as: tions) should raise a strong doubt of an organic
(AIIMS Nov 2012, May 2013) cause, specifically a temporal lobe pathology.
A. Insight B. Orientation 13. D. Prominent visual hallucinations. The presence
C. Judgment D. Rapport of prominent visual hallucinations is a strong
40. Impaired insight is found in: (PGI 1997) pointer towards an organic cause (i.e. a distur¬
A. Acute psychosis bance of brain parenchyma such as tumors).
B. Schizophrenia 14. B. At the end of sleep. While getting up.
C. Anxiety disorder 15. A. Hypnagogic hallucinations. These occur while
D. Obsessive compulsive disorder "going" to sleep. Jactatio capitis nocturna, or
rhythmic movement disorder is a neurological
41. If a person is asked, "what will he do if he sees a disorder characterized by involuntary move¬
house on fire"? Then what is being tested in that ments, usually of head and neck, before and
person? (DNB NEET 2014-15) during the sleep.
A. Social Judgment B. Test Judgment 16. D. Anxiety.
C. Response Judgment D. None 17. A, B, C, E.

ANSWERS _ Olfactory hallucinations can be seen intemporal


lobe epi-lepsy, medial temporal sclerosis (which
1. D. All of the above is a common cause of epilepsy). Though rare,
2. A. Anhedonia. Anhedonia is seen in both depres¬ olfactory hallucinations can also be present in
sion as well as schizophrenia. schizophrenia and Alzheimer's disease.
3. D. Inability to recognize and describe feelings. 18. A, D, E.
4. C. Labile affect. Visual hallucinations are the most common type
5. B. Frontal lobe. The neuroanatomical substrate for of hallucinations in delirium. Temporal lobe
generation of emotions is limbic system however epilepsy can present with all types of hallucina¬
the regulation/control of emotions is a function tions including visual hallucinations. In hebe¬
of frontal lobe. phrenic schizophrenia, the primary symptom is
Basics 9 1
disorganized behavior and formal though dis¬ 31. D. Conversion disorder. Conversion disorder is a
orders however hallucinations can also be seen. neurotic disorder (described in later chapters).
19. D. Synesthesia. Delusion is not a feature of conversion disorder.
20. C. Thought. 32. None > A.
21. E. Loosening of association is logically connected Delusion can be seen in schizophrenia, mania,
thoughts with loss of goal. In loosening of asso¬ depression as well as OCD. However the best
ciation, the connec-tions betweenthe thought is answer here would be OCD, as delusions are
lost.The rest of the statements are true. Verbige¬ rarely seen in OCD.
ration is a senseless repetition of one or several 33. A, B, C, D.
sentences or phrases. For example, a patient Delusions can be seen in allthese disorders. Mel¬
continued to repeat the following sentences for ancholic depression is usually seen in elderlies.
hours "Life is great. The lord is great. Summer 34. B, C.
will come soon" Its an example of verbigeration. Delusion of grandiosity can be seen in paranoid
Vorbeireden or vorbeigehen is seen in Ganser's schizophrenia and schizoaffective disorders.
syndrome (described in later chapters) and is Delusion of grandiosity can be seen inmania but
not in hypomania.
another name for approximate answers inwhich
35. B, C, D.
patient reaches close to the right answer, but
Nihilistic delusions can be seen in paranoid
never gives the right answer.
schizophrenia, Cotard's syndrome and depres¬
22. A. Persistent and inappropriate repetition of the
sion.
same thoughts.
36. A. Delusion of persecution.
23. A. Formal thought disorders are characteristic
37. C. Delusion of persecution. Here, inthe question the
abnormalities in schizophrenia. In schizophre¬
history for delusion of persecution (i.e police is
nia, the abnormalities of affect, perception, motor
following) is clear. The second half where patient
system as well as thought are present, however feels that his mind is being controlled by radio
the characteristic abnormality in schizophrenia waves is suggestive of possible though alienation
is that of thought, and more specifically the form phenomenon but we have not been provided
of thought (known as formal thought disorder). with any further details.
24. A. Formal thought disorder. 38. D. Clarity. Healthy thinking has three characte¬
25. C. Thought block. ristics (1) Continuity (2) Organization and (3)
26. D. All ofthe above. Prolixity is a milder form of "flight Constancy.
of ideas'! As mentioned in the text, flight of ideas 39. A. Insight
can be considered as both a disorder of stream 40. A, B.
of thought and form of thought. Only first two options are psychotic illnesses in
27. A. Formal thought disorder is seen only in schizo¬ which insight is impaired.
phrenia and not in depression. Rest all options 41. B. Test ludgment. Inmental status examination, the
can be present in either of the illnesses. judgment ofthe patient is also described. Patient
28. B. Thought. Delusion is a disorder of content of is given hypothetical scenarios such as "you see
thought. that a house is on fire" or "you find a letter lying
29. D. Superstition. There are many beliefs which are on the road" and is asked "what will you do" This
false and are shared by whole communities is called "test judgment" as patient's judgment is
e.g. black magic, witches etc. These beliefs are being tested in a hypothetical scenario. There are
considered as superstitions. In comparison, other forms of judgment like "social judgment"
delusions are not shared by members of the which describes whether a person is able to inter¬
same sociocultural background. For example, if a act socially in an appropriate manner. Finally, in
villager starts claiming that he is lord hanuman, "personal judgment" patient is asked about his
no one in his village will share his belief. future plans and it is assessed whether he has a
30. C. Content of thought. logical plan for his future or not.
Chapter
Schizophrenia Spectrum and
2 Other Psychotic Disorders

Schizophrenia is the prototype of psychotic disorders. It is C. Affect disturbances: Disturbances of emotions such as
one of the most common serious mental disorders.

_
inappropriate affect.
D. Association disturbances: Disturbances of association
HISTORY of thoughts such as formal thought disorders.

Emil Kraepelin Kurt Schneider


Kraepelin classified psychiatric illnesses into two clini¬
Schneider described a group of symptoms, popularly
cal types: Dementia PraecoxQ and Manic Depressive
known as Schneiderian First Rank Symptoms (SFRS)°
IllnessQ. The basis of this classification is the course of
which were frequently seen in patients of schizophrenia
illness and the cognitive decline. and were characteristic of the illness. It must be however
Dementia Praecox is characterized by a chronic and remembered that these symptoms can also be present in
deteriorating course along with gradual decline of cog¬ other illnesses and hence are not specific or pathogno¬
nitive functions (i.e. gradual decline of memory, atten¬ monic of schizophrenia. There are 11 Schneiderian First
tion and goal directed behavior). The term "dementia" Rank Symptoms.
was used to indicate gradual decline in cognitive func¬ A. Three thought phenomenon: These three together are
tions and the term "praecox' was added since the onset known as thought alienation phenomenon in which
of illness was in young age (praecox means early onset). patient feels as if some one is tampering with his mind
In contrast Manic Depressive illness is characterized and thoughts. The thought alienation includes the fol¬
by distinct0 episodes of illness alternating with period lowing:
of normal functioning. Also, there is no cognitive • Thought insertion (patient reports that someone is
decline. putting thoughts in his mind)
• Thought withdrawal (patient experiences that
Eugen Bleuler thoughts are being taken out of his mind)
• Thought broadcast (patient experiences that
Bleuler coined the term "Schizophrenia"0, which thoughts are leaving his mind and that others are
replaced dementia praecox in scientific literature. Bleuler able to access his thoughts, e.g. patient would say
proposed four symptoms which he called as fundamental that "everybody understands my thoughts, though
(or primary) symptoms of schizophrenia. These symp¬ Inever say anything".
toms are also known as 4 A's of Bleuler0. They include: B. Three made phenomenon: Here the patient experi¬
A .Autistic thinking and behavior (Autism): Excessive ences that his emotions, actions and drives are being
fantasy thinking which is irrational and withdrawn influenced by others. It includes the following:
behavior. • Made volition: The patient experiences that his
B. Ambivalence: Marked inability to take a decision. actions are being controlled by an external agency
Schizophrenia Spectrum and Other Psychotic Disorders 11

and not by himself. For example, a patient would said "I will have dinner at a restaurant tonight".
repeatedly put his hand in the fan, and on asking The German word "Gedankenlautwerden" or the
the reason reported, "I don't want to do it myself french word "echo de pensees" is occasionally
but Iam being controlled by aliens who can mani¬ used to describe these audible thoughts.
pulate my actions, Iam a robot for them and they D. Somatic passivity: In somatic passivity, patient expe¬
have my remote control" riences tactile or visceral hallucinations which he
• Made affect The patient experiences that someone believes are being imposed by some external agent.
is changing his affect (emotions). For example, a For example, a patient reported that he feels intense
patient reported "at times Istart laughing loudly burning sensation inside his right knee and claimed
and at times Icry. The neighbours control my emo¬ that it is because of UV rays sent by FBI agents from
tions, they can change it whenever they want to. I New York"
feel helpless" E. Delusional perception: In Delusional perception, a
• Made impulses: The patient experiences that some¬ delusion is attached to a normal perception. For exam¬
one is putting certain "drives" in his mind. For ple, a patient of schizophrenia looked at the ceiling fan
example, a patient suddenly threw his coffee mug and immediately understood that the "all the people
onto a nurse. On asking about it he reported "a sud¬ in the city consider him a homosexual" In this exam¬
den impulse came over me, this impulse was sent ple there was a normal perception in the first step (i.e.
by CBI officers who wanted me to throw the mug. the patient saw a ceiling fan) and in the second step a
Itried resisting the impulse, but could not control
delusion was attached to this normal perception (i.e.
it". the delusion that everybody in city considers patient
C. Three auditory hallucinations: a homosexual). Delusional perception is a type of
• Voices arguing or discussing: The patient reports "primary de!usion"Q. Primary delusions are those
hearing of two or more voices which argue or delusions which arise directly as a result of morbid
discuss about the patient. The patient is usually
psychological processes whereas secondary delusions
referred to in third person (hence also called third
develop secondarily to some other psychopathologi-
person auditory hallucinations'3). For example, cal phenomenon. For example, a patient who had
the first voice would say "he is a strange man, he
continuous auditory hallucinations of a voice which
doesn't have any good qualities'! The second voice
said "you will be killed" started believing that "some¬
would respond "yes, also look how fat he has
body wants to harm me" Now, this "delusion of perse¬
become" In this example the patient is hearing two
cution" which developed is a secondary delusion as it
voices and the voices are using the word "he" to
developed secondarily to the auditory hallucinations.
refer to the patient, hence patient is being referred
to in third person.
• Voices commenting on patient's action: Here, the
EPIDEMIOLOGY _
patient hears voices which give a running com¬ The lifetime prevalence of schizophrenia is 1% whereas
mentary on the patient's activities. For example, the point prevalence is 0.5-1%. The incidence rate is
a patient who was working in the kitchen heard 0.15-0.25 per thousand.
the following voice "she has peeled the potato and A. Prevalence in specific population: Schizophrenia has
now she is about to switch on the gas. Now, she high heritabilityQ. The prevalence in general popula¬
has started to wash the potatoes" The voice usually tion is 1% however in relatives of patients, the rate
refers to the patient in third person, hence this can is higher. The following table mentions the rates for
again be an example of third person auditory hal¬ specific population groups.
lucinations. Hie usual age of onset of schizophrenia is adoles¬
• Audible thoughts: Here the patients hears a voice, cence'3 and young adulthood. When the onset occurs
which would say aloud whatever patient would after age of 45 years, the disorder is called as late-onset
think. For example, a patient had a thought that "I schizophrenia13.
will have dinner at a restaurant tonight" Immediately It is equally prevalent in men and women, however
he heard a voice of a middle aged women who the onset is earlier in men.
12 Review of Psychiatry

Table 1: Prevalence of Schizophrenia in specific populations. C. Neuropathological factors: The neuropathology of


• General: 1%
schizophrenia is still not clear. Abnormalities have
been found in various structures, such as:
• Non twin sibling of a schizophrenia patient: 8%
• Cerebral ventricles: Reduction in cortical gray matter
• Dizygotic twin of a schizophrenic patient: 12%
volume and enlargement of lateral and third ven¬
• Monozygotic twin of a schizophrenic patient: 47% tricles has been consistently observed.
• Child with one parent with schizophrenia: 12% • Limbic system: Abnormalities in limbic system
• Child with both parents with schizophrenia: 40% components such as hippocampus (smaller in size
and functionally abnormal), amygdala (smaller
size) and parahippocampal gyrus (smaller size)
Schizophrenia is more prevalent in lower socioeconomic
have been observed.
status. It was earlier believed that different body types
• Prefrontal cortex: Anatomical abnormalities have
were related to different personalities and also had dif¬
been found.
ferent vulnerability to some disorders. Three types of body
• Thalamus: Neuronal loss especially in medial dor¬
types were described: asthenic (thin and weak), athletic sal nucleus of thalamus.
(muscular) and pyknic (short and fat). The asthenic0
• Basal ganglia and cerebellum: Abnormalities have
and to a lesser extent athletic persons were believed to be been reported without any conclusive proof.
predisposed for development of schizophrenia whereas
the pyknic were believed to be predisposed to manic SYMPTOMS __
depressive illness (bipolar disorder).
The symptoms of schizophrenia can be divided into vari¬
ous symptom complexes, described as follows:
ETIOLOGY AND PATHOGENESIS_
A. Positive symptoms
A. Genetic factors: (or psychotic symp¬ _y In DSM-4, the presence of
bizarre delusions was considered
• Schizophrenia has a genetic contribution as toms): The two
enough to satisfy the Criterion A for
reflected by higher monozygotic concordance rate positive symptoms schizophrenia, however in DSM-5, the
than dizygotic concordance rate. Several genes include delusions concept of bizarre delusions has been
removed and it no longer carries any
appear to make a contribution to schizophrenia and hallucinations. special diagnostic significance.
and nine linkage sites have been identified: lq, 5q, They respond well
6p, 6q, 8p, lOp, 13q, 15q and 22q. to medications and the presence of positive symp¬
toms is a good prognostic factor0 in schizophrenia.
• Several candidate genes contributing to schizo¬
phrenia have been identified, and they include • Delusions: The most common delusion in schizo¬
a-7 nicotinic receptor, DISC 1(Disrupted in schizo¬ phrenia is delusion of persecution. A category of
delusion that holds special significance in schizo¬
phrenia), COMT (catechol-o-methyl transferase),
phrenia is the so called "bizarre delusions".
NRG 1 (Neuregulin 1), GRM-3 (Glutamate recep¬
Bizarre delusions are those that are considered
tor metabotropic), RGS-4 (Regulator of G protein
physically impossible and culturally implausible
signalling) and DAOA (or G-72) (D-Amino acid
(or ununderstandable). For example, "a patient
oxidase activator).
claimed that he has been sent by aliens from mars
B. Biochemicalfactors:
and his purpose is to evaporate all the water from
• Dopamine hypothesis: This hypothesis proposes earth and make it dry" This patient is having a
that excess of dopaminergic activity0 is responsi¬
bizarre delusion as his belief is both impossible
ble for schizophrenia. and ununderstandable.
• Serotonin: Currently, along with dopamine, an • Hallucinations: The most common hallucinations
excess of serotonin is also considered to be respon¬ in schizophrenia are auditory hallucinations0.
sible for symptoms of schizophrenia. Visual hallucinations are the second most com¬
• Other neurotransmitters like GABA, glutamate, mon, however the presence of visual hallucination
norepinephrine, acetylcholine, nicotine have also should always raise the suspicion of an organic
been implicated in pathogenesis of schizophrenia. mental disorder.
Schizophrenia Spectrum and Other Psychotic Disorders 13

• The positivesymptoms of schizophrenia are due • Stupor: Extreme hypoactivity or immobility and
to dopamine excess in mesolimbic tract (neural minimal responsiveness to stimuli.
pathway from ventral segmental area to nucleus • Excitement: Extreme hyperactivity which is usually
accumbens)Q. non goal directed (i.e. the patient is very active but
B. Negative symptoms: Negative symptoms represent doesn't do any meaningful work).
"loss of normal functions" in patients with schizo¬ • Posturing/catalepsy: Spontaneous maintenance of
phrenia. These symptoms respond poorly to medica¬ posture for long periods of time.
tions and their presence is a bad prognostic factorQ Waxy flexibility: When examiner makes a passive

in schizophrenia. Following are the negative symp¬ movement on patient, there is a feeling of plastic
toms:
resistance which resembles bending of a soft wax
• Avolition: Loss of will or drive to indulge in goal
candle.
directed activities (such as grooming and hygiene,
education and occupational activities).
• Automatic obedience: Excessive cooperation with
examiner's commands despite unpleasant conse¬
• Apathy: Loss of concern for an idea or task or quences. For example, a patient kept on protruding
results. For example, a student who had deve¬
his tongue in response to examiner's commands,
loped schizophrenia failed in exams. However he
despite the fact that his tongue would be pricked
appeared unconcerned with his results.
by a pin everytime he protruded it.
• Anhedonia : Loss of ability to derive pleasure from
activities or relationships. • Echolalia: Mimicking of examiner's speech.
Asociality: • Echopraxia: Mimicking of examiner's movements.
• Indifference to social relationships and
decrease in the drive to socialize. • Negativism: Patient refuses to accept examiner's
instructions or any attempts to move him.
• Affective flattening (or blunting): Inability of patient
to under- stand emotions of others and inability to • Grimacingf: Maintenance of odd facial expres¬
sions.
express own emotions.
• Alogia: Decrease in verbal communication. • Stereotypy: Spontaneous repetition of odd, pur¬
poseless movements. For example, making strange
The negative symptoms are due to decreased
movements of fingers repeatedly.
dopamine activity in mesocortical pathway (neu¬
ral pathway from ventral segmental area to prefron¬ • Gegenhalten: Resistance to passive movement,
which is directly proportional to the strength of
tal cortex).
force applied.
C. Disorganization symptoms: This symptom complex
includes the following symptoms: • Mannerisms: Spontaneous repetition of odd, pur¬
poseful movements. For example, repeatedly
• Formal thought disorder: These are the distur¬
bances in the form of thought characterized by loss saluting the passerby.
of organization of thought. • Perseveration: It is an induced movement which is
senselessly repeated. For example, A patient takes
• Disorganized behavior: It is the odd and inap¬
propriate behavior which may break the social his tongue out and in, when asked however then
norms. For example, a hospitalized schizophrenic keeps on repeating the out and in movement , even
patient would masturbate in front of the nursing when he is no longer asked. It must be noted that
staff, another patient of schizophrenia would wear perseveration occurs in response to an instruction,
sweaters and coats in hot summer season. whereas stereotypy and mannerisms are spontane¬
• Inappropriate affect: Affect which is not in sync ous. Perseveration is also a sign of brain damage
with the social situation. (organic brain disorders)Q.
D. Motor symptoms (catatonic symptoms): The term "cat¬ • Ambitendency: Inability to decide the desired motor
atonia" was given by Karl KahlbaumQ who described movement. For example, when offered a hand for
these motor symptoms for the first time. These symp¬ handshake, patient may repeatedly bring his hand
toms are sometimes described along with disorgani¬ forward and backward as he is not able to decide
zation symptoms. For more clarity, they have been whether he wants to shake the hand or not. It is
described separately here. These include: ambivalence in motor movements0.
14 Review of Psychiatry

DIAGNOSIS E. Residual schizophrenia: Residual schizophrenia is


characterized by progression from an early stage (with
According to DSM-5, two or more of the following symp¬ prominent delusions and hallucinations) to a later
toms should be present for a duration of 1 month period
stage where the delusions and hallucinations have
and at least one of these must be either (1), (2) or (3) become minimal and mostly negative symptoms are
1. Delusions
present.
2. Hallucinations
F. Simple schizophre¬ DSM-5 Update: The DSM-4
3. Disorganized speech (or formal thought disorder)
nia: There are described multiple subtypes of schizo¬
4. Disorganized or catatonic behavior phrenia (like paranoid, catatonic, dis¬
prominent negative
5. Negative symptoms. organized, catatonic, undifferentiated,
symptoms without residual). The DSM-5 has eliminated
The total duration of illness should be at least all of them and does not describe any
any history of posi¬
6 months, and the 6 months period must include at least subtypes.
tive symptoms like
one month of above mentioned symptoms.
delusion and hallucinations. It has the worst prog¬
The ICD-10 also
nosis.
uses similar criterion DSM-5 Update: In DSM-4, only one G. Post schizophrenic depression: A depressive episode
for diagnosis of schizo¬ of the above symptoms was required if
the delusions were bizarre or halluci¬ which develops after the resolution of schizophrenic
phrenia however the nations were one of schneiderian first symptoms. This disorder is associated with an
total duration of symp¬ rank symptoms (either voices discuss¬
ing about the patient or voices giving increased risk of suicide.
toms should be more
a running commentary). However in
than one month unlike DSM-5 this special attribution to bizarre
DSM-5 which requires delusions and schneiderian auditory Other Classifications
hallucinations has been removed.
a total duration more Apart from ICD-10 and DSM-5, various other classifica¬
than six months. tions have been proposed.
A. TI Crow divided schizophrenia into two subtypes,
Types namely Type Iand Type II schizophrenia:
• Type I: Mostly
According to ICD-10, the following are the types of positive symp¬ Substances which can cause schi¬
schizophrenia: toms with nor¬ zophrenia like symptoms: Ampheta¬
A. Paranoid schizophrenia: This type is dominated by mal ventricles, mines, cocaine, phencyclidine and
other hallucinogens, cannabis.
hallucinations and delusions. This is the most com¬ good response
mon type of schizophrenia. It has a late onset and to medications and better prognosis.
a good prognosis0. The personality is usually pre¬
• Type II: Mostly negative symptoms with dilated
served (the person is able to maintain daily activities ventricles, poor response to medications and
and social interaction is normal). poor prognosis.
B. Catatonic schizophrenia: This type is dominated by B. Pfopf schizophrenia: Schizophrenia in a patient with
catatonic (motor) symptoms. It has the best progno¬ mental retardation.
sis of all types. The first line treatment for catatonic C. Van Gogh syndrome: Self mutilation (injuring self)
schizophrenia includes intravenous lorazepam and occurring in schizophrenia has also been called Van
electroconvulsive therapy. Gogh syndrome.
C. Hebephrenic (disorganized) schizophrenia: This type
is dominated by prominent disorganization symp¬
toms and negative symptoms. It has an early onset
TREATMENT _
and bad prognosis. There is severe deterioration of Antipsychotics (also known as neuroleptics) are the main¬
personality (patient is not able to maintain hygiene, stay of treatment for psychotic disorders like schizophre¬
social interaction is inappropriate, odd behaviors are nia, schizoaffective disorders, delusional disorders and
present). others. Antipsychotics have been divided into two classes:
D. Undifferentiated schizophrenia: The schizophrenia not (1) Typical antipsychotics and (2) Atypical antipsychotics
conforming to any of the above subtypes or exhibiting 1. Typical antipsychotics or first generation antipsycho¬
features of more than one of them. tics or dopamine receptor antagonists (DRAs): These
Schizophrenia Spectrum and Other Psychotic Disorders 15 f
drugs mainly act through dopamine, D2 receptor prophylactic use of oral anticholinergics is suggested
antagonism. They were the first antipsychotics that while prescribing typical antipsychotics.
were used in the clinical practice. They are effec¬ • Acute akathisia: It is the commonest side effect
tive against positive symptoms but have mini¬ of antipsychotics and is characterized by an inner
mal effect on negative symptoms. The therapeutic sense of restlessness along with objective, observ¬
effect of improvement in psychotic symptoms is able movements such as fidgeting0 of legs, pacing
mediated by D2 receptor antagonism in mesolim- around, inability to sit or stand in one place for a
bic tract. The typical antipsychotics can further be long time. The treatment options include P block¬
classified according to their chemical groups, as ers0 such as propranolol (drug of choice), anticho¬
described below: linergics and benzodiazepines. The antipsychotic
• Phenothiazines: Chlorpromazine, Thioridazine, Tri¬ can also be changed to a second generation or low
fluoperazine, Prochlorperazine, Triflupromazine, potency first generation antipsychotics, which have
Fluphenazine, Perphenazine lesser incidence of akathisia.
• Thioxanthenes: Thiothixene, flupenthixol • Drug induced parkinsonism: It is characterized
• Butyrophenon.es: Haloperidol, droperidol, penfluri¬ by the triad of rigidity, bradykinesia and resting
dol tremors. The treatment options include use of
• Miscellaneous: Pimozide, loxapine, molindone. anticholinergics or change of antipsychotics to
The typical antipsychotics can further be classified second generation or low potency first generation
as low potency (like chlorpromazine, thioridazine) antipsychotics. The dose reduction can also be tried.
and high potency (like haloperidol and fluphenazine). Often, use of prophylactic anticholinergics prevents
Apart from differing in potency, the low potency and high the development of drug induced parkinsonism.
potency antipsychotics also differ in their side effects • Tardive dyskinesia: The term "tardive" refers to
profile. The common side effects of typical antipsychotics features which develop after prolonged exposure.
are as follows: Tardive dyskinesia develops after long-term treat¬
A. Movement disorders: The antipsychotics can cause vari¬ ment with antipsychotics and can present as invol¬
ous movement disorders, which collectively are often untary movements of the tongue (e.g. twisting,
referred as extrapyramidal symptoms (or extrapy¬ protrusion), jaw (e.g. chewing), lips (e.g. smacking,
ramidal side effects). These side effects are caused by puckering), trunk or extremities. Patient may also
blockade of dopamine receptors in nigrostriatal tract have rapid, jerky movements (choreiform move¬
(neural pathway from substantia nigra to striatum). ments) or slow, sinusoid movements (athetoid
The movement disorders are more commonly seen movements). The management usually includes
with typical antipsychotics in comparison to atypical shifting to a second generation medication.
antipsychotics and amongst typical antipsychotics, • Neuroleptic malignant syndrome: It is a fatal side
high potency typical antipsychotics are more likely effect of antipsychotic use. It is characterized by
to cause this side effect. The movement disorders can muscle rigidity, elevated temperature (greater
be of the following types: than 38°C), and increased CPK (creatine phos-
• Acute dystonia: It is the earliest side effect0 of phokinase) levels. The other symptoms include
antipsychotics and can be seen within minutes diaphoresis, tremors, confusion, autonomic distur¬
of receiving an injectable antipsychotic (also with bances, liver enzyme elevation and leukocytosis.
oral antipsychotic). It is characterized by sudden The pathophysiology involves D2 antagonism at
contraction of a muscle group and can result in various levels. The D2 receptors blockade in corpus
symptoms like torticollis0, trismus (contraction striatum causes muscle contraction and rigidity
of jaw muscles),0 deviation of eye balls that initiates heat generation, whereas blockade
(oculogyric crisis due to contraction of extraocular of dopamine receptors in hypothalamus interferes
muscles), laryngospasm, etc. The management with heat regulation. Hie autonomic disturbances
includes immediate administration of parenteral are caused by dopamine blockade of spinal neu¬
anticholinergics0 like benztropine, promethazine rons. The increased CPK indicates muscle injury.
or diphenhydramine0. To prevent acute dystonia, The early recognition of symptoms and prompt
ÿ 16 Review of Psychiatry

withdrawal of antipsychotics is of paramount earlier, however the incidence is lesser in comparison


importance, otherwise the continuing muscle dam¬ to the typical antipsychotics.
age can cause myoglobinuria and renal failure. B. Endocrine side effects: The incidence of hyperpro¬
The treatment includes skeletal muscle relax¬ lactinemia is also lesser with atypical antipsychotics
ants like dantroleneÿ, dopamine agonists such (except risperidone and amisulpride which have a
as amantadine and bromocriptine are also useful. higher incidence).
Supportive measures including adequate hydration C. Weight gain and increased risk of dyslipidemia, dia¬
are also important in the management. When drug
betes and cardiovascular disease is more commonly
treatment with antipsychotics is restarted, second
seen with atypical antipsychotics in comparison to
generation antipsychotics should be used.
B. Endocrine side effects: The blockage of dopamine
typical antipsychotics.
D. Other side effects include sedation, QTc prolongation
receptors in tuberoinfundibular tract results in
hyperprolactinemia (remember dopamine inhibits (especially with ziprasidone) and seizures.
prolactin secretion and hence dopamine blockade
causes hyperprolactinemia) and can cause galactor¬ Clozapine
rhea, menstrual disturbances in females and impo¬ It was the first atypical antipsychotic to be synthesized.
tence in males.
Clozapine is the drug of choice in treatment resistance
C. Sedation, orthostatic hypotension and anticholinergic
schizophrenia. Clozapine is a unique drug as unlike
side effects are usually see with low potency typical
other antipsychotics, it has a relatively low affinity for D2
antipsychotics.
receptors. This low affinity for D2 receptor explains lack
2. Atypical antipsychotics or second generation anti¬ of extrapyramidal side effects on clozapine. Clozapine has
psychotics or serotonin dopamine antagonists: These a strong affinity for D4 receptors and also acts as an
drugs act through anta- gonism of 5HT 2 receptors as antagonist at 5 HT2A, Dl, D3 and a (alpha) adrenergic
well of D2 receptors. These drugs have a higher ratio receptors. The lack of extrapyramidal symptoms, makes
of 5 HT2 to D2 blockade, in contrast the typical anti¬ clozapine a preferred antipsychotic in patients who are
psychotics primarily act on D2 receptors. Due to lesser intolerant to other antipsychotics because of extrapyrami¬
D2 blockade, atypical antipsychotics have lesser risk of dal side effects including tardive dyskinesia.
causing extrapyramidal side effects as well as hyper¬
Side effects: The common side effects of clozapine include
prolactinemia. Atypical antipsychotics are effective in
treatment of both positive and negative symptoms.
sedation, syncope, hypotension, tachycardia, nausea and
The following drugs are classified as atypical anti¬ vomiting. Other side effects include weight gain (clozapine
psychotics: causes highest weight gain amongst all antipsychotics),
constipation, anticholinergic side effects. A particularly
• Clozapine
• Olanzapine problematic side effect is sialorrhea or hypersalivation.
• Risperidone Clozapine can also cause life threatening side effects
• Paliperidone which include agranulocytosis, seizures and myocar¬
• Iloperidone ditis. In view of possibility of agranulocytosis, during the
• Quetiapine first six months of clozapine treatment, WBC and neu¬
• Ziprasidone trophil counts should be measured every week. Also, if
• Aripiprazole during the therapy, WBC counts fall below 3000/mm3
• Sertindole or neutrophil counts fall below 1500/mm3, the cloza¬
• Zotepine pine therapy should be stopped. The agranulocytosis and
• Lurasidone myocarditis are dose independent side effects of cloza¬
• Asenapine pine whereas seizures are dose dependent (seen only
• Amisulpride at higher dosages).
The side effect profile of atypical antipsychotics is as The only contraindication to clozapine use is a WBC
follows: count of less than 3500/dL at the time of starting cloza¬
A. Movement disorders: Atypical antipsychotics can pine, a history of agranulocytosis during clozapine treat¬
cause all kind of extrapyramidal side effects described ment or use of other drug that is known to suppress the
Schizophrenia Spectrum and Other Psychotic Disorders 17 1
bone marrow (e.g clozapine and carbamazepine cannot 3. Simple, disorganized, undifferentiated subtype
be given together as both are bone marrow suppressants. 4. Male sex
5. Prominent negative symptoms
Specific Points about Antipsychotics 6. Absence of affective symptoms
7. Family history of schizophrenia.
A. Long acting injectable antipsychotics (Depot anti¬
psychotics): In patients who have poor compliance0
with medications (i.e who refuse to take medications)
OTHER PSYCHOTIC DISORDERS _
long acting injectable antipsychotics can be used. A. Acute psychotic disorders: There are disorders which
Hie patients typically receives the intramuscular have symptoms (e.g. delusions, hallucinations and
injections of antipsychotics once a month or once a disorganisation symptoms) similar to schizophrenia,
fortnight. Long acting injectable preparations are however do not meet the duration criterion. These
available for following antipsychotics: disorders have been classified separately in DSM-5
• Flupenthixol and ICD-10. These disorders frequently are preceded
• Fluphenazine by a stressor (stressful life event), have an acute onset
• Haloperidol and often resolve completely. These disorders may
• Pipotiazine also be precipitated by fever0.
• Zuclopenthixol In ICD-10, if the symptoms (delusions, hallucina¬
• Risperidone tions, disorganization) are present for less than one
• Olanzapine month, a diagnosis of acute and transient psychotic
« Paliperidone
disorder is made.
• Aripiprazole In DSM-5, if symptoms (delusions, hallucinations,
B. Thioridazine can cause irreversible retinal pigmenta¬ disorganisation) are present for less than one month,
tion0. Thioridazine can also cause cardiac arrhyth¬ a diagnosis of brief psychotic disorder is made; and
mias0 (prolongation of QT interval). It is also the if symptoms last between 1-6 months, a diagnosis of
drug with least extrapyramidal side effects amongst schizophreniform disorder is made.
typical antipsychotics, overall clozapine is the anti¬ Treatment: Antipsychotics and benzodiazepines are
psychotic with least extrapyramidal side effect. used for the treatment of acute psychotic disorders.
C. Chlorpromazine can cause corneal and lenticular B. Schizoaffective disorder: Schizoaffective disorder has
deposits0. features of both schizophrenia and mood disorders
D. Penfluridol is the longest acting antipsychotic0.
concurrently. Depending on whether manic episode or
E. Ziprasidone is known to cause cardiac arrhythmias
depressive episode is present along with schizophrenia
(prolonga- tion of QT interval).
symptoms, there are two subtypes:
F. Aripiprazole is a partial agonist at D2 receptors (all
• Schizoaffective disorder (Bipolar type or manic
other antipsychotics are D2 antagonists).
type): With manic symptoms
• Schizoaffective disorder (Depressive type): With
PROGNOSIS_ depressive symptoms.
Good prognostic factors: Treatment: It involves combination of mood stabilis¬
1. Acute or abrupt onset ers, antipsychotics and antidepressants depending
2. Late onset (age > 35 years)0 on the presentation. In schizoaffective (manic type
3. Catatonic subtype and paranoid subtype episodes) a combination of antipsychotics and mood
4. Female sex stabiliser is commonly used. In schizoaffective
5. Prominent positive symptoms (depressive type episodes) a combination of antipsy¬
6. Presence of affective symptoms (such as depression0) chotics, and antidepressants is often used.
7. Family history0 of mood disorder. C. Delusional disorder: These disorders are characterized
Bad prognostic factor: by development of either a single delusion or a set
1. Insidious onset of related delusions, which are usually persistent and
2. Early onset (age <20 years) sometimes are life long. Other psychotic symptoms
18

disorder:

replaced by a
stranger who
looks exactly

QUESTIONS_
History

A. Freud
C. Kraepelin

C. Hecker

A. Kahlbaum
C. Maxwell
.
Review of Psychiatry

like hallucinations, disorganization, negative symp¬


toms are usually absent. If hallucinations occur they
are for a very short duration, presence of frequent hal¬
lucinations goes against the diagnosis of delusional
disorder. The following are the types of delusional

• Persecutory type: Delusion of persecution.


• Jealous type: Delusion of infidelity.
• Erotomanic type: Delusion of love.
• Somatic type: Patient may have delusion that he is
infested by parasites (delusional parasitosis), that
he has misshaped body parts (delusion of dysmor-
phophobia) or that his body has a foul odor (delu¬
sion of halitosis).
• Grandiose type: Delusion of grandiosity.
• Unspecified type: In patients where the above men¬
tioned categories are not applicable. Delusion of
misidentification is an example of unspecified type.
Delusion of misidentification can be of many types
like:
- Capgras syndrome: Patient believes that a
familiar person has been replaced by an impos¬
tor. For example, a patient believed that his
wife has been W-

.
— .....
DSM-5 update: The DSM-4 requi¬
red that the delusions should be non
bizarre, however DSM-5 has removed
this condition from the diagnosis of
delusional disorders.

QUESTIONS
....
...... AND ANSWERS

1. The term "Dementia precox" was coined by:

B. Bleuler
D. Schneider

2. The term schizophrenia was coined by:

A. Eugen Bleuler
......

(AI2008)

(DNBNEET2014-15)
B. Emil Kraepelin
D. Kurt Schneider
3. The term "catatonia" was coined by:
B. Freud
D. Adler
1
-
- Fregoli syndrome: Patient believes that familiar

-
persons are taking the guise of strangers. For
example, a patient saw a beggar, and claimed
that his brother is following him in the guise of
the beggar.
Syndrome of inter metamorphosis: Patient
believes

a
that people can undergo changes in
physical and psychological identity and become
different person altogether.
Syndrome of subjective doubles: Patient believes
that he has many doubles who are living life of
their own.
D. Shared psychotic disorders (or induced delusional
disorder): This disorder is characterized by spread of
delusions from one person to another. The individual
who has the delusion (the primary case) is typically
the influential member of close relationship with a
more suggestible person (the secondary case) who
also develops the delusion. When two people are
involved, the term "folie a deux" is used. Occasionally
more than two individuals are involved (known as
folie a trois, folie a quatre, etc).
The patients of delusional disorder are usually able
to function normally in domains which are unaf¬
fected by the delusion. For example, a patient with
delusion of infidelity may incessantly doubt his wife
and fight with her, however he may be perfectly nor¬
mal at work place.
Treatment: Antipsychotics are the drug of choice.

D. Loosening of association
E. Ambivalence
I

4. Bleuler's symptoms of schizophrenia include all


except:
A. Autism
B. Automatism
C. Affect disturbance
(PGIDec 2005)

5. Schneiderian First Rank Symptoms are found in:

A.
B.
C.
D.
E.
Schizophrenia
Organic mental disorders
Schizoaffective disorder
Mood disorder
Delusional disorder
(PGINov 2011)
Schizophrenia Spectrum and Other Psychotic Disorders 19 1
Epidemiology 13. Schizophrenia is characterized by all of the fol¬
lowing symptoms except: (All993)
6. Schizophrenia is associated with which of the A. Delusion of reference B. Delusion of control
following personalities: (AIIMS 1997) C. Waxy flexibility D. Altered sensorium
A. Athletic B. Pyknic
14. The characteristic clinical manifestationof schizo¬
C. Asthenic D. All of the above
phrenia are: (PGI 1998)
7. True about late onset schizophrenia: A. Confusion
(AIIMS Nov 2010) B. Anxiety
A. Onset is after 45 years C. Auditory hallucinations
B. Onset is between 25-30 years D. Visual hallucinations
C. Prognosis is poor 15. Which of the following hallucinations is pathog¬
D. Olfactory hallucinations are common nomonic of schizophrenia? (AIIMS 2K, Delhi2003)
8. Maximum heritability is seen inwhich of the fol¬ A. Auditory hallucinations commanding the patient
lowing illness: B. Auditory hallucinations giving running com¬
(DNB 2005., MP 2004, WB 2003, UP 2001) mentary
C. Auditory hallucinations criticising the patient
A. Depression B. Mania
D. Auditory hallucinations talking to the patient
C. Schizophrenia D. Panic disorder
16. All of the following are characteristic symptoms
Etiology and Pathogenesis of schizophrenia except:
(AIIMS Nov 2007, MCIScreening)
9. Neurotransmitter related to the pathology of A. Third person hallucinations
schizophrenia is: (PGI1997) B. Inappropriate emotions
A. Acetylcholine B. Dopamine C. Sustained mood changes
C. Serotonin D. Norepinephrine D. Formal thought disorder

10. Blood sample of a 45 years old male shows 17. Hallucinations inschizophrenia are characterized
increased levels of homovanillic acid (HVA). This by all of the following except:
patient is most likely suffering from: A. Hallucinations commanding and controlling
(AIIMS Nov 2008) action of the person
A. Dementia B. Schizophrenia B. Hallucinations of voices, singing songs
C. Depression D. Parkinson's disease C. Hallucinations are almost always continuous
D. Hallucinations commenting on action of the
.
11 Schizophrenia is caused by overactivity in which person
of the fol- lowing dopaminergic systems?
18. Which of the following sign is not a part of catato¬
(DNB 2007)
nia? (AIIMS May 2015)
A. Nigrostriatal pathway
A. Akathisia B. Ambivalence
B. Tuberoinfundibular pathway C. Ambitendency D. Akinesia
C. Mesolimbic/Mesocortical pathway
D. None of the above 19. All of the following are features of catatonia except:
(DNB NEET 2014-15)
A. Automatic obedience B. Cataplexy
Symptoms and Diagnosis C. Catalepsy D. Negativism
12. Schizophreniaischaracterizedbyallofthefollowing 20. The following are features of catatonic schizophre¬
symptoms except: (AIIMS 1998, 2000)
nia, except: (MP 2000)
A. Delusion A. Mutism
B. Auditory hallucination B. Echolalia
C. Elation C. Waxy flexibility
D. Catatonia D. Deep tendon reflexes are increased
lH 20 Review of Psychiatry

.
21 Incatatonic schizophrenia, which of the following B. Affective symptoms
sign is not found: (PGIDec 2008) C. Emotional blunting
A. Waxy flexibility D. Insidious onset
B. Automatic obedience
29. All of the following are associated with better
C. Somatic passivity
prognosis in schizophrenia except:
D. Gegenhalten
(AI2006, MCIScreening)
E. Hallucinations
A. Late onset B. Married
22. True about schizophrenia: (PGI2003) C. Negative symptoms D. Acute onset
A. Thought broadcasting
30. Prognosis of schizophrenia is less favorable inthe
B. Third person hallucinations
following clinical scenario: (MCIScreening)
C. Violent behavior
D. Elated mood A. Occurring in women
E. Good self care B. Anxiety is prominent
C. Emotional blunting is present
23. All of the following are true about paranoid schizo¬ D. In presence of rapid onset of psychosis
phrenia except: (MP 1997)
A. Most common type of schizophrenia 31. Type two schizophrenia is characterized by all of
B. Onset in 3rd/4th decade the fol- lowing features except: (AIIMS Nov 2008)
C. Delusion of grandeur is a symptom A. Negative symptoms
D. Rapid deterioration of personality . B. Poor response to treatment
C. Disorganised behavior
24. Defect of conation is typically seen in: D. CT scan abnormalities
(PGI 1997, AIIMS 1996, UP 2006)
A. Simple schizophrenia 32. Van Gogh syndrome is seen in: (PGI2003)
B. Hebephrenic schizophrenia A. Mania B. Depression
C. Catatonic schizophrenia C. Schizophrenia D. OCD
D. Paranoid schizophrenia 33. Which of the following is the most common cause
25. Waxy flexibility is a characteristic sign of: of premature death in schizophrenia? (AI2011)
(Orissa 2004, Jharkhand 2006) A. Homicide
A. Excitatory catatonia B. Suicide
B. Stuporous catatonia C. Toxicity of antipsychotic drugs
C. Obsessive compulsive disorder D. Hospital acquired infections
D. All of the above 34. Expressed emotionality is related to which of the
26. Early onset and bad prognosis is seen in: following illnesses: (MH2010)
(AIIMS 1991) A. Depression
A. Catatonic schizophrenia B. Schizophrenia
B. Hebephrenic schizophrenia C. Mania
C. Paranoid schizophrenia D. Somatoform disorder
D. Undifferentiated schizophrenia
27. Schizophrenia with late onset andbest prognosis: Clinical Vignettes
(DNBNEET2014-15) 35. A patient of Schizophrenia was started on neuro¬
A. Simple schizophrenia leptics, his psychotic symptoms began to improve
B. Hebephrenic schizophrenia however he developed sadness, would talks less to
C. Catatonic schizophrenia
others ,would mostly remain on bed.This presen¬
D. Paranoid schizophrenia
tation could be caused by all of following except:
28. Goodprognosis inschizophrenia is indicated by: (AIIMS2000)
(PGI1998) A. Parkinsonism
A. Soft neurological signs B. Major depression
Schizophrenia Spectrum and Other Psychotic Disorders 21 |
C. Negative symptoms are still persisting talking to himself and sometimes laughingloudly.
D. He is reacting to external stimuli The likely diagnosis is: (AIIMS May 2002)
A. Schizophrenia B. Conversion disorder
36. Kallu, a 24-year-old occasional alcoholic was C. Major depression D. Delusional disorder
brought to psychiatry OPD with a history of
behavioralchanges.According to family members, 41. A 16-year-oldboy does not attend schoolbecause
hehasbecome suspicious that people are trying to of the fear of being harmed by school mates. He
conspire against him, though his father states that thinks that his classmates laugh at him and talk
there is no reason for his fears. Kallu also reports about him. He is even scared of going out to the
of hearing voices that comment on his actions. market. He is most likely suffering from:
What is the most probable diagnosis:fAf/MS 2000) (AI2004)
A. Delirium tremens A. Anxiety disorder
B. Alcohol induced psychosis B. Manic depressive psychosis (bipolar disorder)
C. Schizophrenia C. Adjustment reaction
D. Delusional disorder D. Schizophrenia
37. A 70-year-old male, Babulal was brought to the
hospitalwith the history of third person auditory
Treatment
hallucinations. He has no history of similar prob¬ 42. Depot preparations are available for:
lems previously. What is the most likely diagno¬ (PGINov 2010)
sis? (AIIMS 2001) A. Haloperidol B. Risperidone
A. Dementia B. Delusional disorder C. Olanzapine D. Imipramine
C. Schizophrenia D. Acute psychosis E. Fluphenazine
38. A 60-year-old manis brought to a psychiatrist with 43. A 23-year-old boy with schizophrenia is well-
a 10-year history, that he suspects his neighbors maintained on risperidone for the last 2 months.
and he feels that whenever he passes by they He has no family history of the disease. For how
sneeze and plan against him behindhis back. He long will you continue treatment in this patient?
feels that his wife has been replaced by a double (AIIMS Nov 2015)
and calls police for help. He is quite well-groomed, A. 5 years B. 6 months
alert, occasionally consumes alcohol, likely diag¬ C. 2 years D. 12 months
nosis is: (AIIMS May 2002)
A. Paranoid personality disorder 44. A person with violent behavior and agitation was
B. Paranoid schizophrenia diagnosed to have schizophrenia and was started
C. Alcohol withdrawal onhaloperidol.Followingthis he developed rigidity
D. Conversion disorder and inability to move his eyes. Which of the follow¬
ingdrugs shouldbe added to his treatment intrave¬
39. Lallo, a 40-year-old malehas recently started writ¬ nously for this condition? (AIIMS May 2015)
ingbooks. But the matter inhisbook could not be A. Promethazine B. Elaloperidol
understood by anybody since it containedwords C. Risperidone D. Diazepam
which were never there in any dictionary and the
theme was very disjoint. Nowadays he has become 45. Antipsychotic drugwith least incidence of extrapy¬
very shy and self absorbed. When he addresses ramidal side effects is: (DNB NEET2014-15)
people he speaks about meta philosophical ideas. A. Pimozide B. Thioridazine
What is the likely diagnosis? (AIIMS 2000) C. Clozapine D. Chlorpromazine
A. Mania B. Schizophrenia
46. Not true about clozapine is: (AI-2012)
C. A genius writer D. Delusional disorder
A. Should be discontinued, if WBC counts <3000/
40. A patient is brought with 6 months history of odd mm3
behavior. There is history of a family member hav¬ B. Blood levels should be maintained <350 ng/mL
ing disappeared some years back. He seems to be to avoid agranulocytosis
22 Review of Psychiatry

C. Should not be used along with carbamazepine C. Addition of beta-blocker


D. The action is more on D4 receptors than D2 D. Adding another antipsychotic drug
receptors
52. Akathisia is treated by all except: (AT 1994)
47. A patient of schizophrenia on clilorpromazine A. Trihexyphenidyl B. Diazepam
(CPZ) develops auditory hallucination again. The C. Haloperidol D. Promethazine
next drug to be given is: (AI2000)
53. A psychotic patient on antipsychotic drugs deve¬
A. Haloperidol B. Clozapine
lops torticollis within 4 days of starting therapy.
C. Sulpiride D. Tianeptin
What is the appropriate medication that should
48. A patient with acute psychosis, who is on halo¬ be added in the treatment regimen?
peridol 20 mg/day for last 2 days, has an episode (DNBNEET 2014-15)
characterized by tongue protrusion, oculo¬ A. Central anticholinergic
gyric crisis, stiffness and abnormal posture of B. Peripheral anticholinergic
limbs and trunk without loss of consciousness C. Beta-blocker
for last 20 minutes before presenting to casualty. D. Dantrolene
This improved within a few minutes after
54. A patient who is talcing antipsychotics for 3 weeks,
administration of diphenhydramine HC1. The
presents with high grade fever, raised CPK and
most likely diagnosis is: (AIIMS 2011, May 2006)
myoglobinuria. What is the most probable diag¬
A. Acute dystonia
nosis? (DNBNEET2014-15)
B. Akathisia
A. Neuroleptic malignant syndrome
C. Tardive dyskinesia
B. Tardive dysldnesia
D. Neuroleptic malignant syndrome
C. Acute dystonia
49. 16-year-old boy who was started on an antipsy¬ D. Akathisia
chotic drug, presents with sudden onset of torti¬
55. A 31-year-old male, with mood disorder, on 30 mg
collis. What is the most probable diagnosis?
of haloperidol and 100 mg of lithium, is brought
(DNBNEET2014-15)
to the hospital emergency room with history of
A. Acute muscular dystonia
acute onset of fever, excessive sweating, confusion,
B. Akathisia
rigidity of limbs and decreased communication
C. Neuroleptic malignant syndrome for a day. Examination reveals tachycardia and
D. Tardive dyskinesia labile blood pressure and investigations reveal
50. Anelderlywomansufferingfromschizophreniaison increased CPK enzyme levels and leucocytosis.
antipsychotic medication. She developed pur¬ He is likely to have developed: (AIIMS May 2004)
poseless involuntary facial and limb movements, A. Lithium toxicity
constant chewing and puffing of cheeks. Which of B. Tardive dyskinesia
the following drugs is least likely to be involved in C. Neuroleptic malignant syndrome
this side effect? (AIIMS Nov 2003) D. Hypertensive encephalopathy
A. Haloperidol B. Clozapine 56. A patient was on treatment with trifluoperazine
C. Fluphenazine D. Loxapine for some time. He presents with complaint of
51. A 19-year-old boy suffering from chronic schizo¬ hyperthermia, lethargy and sweating. Needed
phrenia is put on haloperidol at the dose of 20 mg/ investigations are: (AIIMS 2000)
day. A week after the initiation of medicationthe A. CT scan brain and hemogram
B. Hemogram, electrolyte level and creatinine
patient shows restlessness, fidgetiness, irritability
C. ECG, chest X-ray and hemogram
and cannot sit still at one place. The most appro¬
D. Hemogram, CPK and renal function test
priate treatment strategy is: (AIIMS May 2004)
A. Increase in the dose of haloperidol 57. Which of the following is a symptom of neuroleptic
B. Addition of anticholinergic drug malignant syndrome? (DNB NEET 2014-15)
Schizophrenia Spectrum and Other Psychotic Disorders 23 [_

A. Hypotension 65. Alcoholic paranoia is associated with: (AT 2010)


B. Hypothermia A. Fixed delusions B. Hallucinations
C. Increased magnesium level in blood C. Drowsiness D. Impulsivity
D. Catatonia and stupor 66. Delusion of doubles is seen in: (AIIMS 1999)
58. What is produced by the supersensitivity of dopa¬
A. Schizoaffective disorder
B. Capgras syndrome
mine receptors? (DNB NEET 2014-15)
C. Reactive psychosis
A. Dyskinesia B. Hyperphagia
D. Paranoid schizophrenia
C. Hyperpathia D. Hypomania
67. Characteristic symptom in induced psychotic
59. Drug of choice for treatment of neuroleptic (AIIMS 1992)
disorder is:
malignant syndrome is: (DNB NEET 2014-15) A. Insomnia
A. Dantrolene B. Profound mood disturbance
B. Beta-blockers C. Accepting delusions of other person
C. Central anticholinergics D. Suicidal ideation
D. None of the above
68. A person aged 35 years is having firm belief about
60. A young patient of schizophrenia is intolerant to infidelity involving the spouse. He never allows
antipsychotic medications. Which drug is most her to go out of home alone. He often locks his
preferred? house, while going to the office. Inspite of all this,
A. Clozapine B. Olanzapine he is persistently suspicious about the character
C. Risperidone D. Haloperidol of his wife. The probable diagnosis is:
(AIIMS 1999)
61. Antipsychotic drug causing retinal pigmentation A. Schizophrenia
disorder is: (DNB NEET 2014-15) B. Delusional parasitosis
A. Thioridazine B. Clozapine C. De Clerambault's syndrome
C. Chlorpromazine D. None of the above D. Othello syndrome
62. In comparison to haloperidol, clozapine causes 69. Basanti, 27-year-female thinks that her nose is
more: (PGIMay 2015) ugly. Her idea is fixed and is not shared by anyone
A. Weight gain else. Whenever she goes out of the home, she hides
her face? She visits a surgeon for plastic surgery.
B. Agranulocytosis
The appropriate next step would be: (AI2001)
C. Sedation
A. Investigate and then operate
D. Severe extrapyramidal symptoms
B. Reassure the patient
E. Less epileptogenic potential
C. Immediate operation
63. Cognitive remediation is used for: (AIIMS 2013) D. Refer to psychiatrist
A. Cognitive restructuring 70. A 41-year-old woman working as an executive in
B. Memory improvement a company is convinced that the management
C. Correcting cognitive distortion has denied her promotion by preparing false
D. Improving study habits reports about her competence andhave forged her
signature on sensitive documents so as to convict
Other Psychotic Disorders her. She files a complaint in the police station and
requests for security. Despite all this she attends
64. What is the content of most common type of per¬ to her work and manages the household. What is
sistent delusional disorder? (DNB NEET 2014-15) the most likely diagnosis? (AI2004)
A. Delusion of persecution A. Paranoid schizophrenia
B. Somatic delusion B. Late onset psychosis
C. Delusion of jealousy C. Persistent delusional disorder
D. Delusion of grandeur D. Obsessive compulsive disorder
24 Review of Psychiatry

71. A 30-year-old unmarried woman from a low 5. A, B, C, D, E.


socioeconomic status family believes that a rich Although, SFRS were described in relation to
boy staying in her neighborhood is in deep love schizophre-nia, however they are not specific
with her. The boy clearly denies his love towards to schizophrenia. They can be found in all the
this lady. Still the lady insists that his denial is disorders mentioned in this question. However,
a secret affirmation of his love towards her. She if it was not a PGI question, and only option had
makes desperate attempts to meet the boy despite to be chosen, it would be schizophrenia.
resistance from her family. She also develops sad¬ 6. C.
ness at times when her effort to meet the boy does 7. A
not materialize. She is able to maintainher daily 8. C. Schizophrenia has more heritability than bipolar
routine. She however, remains preoccupied with disorder (mania) which in turn has higher heri¬
the thoughts of this boy. She islikelyto be suffering tability in comparison to depression.
from: 9. A, B, C, D.
(AI2004)
A. Delusional disorder All have been implicated but most important are
B. Depression
C. Mania
dopamine and serotonin.
D. Schizophrenia
10. B. The HVA is a metabolite of dopamine and
72. A20-year-oldboycomplaintsofhearingofvoicesand dopamine is usually increased in schizophrenia.
aggressive behavior for last 2 days. He had fever A large number of studies have found that levels
before the onset of these symptoms. The family of HVA are increased in schizophrenia.
members report that he has beenmutteringto self 11. C.
and gesticulating as if he is talking to someone. 12. C.
There is no history of any past psychiatric illness. 13. D. Pleaseremember altered sensorium (or clouding
The likely diagnosis is: (AIIMS Nov 2010) of consciousness) is a sign of delirium. This is a
A. Dementia B. Acute psychosis frequently repeated question.
C. Delirium D. Delusional disorder 14. C. Auditory hallucinations are the most common
type of hallucinations in schizophrenia and the
73. A 30-year-old manhasbecome suspicious that his third person auditory hallucinations are quite
wife is having an affair with hisboss. He thinks his characteristic for schizophrenia.
friend is also involved from abroad andis provid¬ 15. B. Actually, the correct answer is none. No single
ingtechnology support. He also thinks that people symptom or sign is pathognomonic of schizo¬
talk illabout him.His friends tried to convince him phrenia. However, earlier, the Schneider's first
but failed to do so. The patient otherwise is normal, rank symptoms were considered to be patho¬
doesn't have any thought disorder or any other gnomonic. Hence the best answer here is B.
inappropriate behavior. The most likelydiagnosis 16. C. Formal thought disorder, third person hallucina¬
is: (AI2010) tions and inappropriate emotions (inappropriate
A. Paranoid personality disorder affect) are characteristic of schizophrenia.
B. Persistent delusional disorder 17. C. Hallucinations in schizophrenia are usually not
C. Schizophrenia continuous.
D. Obsessive compulsive disorder 18. A. Akathisia is a side effect of antipsychotics. Ambi¬
valence might be confusing here, but please
ANSWERS _ remember ambitendency is nothing but ambi¬
valence of motor movements. Akinesia, which
1. c. is lack of voluntary movements is another term
2. A. for stupor.
3. A. 19. B. Cataplexy is a feature of narcolepsy.
4. B. Automatisms are usually a feature of epilepsy. 20. D.
They are apparently meaningful behaviors, for 21. C, E.
which patient doesn't have any memory later The other three options are classical catatonic
on. It is not seen in schizophrenia. signs. While in catatonic schizophrenia, hallu-
Schizophrenia Spectrum and Other Psychotic Disorders 25 fl
cinations and delusions can be seen, however 38. B. Kindly note that this patient also has Capgras
they are not prominent. syndrome (feeling that his wife has been
22. A, B, C. replaced by a double) which is usually seen in
Schizophrenic patients are much more likely to patients with schizophrenia. Also the history is
engage in violent acts in comparison to those suggestive of delusion of persecution (neighbors
without schizophrenia. are planning against him). Also, note that he is
23. D. quite well groomed suggesting that personality
24. C. is preserved as is seen in patients with paranoid
25. B. Stuporous catatonia has stupor as a prominent schizophrenia.
symptom. Waxy flexibility is seen in stuporous 39. B. The history is suggestive of neologisms (words
catatonia more commonly. which are not present in any dictionary) and
26. B. formal thought disorders (theme is very disjoint).
27. D. The best prognosis is of catatonic schizophrenia. Further, there are negative symptoms (shy and
However in this question, the better answer is self absorbed). All point towards the diagnosis
paranoid schizophrenia, as it is the one which of schizophrenia.
has both late onset and good prognosis. 40. A There is history of disorganised behavior (odd
28. B. Presence of affective symptoms (manic or behavior), hallucinations (talking to self and
depressive) is a good prognostic sign. laughing loudly is most likely a result of patient
29. C. hearing some voices and communicating with
30. C. Emotionalbluntingis quite similar to affective flat¬ the voices), the history of disappeared family
tening and hence is a negative prognostic sign. member is again suggesting that some family
31. C. member may have had a mentalillness because
32. C. of which either he got lost or committed suicide.
33. B. Suicide is the most common cause of premature All factors combined, the likely diagnosis is
death.Around 5- 10% of patients with schizophre¬ schizophrenia.
nia commit suicide. 41. D. The history is suggestive of delusion of persecu¬
34. B. "Expressed emotions" is a term which is used tion (fear that schoolmates may harm him) and
to describe certain attitudes of family members delusion of reference (belief that classmates
of patients with schizophrenia, which have an laugh at him and talk about him).
impact on the illness itself. These attitudes 42. A, B, C, D, E.
includes over involvement, hostility, passing There is a depot preparation available for imipra-
critical comments, etc. mine, which is an antidepressant.
35. D. Kindly remember that the negative symptoms 43. C. The history is suggesting that the patient had
of schizophrenia have a similar presentation first episode of schizophrenia (i.e he developed
as depression. Speaking less, staying on bed schizophrenia for the first time and no history
mostly can be due to either negative symptoms or of any relapse has been provided) and is now
depression. Further, the use of antipsychotics can maintaining well for last two months. It is gene¬
cause drug induced parkinsonism which again rally recommended that after first episode, the
looks quite similar to negative symptoms. treatment with antipsychotics should be conti¬
36. C. There is history of delusions and auditory hal¬ nued for at least two years. If there are more than
lucinations, (running commentary type). Hence, one episodes (i.e. there is history of relapses) the
the diagnosis is most likely schizophrenia. Occa¬ treatment should continue for at least 5 years. In
sional alcohol use is unlikely to cause psychosis. patients with multiple relapses, indefinite treat¬
37. C. Third person auditory hallucination is suggestive ment is given.
of schizophrenia. Although its an incomplete 44. A. The symptoms are suggestive of acute dystonia
question, and information about duration of (inability to move eyes is most likely due to oculo¬
symptoms, any memory disturbances would gyric crisis) and drug induced parkinsonism
have helped in making a more definitive diag¬ (development of rigidity). For both, an anticho¬
nosis. linergic needs to be added.
26 Review of Psychiatry

45. C. (primary case) induces the delusion in another


46. B. Agranulocytosis is an idiosyncratic reaction and person (secondary case).
is not related to blood levels. 68. D. Here, there is only one delusion, i.e. delusion of
47. B. Clozapine. This question intends to give history infidelity, also known as Othello syndrome.
for treatment resistance schizophrenia. However, 69. D. This appears to be a case of delusional disorder,
treatment resistance schizophrenia is defined as
somatic type. Its important to differentiate it
lack of response to two different antipsychotics
from body dysmorphic disorder. Here, the ques¬
(belongingto different chemical classes) whereas
tion says that the idea is fixed (fixed means that
in this question only one antipsychotic has been
used.This might bedue to wrong recall of question. the belief persists despite evidences to contrary
48. A. and despite reassurances by others) and is not
49. A. shared by anyone else and patient is further
50. B. The history here is suggestive of tardive dyskine¬ hiding her face when visiting outside (i.e. acting
sia. Clozapine is the antipsychotic with minimum on her belief). Inbody dysmorphic disorder, the
incidence of tardive dyskinesia. belief is not fixed and may be at least temporarily
51. C. The history here is suggestive of akathisia. changed by reassurances of others. In body
52. C.
dysmorphic disorder, the problem is more of a
53. A.
preoccupationwith the thought that a body part
54. A.
is deformed, this preoccupation is however not
55. C.
fixed (which means person can be reassured at
56. D. Here, we need to rule out the neuroleptic
malignant syndrome and also check the renal least for some time).
functions (as NMS can result in renal failure 70. C. In this question, there is a single delusion that
secondary to myoglobinuria). management is against her (delusion of perse¬
57. D. The symptoms of NMS are quite similar to cata¬ cution) and her actions are according to that
tonia (increased rigidity, stupor). Infact, cata¬ delusion. Please remember that in delusional
tonia is an important differential diagnosis in disorders, the areas of functioning which does
patients with NMS. not involve the delusion, remain unaffected. In
58. A. Itisbelievedthatlong-termblockadeofD2recep- this patient also, the history that she is able to do
tors by antipsychotics causes super sensitivity of
her work and manage household is suggesting
the receptors which results intardive dyskinesia.
that she is able to manage the areas of her life
59. A.
which are not affected by the delusion. In ques¬
60. A. In patients who are intolerant to the extrapy¬
ramidal side effects, clozapine is the preferred tions of delusional disorder, this history is very
antipsychotic. important and should be looked for.
61. A. 71. A.
62. A,B,C. 72. B. The duration of symptoms is less than one month.
63. B. Cognitive remediation is a therapy usually used Also please remember that in a large number of
in schizophrenia for improvement of cognitive cases acute psychosis is preceded by fever, hence
functions such as attention, concentration, don't get confused. Inthis case if the history also
memory, planning and execution. mentioned disturbances of consciousness or his¬
64. A.
tory of disorientation, the likely diagnosis would
65. A. Alcoholic paranoia usually presents with
be delirium.
delusion of infidelity (also known as morbid
73. B. Here again, there is a central delusion that wife is
jealousy).
having an affair, and the rest of history is exten¬
66. B. Please remember delusion of doubles is also
sion of that delusion (i.e. friend is providing
known as Capgras syndrome and is usually seen
support and people are talking ill). The question
in patients with schizophrenia.
has mentioned the lack of any thought disorder
67. C. In Induced psychotic disorder or shared psy¬
and inappropriate behavior to provide evidence
chotic disorder, one person who has the delusion against the diagnosis of schizophrenia.
; • ÿ
ÿ

,
.

Chapter

3 Mood Disorders

Mood disorders are so called as their main feature is Symptoms


abnormality of mood. They are also sometimes referred
The symptoms of depression can be remembered using
to as affective disorders. Mood disorder include:
the pneumonic, SIGECAPS. These symptoms must last for
A. Major depressive disorder (or Unipolar depression or
more than two weeksQ, for the diagnosis of depression.
Depression): Patients have only depressive episodes.
A. Sleep disturbances: Usually insomnia is seen however
B. Bipolar disorder: Patients have both manic and
hypersomnia can also be a symptom. The two char¬
depressive episodes or may present with manic epi¬
acteristic sleep disturbances in depression are "early
sodes only.
morning awakening0" and "reduced latency of REM
C. Hypomania: It is an episode similar to mania, how¬
sleep"
ever is less severe, and does not meet the criterion for
B. Interest (loss): Patient looses interest in the activities
mania.
which used to interest him earlier (anhedonia).
D. Cyclothymia: It is a less severe form of bipolar disorder.
C. Guilt: Patient may have excessive guilt feelings and
E. Dysthymia: Less severe and chronic form of major
may blame himself for trivial matters.
depression.
D. Energy (lack): Patient may have decreased energy

DEPRESSION _ levels and easy fatigability.


E. Cognition/Concentration: Patient may have negative
Various terms such as major depressive disorder, unipolar cognitions (negative thoughts) and may have poor
depression and depression have been used for the same concentration.
illness. It is characterized by major depressive episodes F. Appetite: Usually the appetite and weight are lost, in
(also known as depressive episodes) in the absence of any some patients weight gain may be seen.
manic, mixed or hypomanic episodes. G. Psychomotor agitation or retardation: The term 'psy¬
It is one of the commonest psychiatric disorder (life¬ chomotor' refers to the changes in motor activity
time prevalence is 17%). It is twice as prevalent in women secondary to psychological causes. It may be increased
as in men and the mean age of onset is around 40 years (e.g. in restless patients) or may be decreased (e.g. a
(so its most commonly seen in middle aged females). patient who keeps on lying on the bed and rarely gets
It is more commonly seen in divorced and separated up).
persons. H. Suicidal thoughts and sadness of mood (depressed
Depression is also responsible for maximum DALYs mood).
(disability adjusted life years) amongst all the psychia¬ Out of the above symptoms, SWAG (suicidality, weight
tric disorders. It is also the most commonQ cause of loss, anhedonia and guilt feelings) are quite suggestive of
suicide. depression.
ÿ 28 Review of Psychiatry

Physical Signs retardation and feelings of guilt and higher suicide risk.
The symptoms of endogenous depression were quite
A. Veraguth foldQ: Otto veraguth described a triangular similar to today's psychotic and melancholic depres¬
shape fold in the nasal corner of upper eyelid, called sion. The exogenous depression (reactive depression)
veraguth fold in patients with depression. was believed to occur in response to a negative life
B. Omega sign: It is the omega shaped fold (like the event and the symptoms which were described included
Greek letter omega, O ) in the forehead above the root initial insomnia (difficulty in falling asleep), absence of
of the nose, seen in patients with depression. psychotic symptoms and multiple somatic complaints
Apart from the above mentioned symptoms, the and lower suicide risk.
patient may have few other special features which must
be mentioned along with the diagnosis, such as psychotic Etiology
features, atypical features, melancholic features and cata¬
A. Biologicalfactors:
tonic features. These have been described below:
• Neurotransmitters disturbances: Decreased lev¬
A. With psychotic features: Patients with severe depres¬
els of serotonin and norepinephrine9 are most
sion may develop psychotic symptoms (delusions and
important factors implicated in the pathophysio¬
hallucinations). These psychotic symptoms could be
logy of depression. Dopamine has also been found
mood congruent (i.e. content of delusion/halluci¬
to be decreased in a subset of patients.
nation is consistent with the depressed mood, e.g. a
• Hormonal disturbances: Elevated HPA activity
severely depressed patient developed a delusion that
(hypothalamic-pituitary-adrenal axis activity) has
the world is about to end, nihilistic delusion) or mood
been documented. Also, hypothyroidism9 is a
incongruent (i.e. content of delusion is inconsistent common cause of depression.
with the depressed mood, e.g. a severely depressed
• Neuroanatomical considerations: Decreased acti¬
patient developed the delusion that he is the richest
vity in dorsolateral prefrontal cortex9 and
man on earth). It must be remembered that psychotic
increased activity in amygdala (and other limbic
symptoms are present only in severe depression and tissue) has been found in depression.
sometimes the term "psychotic depressionQ" is used
B. Genetic factors: Gene mapping studies have found
for depression with psychotic features. evidence of linkage to locus for cAMP response ele¬
B. With atypical features: These patients present with ment binding protein (CREB 1) on chromosome 2.
reverse biological symptoms such as hypersomnia, Serotonin transporter gene has also shown linkage.
overeating and weight gain. C. Psychological theories:
C. With melancholic features: Depression with melan¬
• Cognitive theory: It was proposed by Aaron Beck9.
cholic features (or involutional melancholia9) is According to this theory negative thoughts have a
usually seen in old age. It is characterized by severe central role in development of depression. He pro¬
anhedonia, profound guilt feelings, early morning
posed that there are three central thoughts/ideas in
awakening and weight loss, agitation and high sui¬ depression, the so called cognitive triad of depres¬
cide risk. sion9. These include:
D. With catatonic features: Patient with depression may
1. Negative view of self (ideas of worthlessness9)
develop catatonic symptoms such as stupor9, negati¬ 2. Negative views about environment— A tendency
vism, etc.
to experience world as hostile (ideas of help¬
lessness9) and
Endogenous vs Exogenous (Reactive) Depression 3. negative view about future (ideas of hopeless¬
In older classificatory system, two subtypes of depres¬ ness).
sion were described. Endogenous depression which • Learned helplessness: According to this theory, due
occurred in the absence of any precipitating negative to repeated adverse events, patient starts believ¬
life event, and was considered to be caused by biologi¬ ing that he has no control over events happening
cal factors. The symptoms described were early morn¬ around him and loses the motivation to act which
ing awakening, psychotic symptoms, psychomotor results in depression.
Mood Disorders 29 |

Treatment channels. The blockage of sodium channels in


brain can cause seizures.
1. Pharmacotherapy. The use of specific pharmaco¬ • Sedation due to blockage of HI histaminic
therapy doubles the chances that a depressed patient receptors.
will recover in 1 month. All the available anti¬ • Important properties of individual drugs:
depressants take up to 3-4 weeks to exert significant a. Amoxapine0 has D2 blocking action and
therapeutic effects. The available antidepressants hence can cause extrapyramidal side effects
do not differ in the overall efficacy, speed of like antipsychotics.
response or long-term effectiveness and the choice b. Imipramine0 is used in the treatment of noc¬
of antidepressants is mostly determined by the side turnal enuresis (however the drug of choice
effect profile0 of the drugs. Antidepressant treatment is desmopressin; the treatment of choice is
should be maintained for at least 6 months or equal behavioral methods like night alarms).
to the duration of a previous episode, whichever is c. Clomipramine is the first line therapy in
greater. Prophylactic treatment with antidepressants OCD, however due to better side effect pro¬
is effective in reducing the number and severity of file, SSRIs are preferred over clomipramine.
episodes. It should be given to patients who have B. Selective serotonin reuptake inhibitors (SSRIs):
had three or more prior depressive episodes or who These are the most commonly prescribed antide¬
have chronic major depressive disorder (> 2 years pressants. They act by blocking the reuptake of
duration is chronic depression). The following classes serotonin and do not have problematic side effects
of medications can be used. seen with TCAs. The SSRIs include fluoxetine, flu-
A. Tricyclic and tetracyclic antidepressants (TCAs): voxamine, citalopram, escitalopram, sertraline,
These were the first class of antidepressants that paroxetine and vilazodone. The SSRIs are the first
were widely used in clinical practice. They act by line drugs for depression, obsessive compulsive
blocking the transporters of serotonin and norep¬ disorder, post-traumatic stress disorder, panic dis¬
inephrine and hence increase the levels of these order, generalized anxiety disorder and phobias.
neurotransmitters in synapses. Secondary effects The side effects of the SSRIs include nausea (most
of TCAs include antagonism of muscarinic, his- common) followed by anxiety and diarrhoea. Other
taminic HI, al and a 2 adrenergic receptors and side effects include delayed ejaculation (hence
blockage of cardiac sodium channels. These secon¬ SSRIs are used in the treatment of premature eja¬
dary effects are responsible for the unfavorable side culation), decreased libido, anorgasmia, sedation,
effect profile of these drugs. delayed platelet aggregation, sweating and weight
The class TCAs include the following drugs-. Imi- gain.
pramine, desipramine, trimipramine, amitriptyline, Serotonin syndromeQ: Concurrent administration
nortriptyline, protriptyline, amoxapine, doxepin, of an SSRI with MAO inhibitor, L-tryptophan or
maprotiline and clomipramine. The TCAs differ in lithium can raise plasma serotonin concentration,
their affinity for transporters, with clomipramine producing serotonin syndrome. The symptoms are
being the most serotonin selective and desipra¬ diarrhea, restlessness, hyperreflexia, myoclonus,
mine the most norepinephrine selective of TCAs. seizures, may result in death. It is treated using
The side effects of TCAs include the following: cyproheptadine and supportive care.
• Anticholinergic side effects0 like constipation, Vortioxetine: A recently introduced antidepres¬
urinary retention0, blurred vision, dry mouth, sant works as an inhibitor of serotonin reuptake,
decreased sweating and delirium. Due to signi¬ but also has other actions like agonism at 5-HT1A
ficant anticholinergic side effects TCAs should receptor, partial agonism at 5-HT1B receptor and
be avoided in glaucoma0 and prostate hypertro¬ antagonism at 5-HT3. 5-HT1D and 5-HT7 receptors.
phy. C. SNRIs (Serotonin Norepinephrine Reuptake Inhibi¬
• Side effects due to blockade of a (alpha) recep¬ tors): These drugs produce blockade of neuronal
tors like postural hypotension. serotonin and norepinephrine uptake transporters
• Severe side effects like cardiac arrhythmias, and hence are also referred as dual reuptake inhibi¬
hypotension due to blocking of cardiac sodium tors. They include venlafaxine, desvenlafaxine,
|30 Review of Psychiatry

duloxetine, milnacipran, levomilnacipran. The side of reuptake of both norepinephrine and dopa¬
effect profile is quite similar to SSRIs. In addition, mine0. The advantage of bupropion is a good
SNRIs can cause hypertension at higher dosages. side effect profile with low risk of sexual side
D. Monoamine oxidase inhibitors: These drugs act by effects, weight gain or sedation. The common
inhibiting the metabolism of monoamines. There side effects are insomnia, tremors, restlessness
are two isoforms of the enzymes (MAO), MAO-A and nausea. A particular worrisome side effect
(involved in metabolism of serotonin, norepi¬ is seizures (usually seen at higher dosages).
nephrine and dopamine) and MAO-B (preferential Bupropion is also used for smoking cessation.
metabolism of dopamine). The nonselective MAO • Tianeptine and amineptine: These antidepres¬
inhibitors which includes tranylcypromine, phen¬ sants work by enhancing0 the reuptake of sero¬
elzine and isocarboxazid inhibits both the isoforms tonin (serotonin reuptake enhancer).
irreversibly. These drugs are rarely used now as • Antipsychotics: If patient has depression with
they can cause hypertensive crisis. psychotic symptoms, a combination of antide¬
Cheese reaction: Cheese, red wine and beer con¬ pressants and antipsychotics is used.
tains tyramine (which is an indirectly acting sym¬ 2. Psychotherapy: It is the treatment using psychological
pathomimetic). Normally, when these items are techniques. The following psychotherapeutic tech¬
consumed, the MAO-A present in the gastrointes¬ niques are effective in depression:
tinal tract degrades the tyramine. However when A. Cognitive behavioral therapy: This therapy aims
MAO inhibitors are used, the tyramine escapes at correcting cognitive distortions (faulty ways of
degradation and gets absorbed resulting in dan¬ thinking) and faulty behaviors. It is the most effec¬
gerous elevation of blood pressure, causing hyper¬ tive0 psychotherapeutic technique in depression.
tensive crisis (also called cheese reaction). Hence B. Interpersonal therapy: In interpersonal therapy, the
these food items are restricted in a patient who is focus is on management of patient's current inter¬
on MAO inhibitors. Phentolamine is the drug of personal problems (e.g. relationship problems).
choice for cheese reaction. C. Other less commonly used therapeutic techniques
E. Atypical antidepressants: There are many other include behavior therapy, family therapy and psy-
antidepressants which have novel mechanisms of choanalytically oriented therapy.
actions. These include: 3. Other somatic treatments:
• Trazodone and nefazodone: These drugs are A. Electroconvulsive therapy (ECTs): The indications
classified as SARI (serotonin antagonist and for ECT in depression includes:
reuptake inhibitors). The mechanism of action • Severe depression with suicide risk0 (If the
is weak inhibition of serotonin reuptake and patient is suicidal, ECT is the preferred treat¬
strong antagonism at 5 HT2A and 5 HT2C recep¬ ment modality)0.
tors. Trazodone can cause priapism0 as a side • Severe depression with stupor0.
effect. • Other indications include depression with psy¬
• Mirtazapine : Mirtazapine belongs to a class chotic symptoms, refractoriness to other treat¬
called NSSA (nor adrenergic and specific sero¬ ment modalities.
tonergic antidepressant). The mechanism of B. Transcranial magnetic stimulation°: It is a newer
action is antagonism of central presynaptic a-2 modality which uses magnetic energy to stimulate
(alpha-2) receptors which results in increased nerve cells. It is nonconvulsive, requires no anes¬
firing of norepinephrine and serotonin neu¬ thesia, has a safe side effect profile and is not asso¬
rons. The other important action is antago¬ ciated with cognitive side effects. Its use is yet not
nism of postsynaptic serotonin 5 HT2 and widespread.
5 HT3 receptors. Mirtazapine causes sedation C. Vagal nerve stimulation°: This modality involves
and weight gain but doesn't have problematic stimulation of vagal nerve using an electrode.
sexual side effects. D. Deep brain stimulation°; This modality involves
• Bupropion: Bupropion belongs to a class called implantation of leads into specific brain areas and
NDRI (norepinephrine dopamine reuptake has been used in patients with chronic and intrac¬
inhibitors). The mechanism of actionis inhibition table depression.
Mood Disorders 31 \
E. Sleep deprivation: Sleep deprivation can produce B. Increased self esteem or grandiosity (e.g. patient
significant benefits however these are transient believes himself to be the richest, most powerful, most
and are typically reversed by next night of sleep. goodlooking person on the earth, etc.)
Research is ongoing to produce sustained benefits. C. Decreased need for sleepQ (e.g. patient feels rested
F. Phototherapy: It has been primarily used for after 2 hours of sleep)
seasonal affective disorders (mood disorder D. Over-talkativeness
with seasonal patternQ). In this disorder patients E. Flight of ideas'3
typically develop depressive symptoms during F. DistractibiIityQ (not able to concentrate on task in
winter seasons which are associated with decreased hand)
day time. The phototherapy involves exposure to G. Increase in goal directed activities (overactivity, hyper¬
bright light in range of 1500-10,000 lux or more. sexuality, overfamiliarity) or psychomotor agitation
Usually a combination of pharmacotherapy and psy¬ H. Excessive involvement in activities that have high
chotherapy is used in management of depressed patients, potential for painful consequences (e.g. unrestrained
in cases of suicide risk, ECT is the preferred treatment. buying sprees, sexual indiscretions or foolish business
investments).
BIPOLAR DISORDER _ These symptoms should last for atleast 7 days'3 and
must cause marked impairment in social and occupa¬
Bipolar disorder is characterized by episodes of both tional functioning.
mania and depression. Even if a patient has only manic Psychotic symptoms: Apart from the above mentioned
episodes'3, he would still be diagnosed with bipolar dis¬ symptoms, patient may also develop psychotic symp¬
order, as in all likelihood he would develop a depressive toms (delusions and hallucinations). These may be
episode in future. Bipolar disorder is equally prevalent mood congruent (e.g delusion of grandiosity) or mood
among men and women. Manic episodes are more com¬ incongruent (e.g. delusion of persecution). In the
mon in men, and depressive episodes are more common presence of psychotic symptoms, the diagnosis made is
in women. The average age of onset of bipolar disorder is manic episode with psychotic symptoms.
30 years. It is more commonly seen in divorced and sin¬
Hypomania: The symptoms of hypomania are similar
gle persons. Bipolar disorder has multiple subtypes which to mania however they are not severe enough to cause
have been illustrated in the following Table 1. marked impairment in social and occupational function¬
ing. Also, the duration criterion for hypomania is 4 days.
Symptoms Mixed episodes: Mixed episodes have both manic and
The symptoms of manic episode are as follows: depressive symptoms lasting for at least 7 days.
A. Elevated mood (undue happiness) or irritable mood
Etiology
Table 1: Types of bipolar disorders. • Neurotransmitters: Increased levels of dopamine has
been implicated in pathophysiology of manic episode.
Bipolar 1/2 Schizobipolar disorder (schizoaffective
disorder) The changes in depression have been already discussed.
Bipolar I Mania with depression (or mania alone) • Genetic factors: The chromosomes 18qQ and 22q have
the strongest evidence of linkage to bipolar disorder.
Bipolar I 1/2 Depression with protracted hypomania
Chromosome 21q has also been linked.
Bipolar II Depression with discrete hypomanic episodes
Bipolar II 1/2 Depression superimposed on cyclothymia Treatment
Bipolar III Depression plus induced hypomania (e.g. The treatment in bipolar disorder depends on the phase.
hypomania occurring solely in association with
Patient requires treatment during acute illness (acute
antidepressants or other somatic treatment
manic or mixed or depressive episodes) and also need
Bipolar III 1/2 Bipolar disorder associated with substance use
prophylaxis to prevent further episodes (maintenance
Bipolar IV Depression superimposed on hyperthymic treatment). The following classes of drugs are usually
temperament used in bipolar disorder:
ÿ 32 Review of Psychiatry

A. Mood stabilizers: Commonly used mood stabiliz¬ C. Maintenance


ing drugs include lithium, valproate, carbamazepine • Usually given after two or more acute episodes in
oxcarbazepine and, lamotrigine. Many atypical anti¬ bipolar Iillness or after a single manic episode if
psychotics also have mood stabilizing properties: it was associated with significant risk.
• Lithium is considered the prototypical mood sta¬ • Lithium and valproate have the best evidence.
bilizer0. However it takes around 1-2weeks to start • Treatment should be continued for at least two
acting. It is usually supplemented by other mood years.
stabilizers, antipsychotics or benzodiazepines in
early phase of treatment. Lithium
• Valproate: Valproate has surpassed lithium in use
for acute mania due to better tolerability. Lithium is used for treatment of acute episodes (both
mania and depression) as well as prophylaxis in bipolar
• Lamotrigine : It is mostly used in treatment of acute
depressive episode of bipolar disorder (bipolar disorder. Lithium is a monovalent cation and gets rapidly
depression0). and completely absorbed after oral administration. The
B. Antipsychotics: Usually atypical antipsychotics are plasma half-life is initially 1.3 days and gets increased
used due to better tolerance and side effect profile. to 2.4 days after continued administration for more than
C. Benzodiazepines: High potency benzodiazepines such one year, lithium is not metabolized in the body and gets
as lorazepam and clonazepam are frequently used in excreted unchanged through the kidney.
acute mania due to their calming effect.
D. Antidepressants: Antidepressants are never used Indications
alone in bipolar disorder. When used alone in bipolar A. Acute manic episode: Lithium is an effective treatment
depression they can cause switch (patient may go into for acute mania however since its onset of action is
mania), hence they are always used along with mood delayed (1-3 weeks), an antipsychotic, benzodiaz¬
stabilizers. epine or valproate is usually added for initial period.
Lithium is also effective for prophylaxis against future
Treatment Guidelines manic episodes.
A. Acute manic or mixed episode: B. Bipolar depression: Lithium is effective for treatment
of bipolar depression and prophylaxis of same, how¬
• For severe mania or mixed episode, initiate lithium
in combination with an antipsychotic or valproate ever the antimanic efficacy of lithium is more than its
in combination with an antipsychotic. antidepressive efficacy.
C. Maintenance treatment: Maintenance treatment with
• For less ill patients, monotherapy with lithium, val¬
proate or an atypical antipsychotic such as olan¬ lithium decreases the frequency, severity and dura¬
zapine may be sufficient. tion of manic and depressive episodes in patients with
bipolar disorders.
• Short-term treatment with benzodiazepines is often
used. D. Lithium is also use in patients with schizoaffective
disorders as well as an adjuvant to antidepressants in
• For mixed episodes, valproate is preferred over lithium.
• If patient has psychotic symptoms, antipsychotics major depressive disorder.
must be added to the treatment regimen. E. Other indications in which lithium has been used
B. Acute depression (bipolar depression): but is not the first line treatment include obsessive
• Initiate lithium or lamotrigine. compulsive disorder, aggression, headache (cluster,
• In severely ill patients, initiate treatment with both migraine0), gout, epilepsy, movement disorders, neu¬
lithium and an antidepressant. tropenia0, ulcerative colitis.
• Quetiapine alone and combination of olanzapine Lithium has a narrow therapeutic index and therapeu¬
and fluoxetine are other treatment options. tic drug monitoring is required. The effective serum con¬
• Antidepressant mono therapy should never be given. centration for treat ment of acute mania is 1.0-1.5 mEq/
• Electroconvulsive therapy for patients with high dL°. Hie serum concentration required for maintenance
suicide risk. treatment is 0.6-1.2 mEq/dL0.
Mood Disorders 33

Side Effects B. Dysthymia: It is the


presence of mild DSM-5 Update: In DSM-5 a new
A. Neurological side effects: Lithium can cause postural depressive symp¬
diagnosis of "premenstrual dysphoric
tremors (usually treated with beta blockers like pro¬ disorder" has been added. This dis¬
toms (not enough order is characterized by symptoms
pranolol'3), lack of spontaneity and memory distur¬ such as mood swings, irritability, leth¬
to diagnose a major
bances, rarely it can cause raised intracranial tension argy, physical symptoms like breast
and peripheral neuropathy. depressive episode) tenderness, which start before the
onset of menses and improve after
B. Endocrine: Hypothyroidism, rarely hyperthyroidism, for a period of more onset of menses.
hyperpara- thyroidism. than two years.
C. Renal: Most common is polyuria at time progress¬ Chronic depression: If the depression continues for
ing to diabetes insipidus which is treated with use of more than 2 years, it is known as chronic depression.
thiazide diuretics or potassium sparing diuretics (like C. Cyclothymia: It is a milder form of bipolar disorder,
amiloride, spironolactone or triamterene) . Rarely in which manic symptoms and depressive symptoms
nephrotic syndrome, renal tubular acidosis or inter¬ occur, but they are never severe enough to make a
stitial fibrosis can be seen. diagnosis of mania/hypomania or depression. The
D. Others include acne, psoriasis, nausea, vomiting, diar¬ symptoms should last for adeast 2 years.
rhoea, weight gain, benign T wave changes. Rapid cycling: If a patient of bipolar disorder has four
Lithium toxicity: The risk factors of lithium toxicity include or more than four episodes of mania/hypomania/
renal impairment, dehydrationQ and low sodium dietQ. depression in one calendar year.
Usually the sign of toxicity starts to appear at levels above
1.5 mEq/dL. The early signs include GI symptoms like
abdominal pain, vomiting and neurological symptoms like
SUICIDE _
coarse tremors, ataxiaQ and dysarthria. The later signs and The psychiatric illnesses associated with highest risk of
symptoms include impairment of consciousness, muscular suicide are depressive disorder, schizophrenia, alco¬
fasciculations, increased deep tendon reflexes'3 and con¬ hol dependence and other substance dependence and
vulsions. There might be circulatory failure and death. The personality disorders (especially borderline personality
management involves stopping lithium, correcting dehy¬ disorder and antisocial personality disorder. Low CSF
dration, use of polyethy lene glycol (and not activated
levels of 5-hydroxyindoleacetic acid (5 HIAA)Q, which
charcoal) to remove unabsorbed lithium from GI tract.
is a metabolite of serotonin, are associated with higher
In severe cases, hemodialysis may be required.
suicide risk.
OTHER MOOD DISORDERS_ The following are the risk factors for suicide:
1. Male sexQ
A. Recurrent depressive @-
2. Age 45 years
DSM-5 Update: In DSM-5, the
>
disorder. If there
diagnosis of dysthymia has been 3. Divorced, widowed
are more than one removed. A new diagnostic category
4. Unemployed
depressive episodes, of "persistent depressive disorder"
diagnosis of recur¬ has been made, which includes both 5. Chronic illness
chronic depression and previous dys¬
rent depressive dis¬ 6. Family history of suicide
thymic disorder.
order is made. 7. Poor social support.
34 Review of Psychiatry

QUESTIONS AND ANSWERS

QUESTIONS _ Symptoms and Diagnosis


Depression 9. Persistent feeling of guilt is seen in:
(DNB NEET 2014-15)
Epidemiology and Etiology A. Obsessive compulsive disorder
1. Most commonly depression is seen in: B. Mania
(AI1996, 1998) C. Depression
A. Middle aged men B. Middle aged female D. Schizophrenia
C. Young girl D. Children
10. DSMIV, duration criterionfor diagnosis of depres¬
2. Neurotransmitters involved in depression are: sion is: (DNB NEET2014-15)
(All 995) A. lweek B. 2 weeks
A. GABA and dopamine C. 3 weeks D. 4 weeks
B. Serotonin and norepinephrine
C. Serotonin and dopamine .
11 Which of the following symptoms must be present
D. Norepinephrine and GABA for the diagnosis of major depressive disorder:
(MH2010, 2007)
3. Which of the following is nota part of cognitive triad A. Loss of interest or pleasure
of beck? (AIIMS Nov 2015) B. Recurrent suicidal tendencies
A. Hopelessness B. Worthlessness C. Insomnia
C. Helplessness D. Guilt
D. Indecisiveness
4. All of the following about "Aaron becks cognitive
12. Disruptionordisorganizationofbiologicalrhythmis
theory of depression" is true, except:
observed in:
(DNB Dec 2010)
A. Schizophrenia B. Anxiety
A. Negative thought of past
C. Depression D. Mania
B. Negative thought of future
C. Negative thought of environment 13. "Nihilistic delusions" are seen in: (PGI2000)
D. Negative about self A. Endogenous depression
5. True about major depressive disorder:
B. Double depression
(PGINov 2011) C. Depression in involutional stage
A. Abnormally diminished activity in prefrontal D. Cyclothymia
cortex E. Dysthymia
B. Lesion of corticospinal tract 14. True about psychotic feature in depression:
C. Monoaminergic system disturbances
(PGIDec 2004)
D. Genetic predisposition is present
A. Found in severe depression
6. Depression is seen in: B. Found in moderate depression
A. Hyperthyroidism B. Hypogylcemia C. Mood incongruent psychotic feature
C. Adrenal disorder D. Pheochromocytoma D. Cyclothymia
7. Depression is a feature of which of the following E. Dysthymia
condition: 15. Intense nihilism, somatization and agitation in
A. Hypopituitarism B. Hyperthyroidism old age are the hallmark symptoms of:
C. Hypothyroidism D. Hypoglycemia A. Involutionalmelancholia
8. Depression is not caused by: B. Atypical depression "
A. Clonazepam B. Levodopa C. Somatized depression
C. Metronidazole D. Corticosteroid D. Depressive stupor
Mood Disorders 35

16. True about major depressive disorder -.(PGI2003) A. Leave him as normal adolescent problem
A. Commonly seen in female B. Rule out depression
B. Recovery is complete after treatment C. Rule out migraine
C. Associated with hypothyroidism D. Rule out an oppositional defiant disorder
D. Family history of major depression
22. A 40-years-old female patient presents with his¬
17. Dysthymia is: (DNB NEET 2014-15) tory of depressed mood,loss of appetite, insomnia
A. Chronic mild depression and lack of interest in surroundings for past one
B. Chronic severe depression year. These symptoms followed soon after a busi¬
C. Bipolar disorder ness loss one year back. Which of the following
D. Personality disorder statements is true regarding the management of
18. Most common type ofpostpuerperal psychosis is: this patient:
(PGI1999) A. No treatment is necessary as it is due to business
A. Depression B. Anxiety loss
C. Mania D. Suicide B. SSRI is the most efficacious of the available drugs
C. Antidepressant treatment is based on the side
Clinical Vignettes effect profile of the drugs
19. A 41-year-old woman presented with a history of D. Combination therapy of two antidepressant
aches and pains all over the body and generalized should be given
weakness for four years. She cannot sleep because 23. A patient presents with depressed mood, loss of
of the illness and has lost her appetite as well. She sleep, loss of hope, feeling of worthlessness and
has lack of interest in work and doesn't like to diminished concentration for last 1 month.
meet friends and relatives. She denies feelings of Which of the following is the drug of choice inthis
sadness. Her most likely diagnosis is: patient? (DNB NEET 2014-15)
(AIIMS Nov 2002) A. SSRIs
A. Somatoform pain disorder B. Atypical antidepressants
B. Major depression C. Lithium
C. Somatization disorder D. Tricyclic antidepressants
D. Dissociative disorder
24. A woman hasmild depressive symptoms after few
20. A 60-year-old male isbrought byhiswife. He thinks days of delivery which disappeared after 2 week
that he hadcommitted sins throughout his life. He in postpartum period. The most likely cause is:
is very much depressed andhas considered com¬ (PGIMay 2015)
mitting suicide but has not taken any such steps. A. Postpartum blue B. Mania
He is also taking sessions with a spiritual guru. He C. Postpartum depression D. Mild depression
does not get convincedwhenhiswife tells himthat E. Postpartum psychosis
he has led a pious life. How will you treat him: 25. A patient presents to the emergency department
A. Antipsychotic plus antidepressant with self harm and indicates suicidal intent.Which
B. Antidepressant with cognitive behavioral therapy of the following conditions does not warrant an
C. Guidance and recounselling with guru plus immediate specialist assessment: (AI2010)
antidepressant A. Formal thought disorder
D. Antidepressant alone B. Acute alcohol intoxication
.
21 An 18-year-oldstudent complaints of lack of inter¬ C. Chronic severe physical illness
est in studies for last 6 months. He has frequent D. Social isolation
quarrels with his parents and has frequent head¬ 26. A 50-year-old male presents with a three year
aches. The most appropriate clinical approach history of irritability, low mood, lack of interest
would be: (AI2005) in surroundings and general dissatisfaction with
36 Review of Psychiatry

everything. There is no significant disruption in 33. Repetitive transcranial magnetic stimulation


sleep and appetite. He is likely to be suffering (rTMS) is approved by USFDA for the treatment
from: of: (AI 2012)
A. Major depression A. Resistant schizophrenia
B. No psychiatric disorder B. Obsessive compulsive disorder
C. Dysthymia C. Acute psychosis
D. Chronic fatigue syndrome D. Depression
27. A patient is depressed for past 3 years, does not 34. A young female on antidepressants presents to the
go out of his house much and is cut off from the emergency with altered sensorium and hypoten¬
society. But with normalsleep and normalweight. sion. ECG reveals wide QRS complexes and right
Most probable diagnosis is? axis deviation. What is the next best step?
(DNB December 2011) (AIIMS Nov 2015)
A. Major depression A. Sodium bicarbonate B. Hemodialysis
B. Dysthymia C. Fomepizole D. Flumazenil
C. Chronic fatigue syndrome
35. Rathi, 26-years-old female has been diagnosed
D. No psychiatric illness
to be suffering from depression. Now for the past
2 days she has suicidal tendency, thought and
Treatment
ideas. The best treatment is: (AIIMS 2001)
28. Which is not a serotonin norepinephrine reuptake A. Amitriptyline
inhibitor: (PGINov 2009) B. Selegiline
A. Paroxetine B. Mirtazapine C. Haloperidol + chlorpromazine
C. Escitalopram D. Venlafaxine D. ECT
29. SSRIs shouldbe carefully usedinthe young for the 36. A patient comes instuporous condition. Patient's
management of depression due to increase in: parents give history of patient being continu¬
(DNB NEET 2014-15) ally sad and suicidal attempts and not eating and
A. Nihilismideation B. Guilt ideation sleeping for most of the time. The treatment is:
C. Suicidal ideation D. Envious ideation (AIIMS 2000)
A. ECT B. Antidepressant
30. Features of serotonin syndrome associated with
C. Antipsychotic D. Sedative
SSRIs, and MOAIs are all except:
(DNB NEET 2014-15) 37. A patient on antidepressant therapy developed
A. Tremors B. Agitation sudden hypertension on consuming cheese. The
C. Cardiovascular collapse D. Hypothermia antidepressant is possibly: (PGI 1999)
A. Amitriptyline B. Tranylcypromine
31. The clinical effects of the antidepressant drugs is
C. Fluoxetine D. Sertraline
mainly based on: (DNB NEET 2014-15)
A. Change in neurotransmitter receptor sensitivity 38. Tricyclic antidepressants have all of the following
B. Decreased levels of neurotransmitters actions except: (PGI 1999)
C. Change in efficacy of neurotransmitters A. Anticholinergic action
D. None of the above B. Anti MAO (monoamine acid oxidase) action
C. Blocks 5 HT (serotonin) or NE (norepinephrine)
32. Mechanism of action of bupropion is:
reuptake
(DNB NEET 2014-15)
D. Causes sedation
A. Increased levels of GABA
B. Increasedlevels of norepinephrine inthe synap¬ 39. A patient on treatment for psychiatric disorder
tic cleft takes overdose of a drug, develops bradycardia,
C. Increasedlevels of dopamine inthe synaptic cleft hypotension, decreased sweating and salivation.
D. Both B and C The likely drug is: (AIIMS 1999)
Mood Disorders 37

A. Amitriptyline B. Lithium 48. Phototherapy isused to treat which of the follow¬


C. Selegiline D. Amphetamine ing psychiatric condition: (DNB NEET2014-15)
A. Depression
40. Tricyclic antidepressant are contraindicated in: B. Mental retardation
(DNB 1997, AI 1991)
C. Schizophrenia
A. Glucoma B. Brain tumor
D. Obsessive compulsive disorder
C. Bronchial asthma D. Hypertension

.
41 Following drugs have abuse liability except: Suicide
(DNB 2003)
49. Increased suicidal tendency is associated with:
A. Buprenorphine (DNB NEET 2014-15)
B. Alprazolam
A. Increased noradrenaline
C. Fluoxetine
B. Decreased serotonin
D. Dextropropoxyphene
C. Decreased dopamine
42. Tianeptine acts by: (AIIMS 1998) D. Increased GABA
A. MAO inhibitor
50. Risk factor for suicide indepression are all except:
B. Serotonin uptake inhibitor (DNB June 2011)
C. Serotonin uptake enhancer
A. Female
D. 5-HT agonist
B. Male >45 years
43. What is/are the side effects of SSRI: C. Child with conduct disorder
A. Insomnia B. Sedation D. Family history
C. Nausea
E. Weight gain
D. Seizure precipitation
.
5 1 Suicidal tendencies are most common in:
(PGI2000)
44. Not true regarding serotonin syndrome is: A. Involutional depression
A. It is predictable and not idiosyncratic B. Reactive depression
B. SSRIs and MAOIs cause it C. Psychotic depression
C. IV dantrolene is the treatment of choice D. Childhood depression
D. Hypertension, hyperthermia and hyperreflexia 52. Emile Durkheim is linked with work on which of
are the signs the following conditions in psychiatry?
45. Stimulation of which of the following nerve cause (DNB NEET 2014-15)
elevation of mood: (AIIMS Nov 2009) A. Suicide
A. Olfactory nerve B. Optic nerve B. Obsessive compulsive disorder
C. Trigeminal nerve D. Vagus nerve C. Anxiety disorder
D. Schizophrenia
46. Following are the somatic therapies used in
depression, except: (DNB NEET 2014-15) 53. Incidence of suicide in India is: (PGIJune 2005)
A. Electroconvulsive therapy A. 8-10/100 population
B. Deep brain stimulation B. 8-10/1000 population
C. Transcranial magnetic stimulation C. 8-10/10000 population
D. Ultrasound brain stem stimulation D. 8-10/100000 population
47. The evidence-based psychological therapy of 54. Suicidal tendencies are seen in: (PGI2002)
choice for depression is: (AIIMS May 2014) A. Depression
A. Group discussion therapy B. Post-traumatic stress disorder
B. Counselling C. Schizophrenia
C. Cognitive behavioral therapy D. Substance abuse
D. Psychological psychotherapy E. Anxiety
38 Review of Psychiatry

62. Mania is characterized by: (PGI1999)


Bipolar Disorder
A. Paranoid delusions B. Loss of orientation
Classification C. High self esteem D. Loss of insight
55. Chromosome associated with bipolar disorder: 63. Which of the following is not a symptom of mania:
(PGIDec 2005) (DNB NEET 2014-15)
A. Chromosomel6 B. Chromosome 13 B. Elated mood
A. Distractibility
C. Chromosome 18 D. Chromosome 11
C. Delusion of grandeur D. Increased sleep
E. Chromosome 23
56. Bipolar IIdisorder includes: Clinical Vignettes
(DNB NEET 2014-15, AIIMS 2011)
A. Cyclothymic disorder 64. A 20-year-old man has presented with increased
B. Dysthymia alcohol consumption and sexual indulgence,
C. Single manic episode irritability, lack of sleep, and not feeling fatigued
D. Major depression and hypomania even on prolonged periods of activity. All these
changes have been present for 3 weeks. The most
57. All of the following are included in diagnosis of likely diagnosis is: (AI2003)
bipolar disorder except: (AI, 2007) A. Alcohol dependence
A. Mania alone B. Schizophrenia
B. Depression alone C. Mania
C. Mania and depression D. Impulsive control disorder
D. Mania and anxiety
65. A 67-year-old lady is brought in by her 6 children
58. Which of the following is/are included in bipolar saying that she has gone senile. Six months after
disorders: (PGINov 2010) her husband's death she has become more reli¬
A. Hypomania B. Cyclothymia gious, spiritual and gives lots of money in dona¬
C. Paranoid disorder D. Hyperthymia tion. She is occupied in too many activities and
E. Kleptomania sleeps less. She now believes that she has a goal
to change the society. She does not like being
59. The period of normalcy is seen between two psy¬
chosis. The diagnosis is: (AI 1999) brought to the hospital and is argumentative on
A. Schizophrenia being questioned on her doings. The diagnosis is:
B. Manic depressive psyhosis (AI2002)
A. Depression
C. Alcoholism
D. Depression
B. Schizophrenia
C. Mania
D. Impulse control disorder
Symptoms and Diagnosis
66. A 42-year-old male with a past history of a manic
60. According to the ICD-10revision, for establishing
episode presents with an illness of 1month dura¬
a diagnosis of mania, the symptoms should persist
tion characterized by depressed mood, anhedonia
for at least: (AIIMS May 2014, DNB 2010) and profound psychomotor retardation. The most
A. lweek B. 2 weeks
appropriate management strategy is prescribing
C. 3 weeks D. 4 weeks a combination of: (AIIMS 2004)
61. The clinical features of mania include: A. Antipsychotics and antidepressants
(PGI2006, 2002) B. Antidepressants and mood stabilizers
A. Anhedonia C. Antipsychotics and mood stabilizers
D. Antidepressants and benzodiazepines
B. Elated mood
C. Avolition
D. Delusion of grandiosity Treatment
E. Distractibility 67. Drug of choice inacute mania is: (DNB NEET2015)
Mood Disorders 39 1
A. Lithium B. Chlorpromazine C. Treatment of first depressive episode
C. Valproic acid D. Risperidone D. Prevention of recurrence in bipolar mood dis¬
order
68. Which of the following drugs is/are used in treat¬
ment of acute mania? (DNB NEET 2014-15) 76. A male patient with bipolar disorder is controlled
A. Lithium B. Oxcarbazepine on medications. Symptoms of mania start to
C. Risperidone D. All the above appear whenever he himself tapers down the
drugs. What type of treatment can improve com¬
69. Treatment of bipolar disorder includes: (PGI2011)
pliance in this patient? (AIIMS Nov 2015)
A. Antidepressant drugs
A. Psychoeducation
B. Aversion therapy
B. CBT
C. ECT
D. Lithium carbonate C. Supportive psychotherapy
D. Insight oriented psychotherapy
70. Drugof choice for rapid cycling manic depressive
psychosis (bipolar disorder) is:(DNB2004, A11999) 77. All of the following are mood stabilizing drug in
bipolar disorder except: (PGIMAY 2013)
A. Lithium B. Carbamazepine
C. Sodium valproate D. Haloperidol A. Lithium B. Valproate
C. Carbamazepine D. Clonazepam
71. Prophylactic maintenance serum levels of lithium
E. Lamotrigine
is: (AI 1994, DNB 1997)
A. 0.2-0.8 mEq/L B. 0.7-1.2 mEq/L 78. A 30-years-old pregnant woman comes to your
C. 1.2-2.0 mEq/L D. 2.0-2.5 mEq/L clinic with decreased sleep, increased appetite
and hyperactivity for last 2 weeks. A diagnosis of
72. True about Lithium toxicity: (PGI2012)
mania is made. Further probing reveals four epi¬
A. Causes ebstein anomaly
sodes of major depression in the past two years.
B. Decreases neutrophil count
What drug will you prescribe to this patient?
C. Decreases eosinophil count
(AIIMS Nov 2015)
D. Optimum concentration is 0.2-0.6 mEq/L
A. Haloperidol B. Lithium
E. Decreases sodium excretion
C. Promethazine D. Clonazepam
73. True about Lithium treatment in mania:
A. Commonest side effect is tremor
B. Toxic level is <1.5 mg/ dL serum level
ANSWERS _
C. Amyloid is DOC for Liinduceddiabetes insipidus 1. B.
D. Lithium is 90% protein bound 2. B.
E. Tremor is treated with propranolol 3. D.
4. A.
74. A patient is brought to the casualty in the state of 5. A, C and D.
altered sensorium. He was on lithium treatment 6. A, C.
for affective disorder and has suffered through Depression is the most common psychiatric ill¬
an attack of epileptic fits. On examination he has
ness associated with both hypothyroidism and
worsening tremors, increased DTR's and inconti¬
hyperthyroidism. Also, in adrenal disorders
nence of urine. He has also undergone an episode
like Cushing's syndrome and Addison's disease,
of severe gastroenteritis 2 days ago. The serum
depression is commonly associated.
lithiumwas found to be 1.95 mEq/L. The probable
7. A, B, C. Inhypopituitarism, depression is commonly
cause for his present state is: (AIIMS 2001)
seen.
A. Lithium toxicity B. Dehydration
8. C. Few common medications which cause
C. Manic episode D. Depressive stupor
depression include antihypertensives (reserpine,
75. Best use of lithium is in: (DNB NEET 2014-15) methyldopa, beta blockers), steroids (cortico¬
A. Treatment of schizophrenia steroids, oral contraceptive pills), barbiturates
B. Treatment of recurrent depression and benzodiazepines (like clonazepam).
| 40 Review of Psychiatry

9. C. 22. C. The diagnosis in this case is depression. The


10. B. depression can be precipitated by various
11. A. According to DSM-IV and DSM-5, to diagnose stressors, and irrespective of what precipitated
depression at least one of the following two it, it should always be treated. Further, all the
symptoms shouldbe present (1) depressed mood available antidepressants have similar efficacy
(2) loss of interest or pleasure. and the choice of antidepressants is usually
12. C. There are characteristic disturbances of sleep dictated by the side effect profile of the drug.
(early morning insomnia and reduced latency SSRIs are usually used as the first antidepressants
of REM sleep) in depression. because of their favorable side effect profile.
13. A,B,C. 23. A.
Psychotic symptoms are seen more commonly 24. A. The postpartum blue includes transient symp¬
in both endogenous and melancholic depres¬ toms such as irritability, tearfulness, sadness
sion (depression in involutional stage). It can of mood, decreased sleep and appetite. These
also be seen in double depression, though less symptoms usually get resolved by 10th day
commonly (double depression is depression postpartum.
superimposed over dysthymia). 25. B. The suicidal intent in a person with formal
14. A,C.
thought disorder (most likely a patient with
The presence of psychotic symptoms itself makes
schizophrenia), chronic severe physical illness
a depression severe, hence psychotic symptoms
and social isolation, should betaken very serious¬
are seen only insevere depression. The psychotic
ly and immediate measures taken must include
symptoms can be either mood congruent or
assessment by a specialist. If patient has
mood incongruent.
expressed suicidal intent in an inebriated state,
15. A.
it must still be ensured that he doesn't harm
16. A, C and D.
himself however a specialist assessment can be
The recovery is often incomplete inpatients with
deferred till he is sober.
depression.
26. C. Long standing and less severe depressive symp¬
17. A.
toms along with normal sleep and appetite indi¬
18. A. The most common type of postpuerperal psycho¬
cates dysthymia.
sis is depression.
27. A. In this case, the better answer would be
19. B. Inthis case, patient has significant somatic symp¬
toms such as aches and pain and generalized
depression. Though the question, mentions
weakness. Ina large number of patients, depres¬ that sleep and appetite is normal, however
sion presents mostly with somatic complaints the fact that he is cut off from society indi¬
and patient may deny psychological symptoms cates severe symptomatology and hence

such as sadness of mood. Further, in this patient depression would be a better diagnosis than
there are sleep and appetite abnormalities along dysthymia here. Usually in patients with dys¬
with loss of interest which clinches the diagnosis thymia, the overall functioning is minimally
of depression. affected and that's an important clue for the
20. A. This patient has depression with psychotic symp¬ diagnosis.
toms. The patient belief that he committed sins 28. A,B,C.
inhis life, andthe fact that despite his wife assur¬ 29. C. The use of SSRIs can increase suicidal ideations.
ances he continues to holdthe belief is suggestive This side effect is more common in children and
of delusion. Hence, this patient should betreated adolescents and hencethese medications should
with antidepressants and antipsychotics. be used cautiously in that age group.
21. B. In children and adolescents, depression 30. D. Inserotonin syndrome, hyperthermia is a feature.
frequently presents with irritability, lack of 31. A. The recent research has shown that its not the
interest and changes in behavior such as increase in neurotransmitters levels in synapse
withdrawn behavior or quarrelsome behavior. which causes antidepressant effect. Rather, sec¬
Its important to rule out depression first. ondary to increased neurotransmitter levels, the
Mood Disorders 41

receptor sensitivity changes over a course of time 49. B. Decreased levels of 5 HIAA (which is a metabo¬
and that is responsible for antidepressant effect. lite of serotonin) are related to increased risk of
32. D. suicide.
33. D. 50. A. Maleshave higher suicideriskthanfemales. Please
34. A. This patient was most likely on tricyclic antide¬ remember that females make more suicide
pressants and it appears to be a case of tricyclic attempts than males, however males complete
antidepressant over-dosage as the patient is suicide more commonly than females. This dif¬
experiencing arrhythmias, hypotension and has ference is mostly due to method used, males
also developed altered sensorium. The mainstay tend to use more lethal methods such as gun
of treatment in TCA induced cardiotoxicity is and hence are more likely to complete suicide.
intravenous sodium bicarbonate. It is used if the 51. A, C.
QRS interval is prolonged (usually more than 100 Endogenous depression, depression with psy¬
milliseconds) and can reverse the toxic effects of chotic symptoms (psychotic depression) and
TCAs. Because of large volume of distribution involutional depression (depression with mel¬
and high protein binding of TCAs, hemodialysis ancholic features) are associated with higher
is not effective. Further flumazenil and fomepi- suicide risk.
zole have no role. 52. A. Emile Durkheim studied extensively the social
35. D. In depression with suicide risk, ECT is the treat¬ factors associated with suicide.
ment of choice. 53. D. The data for incidence of suicide is released
36. A. This patient most likely has depression with by government every year. According to NCRB
stupor. ECT is again the treatment of choice. (National Crime Record Bureau), in 2014, the
37. B. This is history of cheese reaction on MAO inhibi¬ suicide rate in India was 10.6/lac of population.
tors. 54. A,C,D.
38. B. 55. C.
39. A. The symptoms are suggestive of tricyclic antide¬ 56. D.
pressants overdose (anticholinergic side effects). 57. B. Evena single episode ofmania is sufficient to make
40. A. Due to anticholinergic action, TCAs should be a diagnosis of bipolar disorder.
avoided in glaucoma. 58. A,B.
41. C. SSRIs do not have any abuse liability. Both 59. B. Manic depressive psychosis was the older name
opioids (buprenorphine, dextropropoxyphene) for bipolar disorder. In bipolar disorder, in bet¬
and benzodiazepines (alprazolam) have abuse ween the episodes, patient is usually normal.
liability. 60. A.
42. C. 61. B,D,E.
43. A,B,C,E. 62. C,D.
SSRIs can cause bothsedation as well as insom¬ Insight is absent in mania and usually high self
nia. In long-term they can cause weight gain. esteem is also a clinical feature.
Nausea, diarrhoea, anxiety and sweating are 63. D. The sleep is usually decreased in mania
some common side effects. 64. C. Please remember that in manic stages, the sub¬
44. C. Dantrolene is not the treatment of choice, though stance intake also frequently increases.
it is at times used to control the hyperthermia. 65. C. Kindly don't get confused with the fact that
45. D. Vagal nerve stimulation can beused for treatment the symptoms are following husband's death.
of depression. Even negative life events can precipitate manic
46. D. ECT, deep brain stimulation as well as transcra¬ episode. This patient has increased religio¬
nial magnetic stimulation can be used for treat¬ sity, overspending (giving excessive donation),
ment of depression. increased activity levels, decreased sleep, new
47. C. interests and goals (of changing society) and lack
48. A. Depression associated with a seasonal pattern of insight (doesn't want to come to hospital). All
can be treated with phototherapy. these symptoms are suggestive of mania.
42 Review of Psychiatry

66. B. The patient had a manic episode inpast and cur¬ 75. D.
rently he is in severe depression (as suggested 76. A. Psychoeducation is a form of psychological
by profound psychomotor retardation). The intervention in which patient as well as family
complete diagnosis would be bipolar disorder members are educated about various aspects of
(currently severe depressive episode). Hence, disease and its treatment. It involves discussion
this patient should receive bothmoodstabilizers about the symptoms, the need for medications
and antidepressants. as well as maintenance of a regular life style. Psy¬
67. A. This question doesn't makemuch sense. There is choeducation decreases the chances of relapses
no drug of choice inacute mania. The drug is cho¬ in bipolar disorder.
sen depending on the symptoms. If symptoms 77. D.
are severe and we need immediate improve¬ 78. A. This is an interesting question and slighdy con¬
ment, a combination of mood stabilizers with troversial too. We need to consider various fact.
antipsychotic would be preferred. If symptoms First of all, since this patient had 4 episodes
are less severe, either lithium, valproate or an (mania + depression) in last 5 years, prior to
antipsychotic can be used. Here, the answer is the current manic episode, ideally she should
given as lithium as it is the prototypical drug already be on a prophylaxis. However we have
used in mania and can be considered as a gold not been provided with any information about
standard. It is true that lithium takes 1-3 weeks the prophylaxis. Now, the question is asking
for its onset of action, but that doesn't mean about the management of acute manic episode.
that it can't be used in acute mania. In fact, all According to APA (American Psychiatric Associa¬
the guidelines start the treatment algorithm for tion) practice guidelines, in a patient with severe
acute mania with lithium only. mania a combination of mood stabilizers (like
68. D. lithium or valproate) and antipsychotics should
69. A,C,D. be used, whereas in mild to moderate mania,
Lithium and antidepressant are obviously used in monotherapy with lithium, valproate or an an¬
bipolar disorders. Incases of severe mania which tipsychotic (such as haloperidol) can be used.
is not responding to medications, or in bipolar This is for all patients and not specifically for
depression with high suicide risk, ECTs can also pregnancy. Further, APA guidelines mention that
be used. "in pregnancy, antipsychotics may represent an
70. C. alternative to lithium for treating the symptoms
71. B. of mania. In addition, there is no evidence of
72. E > A. teratogenicity with exposure to haloperidol,
Dehydration as well as low sodium levels pre¬ perphenazine, thiothixene and trifluopera¬
dispose to lithium toxicity. Ebstein anomaly is a zine" It must also be remembered that lithium
teratogenic effect of lithium and as such is not a takes around 1-3 weeks to start acting whereas
sign of lithium toxicity. Lithium causes neutro¬ antipsychotics have a much faster onset, and in
philia and eosinophilia. a patient who is in acute manic episode for last
73. A, C, E. two weeks, we need faster response. Maudsley
74. A. The gastroenteritis causes dehydration and prescribing guidelines, which is another well
may result in lithium toxicity (the body handles accepted prescribing guideline says "In acute
lithium similarly to sodium. In presence of mania in pregnancy, use an antipsychotic, if
dehydration, sodium absorption is increasedand ineffective, consider ECT" Hence, inview of these
lithium absorption is also increased in kidneys). reference for acute management of mania in
The lithium toxicity may present with tremors, pregnancy, haloperidolwould be a better answer
increased reflexes and seizure. than lithium.
5|I:
lif
Chapter
Neurotic, Stress Related and
I4 Somatoform Disorders

ANXIETY _ In panic disorder, the patients have recurrent panic


attacks which are not restricted to any particular situa¬
Anxiety is a common experience. It is an alerting signal tion or setting. The patient is usually free from anxiety
and helps a person to take measures to deal with a threat. symptoms in between the attack however anticipatory
It must be differentiated from fear. Fear is the response anxiety (fear that next panic attack can occur anytime)
one would have if he sees a snake. The fear is a response is common. The mean age of presentation is around 25
to a known, external and definite threat. Anxiety is the years and females are two to three times more commonly
response one would have before exams. It is the response affected than men. Panic disorder presents with a number
to an unknown, internal and vague threat. of comorbid conditions, most commonly agoraphobia.
The neurotransmitters which have been implicated in
Manifestations of Anxiety: panic disorders include norepinephrine, serotonin and
• Feeling of nervousness GABAQ. Recently cholecystokininQ has also been found
• Sweating, tachycardia, restlessness, tremors, mydriasis as a mediating neurotransmitter in panic disorder.
• Diarrhea, urinary frequency
• Cold clammy skinQ, hyperreflexia. Differential Diagnosis
ANXIETY DISORDERS _ Due to predominance of somatic symptoms, panic dis¬
order must be differentiated from common physical dis¬
Anxiety disorders are a group of related disorders which orders such as myocardial infarctionQ, angina, mitral
include: valve prolapse asthma, pulmonary embolism, pheo-
• Panic disorder chromocytoma, carcinoid syndrome, hypoglycemia,
• Agoraphobia hyperthyroidism.
• Specific phobia
• Social anxiety disorder Treatment
• Generalized anxiety disorder. Usually a combination of pharmacotherapy and psycho¬
therapy is used.
Panic Disorder
A. Pharmacotherapy: The drugs mostly used include
Panic attack is an acute attack of intense anxiety accom¬ benzodiazepines® and SSRIs. Frequently, both ben¬
panied by "feeling of impending doom'! The symptoms zodiazepines and SSRIs are started concurrently, fol¬
during panic attack usually involve sudden onset of pal¬ lowed by slow tapering of benzodiazepines. Other
pitations, chest pain, choking sensations, dizziness and medications which are used include venlafaxine, bus-
feeling of unreality (depersonalization or derealization). pirone and clomipramine.
Along with these physical symptoms there is also a fear B. Psychotherapy: Cognitive behavioral therapy® is quite
of dying, losing control or going mad. effective in management of panic disorder. Other less
44 Review of Psychiatry

commonly used therapies include Relaxation tech¬ 2. Natural environment type (storms, water, height, etc.)
niques and psychodynamic psychotherapy. 3. Blood-injection-injury type (needles, invasive medical
procedures)
Agoraphobia 4. Situational type (cars, elevators, planes)
5. Others.
It is the fear of places from where escape might be diffi¬
cult0. This basic fear can manifest in various forms such as: Treatment
• Fear of being in open spaces0
A. Pharmacotherapy: The pharmacotherapy is at best
• Fear of crowded places0
used as an adjunct to psychotherapy and includes
• Fear of enclosed places0
benzodiazepines, beta blockers and, SSRIs.
• Fear of travelling alone0
B. Psychotherapy: Behavior therapy is the most effective
• Fear of using public transportations. treatment0 for phobias. A variety of behavioral tech¬
Agoraphobia and panic disorder usually coexist.
niques, all of which involve exposure to phobic stimu¬
Agoraphobia is the most disabling phobia and patient
lus, have been used, which are described as follows:
may become home bound.
• Systematic desensitization: In this method, the
patient is exposed to a series of anxiety provoking
Treatment
stimuli, starting with the least anxiety provoking
A. Pharmacotherapy. The pharmacotherapy usually stimulus. After the exposure, relaxation techniques
includes benzodiazepines and SSRIs. Other medica¬ (usually progressive muscle relaxation) are used to
tions which are used include venlafaxine, buspirone induce relaxation. As the patient masters the tech¬
and clomipramine. nique of relaxation in the presence of an anxiety
B. Psychotherapy: Cognitive behavioral therapy is fre¬ provoking stimuli, he moves up to the next stimu¬
quently used. Behavioral therapy (using techniques lus. This technique has best evidence0 in treatment
such as systematic desensitization, exposure and of phobias.
response prevention, flooding0) is also effective. Less • Therapeutic graded exposure or in vivo exposure (or
exposure and response prevention): It is similar to
commonly used are relaxation techniques and psy¬
systematic desensitization except that no relaxa¬
chodynamic psychotherapy.
tion techniques are used. The patient learns to get
habituated to anxiety.
Specific Phobias • Flooding (Implosion): Here, the patient is exposed
A specific phobia is a strong, persistent and irrational fear to phobic stimulus in its most severe form. The

of an object or a situation. The DSM-5 includes distinctive patient experiences intense anxiety which gradu¬
ally decreases.
types of phobias:
1. Animal type (spiders, insects, dogs) • Modeling (Participant modeling»°): Here, therapist
himself makes the contact with phobic stimulus
and demonstrates this to the patient. Patient learns
Table 1: Common phobias. by imitation, primarily by observation. Apart from
Acrophobia Fear of heights behavioral therapy, other less commonly used psy¬
Ailurophobia Fear of cats chotherapeutic techniques include Psychodynamic
Hydrophobia Fear of water psychotherapy (Insight oriented psychotherapy),
hypnosis, supportive therapy and family therapy.
Claustrophobia Fear of closed spaces
Cynophobia Fear of dogs
Social Anxiety Disorder (Social Phobia)
Mysophobia Fear of dirt and germs
It involves the fear of social situations, including situa¬
Pyrophobia Fear of fire
tions that involve contact with strangers. Patients with
Xenophobia Fear of strangers
this disorder are afraid of embarrassing themselves in a
Zoophobia Fear of animals social situation. The treatment is usually similar to speci¬
Thanatophobia Fear of death fic phobias.
Neurotic, Stress Related and Somatoform Disorders 45 f
Generalized Anxiety Disorder Etiology
This disorder is characterized by excessive anxiety which Serotonin dysregulationQ is considered to be involved
is generalized and persistent and is not restricted to any in the etiopathogenesis of OCD. Less evidence exists for
particular situation (also called "freely floating" anxiety) dysregulation of noradrenergic system in OCD.
and excessive worries. The physical symptoms associ¬ The neuroanatomical model of OCD emphasizes
ated with anxiety are also present. The treatment includes the role of cortico-striatal-thalamic-cortical circuitry
pharmacotherapy (SSRIs, benzodiazepines, buspirone (CSTC). This circuit starts with prefrontal cortex and pro¬
and venlafaxine) and psychotherapy (cognitive behavio¬ jects to striatum which further projects to thalamus and
ral, insight oriented psychotherapy and supportive psy¬ then back to prefrontal cortex. Dysfunction in this circuit
chotherapy). is considered to be responsible for the symptoms of OCD.

OBSESSIVE COMPULSIVE AND Symptoms


RELATED DISORDERS _ OCD has four major symptom patterns.
A. Contamination: Most commonly patients present with
Obsessive-Compulsive Disorder (OCD) obsession of contamination followed by washing
The essential feature of this disorder includes recurrent behavior and avoidance of situations which provoke
obsessive thoughts. For example, a patient repeatedly
obsessional thoughts and compulsive acts.
gets thought that his hands are dirty, which causes
Obsessions are defined by the following properties:
A. Recurrent andintrusive thoughts, images or impulses anxiety, he understands that this thought is senseless
which cause marked anxiety or distress and tries to stop this thought (obsessional thought)
B. The person recognizes that the obsessional thoughts, however is forced to repeatedly wash his hands (com¬
images or impulses are a product of their own mindQ pulsive behavior) which decreases this thought for
some time. He further avoids using public toilet as
(and not imposed by others such as is in thought
insertion) these thoughts get increased in a dirty environment
(avoidance).
C. The person recognizes that the thoughts, images or
B. Pathological doubt: Second most common pattern
impulses are irrational and senselessQ and experi¬
is the obsession of doubt which is usually followed
ences the obsessions and compulsions as ego dys-
by compulsion of checking. For example, a patient
tonic (i.e. unwanted and unacceptable) (in contrast
would repeatedly doubt if he had locked the door
a patient with a delusion, believes in the delusion
properly (obsession) and would repeatedly check the
and doesn't find it senseless or irrelevant)
lock (compulsion).
D. The person attempts to suppress or resist such
C. Intrusive thoughts: Here, patient gets intrusive obses¬
thoughts, images or impulses or tries to neutralize
sional thoughts without an observable compulsion,
them, with some other thoughts or actions.
though mental compulsions are commonly present.
Compulsions are defined by following properties-. The thoughts are usually with sexual and aggres¬
A. Repetitive behaviors (such as hand washing, check¬ sive contentQ. For example, a patient repeatedly
ing) or mental acts (such as counting, praying) that gets the thought about having sex with god, this
the person performs in response to an obsession. thought causes intense anxiety and patient under¬
B. The repetitive behaviors and mental acts are done to stand that this thought is senseless and tries to stop
reduce the distress and anxiety caused by obsessions. the thought but is not able to do so (obsessional
The symptoms of obsessions and compulsions should thought), to decrease the anxiety patient starts to
be present for at least two weeks for the diagnosis of OCD chant prayers in his mind which decreases the anxi¬
The lifetime prevalence of OCD is around 2-3%. ety temporarily (mental compulsions).
Depression is the most common comorbidity in OCD and D. Symmetry: The patient has a need for symmetry or
both must be treated together. precision. This can result in compulsion of slowness.
Hi 46 Review of Psychiatry

For example, a patient would take hours while arrang¬


ing pens on the table. He would ensure that all the
pens are aligned exactly parallel to each other and are
DISORDERS __
TRAUMA AND STRESSOR RELATED

at exact same distance to each other. Post-traumatic Stress Disorder (PTSD)


and Acute Stress Disorder (ASDJ
Course and Prognosis These disorders follow significant traumatic events in
Around 50% of patients with OCD have a sudden onset of which there is a serious injury or threat of serious injury
symptoms. The course is usually chronic. Around 20-30% to self or others and a feeling of helplessness and horror
of patients have significant improvement in their symp¬ during the event. The traumatic events causing PTSD and
toms, around 40-50% have moderate improvement and ASD are sufficiently overwhelming to affect anyone (such
remaining 20-40% have no improvement or further dete¬ as war, earthquake, floods, rape, serious accidents). The
rioration. clinical symptoms are usually seen in the following three
domains:
• Intrusion symptoms: These are characterized by flash¬
Treatment backs*2 (individual may feel as if trauma is reoccur-
A combination12 of pharmacotherapy and psychotherapy ring) and nightmares (dreams about the trauma).
is the preferred approach. • Avoidance: The patient avoids all those stimuli which
A. Pharmacotherapy. The standard approach is to start can remind him of the trauma.
treatment with an SSRIQ. Clomipramine is also consi¬ • Arousal symptoms: These include hypervigilance,
dered the first line treatment however due to its adverse exaggerated startle response, insomnia, poor concen¬
tration.
side effect profile, it is rarely used as a first drug. If
In addition, symptoms such as emotional numbing*2,
treatment with SSRIs or clomipramine is unsuccess¬
emotional detachment*2 and anhedonia*2 can also be
ful, augmentation with antipsychotics (like haloperi-
present. The onset of symptoms may be delayed, if symp¬
dolQ, quetiapine, risperidone and olanzapine) is used.
toms appear 6 months*2 after the trauma, it is diagnosed
Other drugs which have been used include venlafax-
as PTSD with delayed onset*2.
ine, lithium, valproate and carbamazepineQ.
For a diagnosis of post-traumatic stress disorder, the
B. Psychotherapy. Cognitive behavioral therapy rely¬
above mentioned symptoms should be present for more
ing primarily on behavioral technique of exposure than one month, if the duration of symptoms is less than
and response prevention*2 (ERP) has the best evi¬ one month, a diagnosis of acute stress disorder is made.
dence amongst all the psychotherapeutic techniques. The area of brain involved in the pathogenesis of
Exposure and response prevention involves exposure PTSD are hippocampus and amygdala*2.
of patient to a stimulus which is known to produce
obsessional thoughts (exposure) followed by asking Treatment
the patient to not indulge in the compulsive behavior
(response prevention). Selective serotonin reuptake inhibitors (SSRIs)*2 are the
Other types of behavioral therapy such as desen- first line pharmacological treatment in PTSD. Psycho¬
sitizadon, thought stopping, flooding, and aversive
therapeutic interventions include cognitive behavioral
conditioning have also been used. therapy*2 (treatment of choice), psychodynamic psycho¬
Psychodynamic psychotherapy, family therapy can
therapy and eye movement desensitization and repro¬
also be used. cessing (EMDR).
C. Other treatment modalities: In extreme cases that are
treatment resistant electroconvulsive therapy and psy¬
Adjustment Disorders
chosurgery can be considered. The psychosurgical These disorders are characterized by emotional responses
techniques usually include cingulotomy and capsulo- to stressful events like financial problems, medical ill¬
tomy (also known as sub caudate tractotomy). ness, relationship problems or death of a loved one. The
Neurotic, Stress Related and Somatoform Disorders 47

symptom complex that develops usually involve anxiety not able to perform a task, however if he suddenly deve¬
and depressive symptoms. The symptoms of adjustment lops paralysis, now his guilt will decrease, as it is under¬
disorders include depressed mood, anxiety, worry, a feel¬ stood that paralyzed patient can't work. So, this patients
ing of inability to cope and some degree of disturbance in psyche is unconsciously producing symptoms of paralysis
individuals daily functioning. It is at times difficult to dif¬ to reduce the unpleasant guilt feelings.
ferentiate adjustment disorder from depression (depres¬
Secondary gain: It refers to external psychological moti¬
sion can also follow a negative life event). If the symptoms
vation. For example, this patient who developed sudden
are severe and a diagnosis of depression can be made,
paralysis is now not expected to work outside or make
the diagnosis of depression will always get precedence
money for the family and he is relieved of his duties.
over the diagnosis of adjustment disorder. Also, one
needs to differentiate adjustment disorder from uncom¬ Tertiary gain: It refers to the gain that a third person
plicated bereavement/grief reactions (in uncomplicated derives because of patients symptoms. For example, the
bereavement, the symptoms and dysfunctions which wife of this paralyzed patient starts to get lots of money
develop after death of a loved one are within expected from her parents as they feel sympathetic towards her.
limits, whereas in adjustment disorder the symptoms
and dysfunction are beyond the expectable reaction to Types
the stressor). Other differential diagnosis of adjustment
A. Dissociative amnesia: Here, the main feature is loss of
disorder includes depression, PTSD and brief psychotic
memory. The amnesia is usually for traumatic events
disorders. These diagnoses should be given precedence
of personal significanceQ (such as accidents or unex¬
if their diagnostic criterion are met, irrespective of the
pected bereavements). For example, a rape survivor is
presence of stressors.
not able to recall any thing about her rape.
B. Dissociative fugue: It is characterized by a sudden,
Treatment
unexpected travelQ away from home or work place,
Psychotherapy is the treatment of choice. Supportive psy¬ with inability to recall some or all of one's past. The
chotherapy is commonly used. The medications are used basic self care is maintained0 during the travel and
as an adjuvant to psychotherapy and include antidepres¬ patients behavior during this time may appear com¬
sants and antianxiety drugs. pletely normal to independent observers. Alongside
when asked, the patient may be confused about his
DISSOCIATIVE DISORDERS personal identity or may even assume a new identity
(CONVERSION DISORDERS) _ (e.g. a doctor may
claim that he is in
These disorders were previously classified as "hysteria" In DSM-5, dissociative fugue is not
fact a cab driver a separate diagnosis. Instead it has
however that term is no longer used. Dissociative been made a specifier (special kind
and give a different
disorders are characterized by disturbances in one or of) of dissociative amnesia.
name when asked).
more of mental functions such as memory, identity,
C. Dissociative stupor: Here, the patient is in stupor
perception, consciousness and motor behavior. These
which is caused by psychological factors.
symptoms are produced by the "psyche" (mind) to deal
D. Dissociative trance and possession disorder: It is char¬
with the unconscious conflicts that are producing anxiety.
acterized by loss of sense of identity and full aware¬
These symptoms are produced unconsciously and help
ness of the surroundings. The patient behaves as
the patient to get attention. The symptoms appear
if taken over by another personality such as a god¬
suddenly and are caused by psychological trauma (such
dess or a spirit. For example, a middle aged women
as stressful events or disturbed relationship). Quite often,
claimed that she has been possessed by a goddess and
the genesis of dissociative disorders is explained in terms
demanded that everybody should pray in front of her.
of primary, secondary and tertiary gains. All these gains
E. Dissociative disorders of movement and sensation:
function unconsciously.
Here the patient presents with symptoms that suggest
Primary gain. It refers to internal psychological motiva¬ deficit in motor or sensory functions, however there is
tion. For example, a person might be feeling guilty as he is no evidence of any physical disorder. The symptoms
| 48 Review of Psychiatry

are instead caused indicating that the question was understood. For
by psychological The DSM-4 had the diagnosis example, when asked the color of sky, patient may
factors. Depending of "depersonalization disorder", in answer it red. Although, the answer is not correct but
DSM-5 the name has been changed
on the symptoms, to Depersonalization/Derealization
it is obvious that patient understood that the question
patient may be dia¬ disorder. was about color. Other symptoms include clouding of
gnosed with disso¬ consciousness*3, auditory and visual hallucinations'3
ciative motor disorder (e.g. paralysis, ataxia), dissocia¬ and other dissociative symptoms. Ganser's syndrome
tive convulsions (e.g. pseudoseizures) and dissociative is frequently seen in prisoners, however is not con¬
anaesthesia and sensory loss (e.g. sensory losses, fined only to themQ and can be seen in other popula¬
visual disturbance). Ihe symptoms often do not con¬ tions also.
firm13 with anatomical and physiological principles
(e.g. sensory loss which doesn't confirm to any nerve Treatment
lesion). Usually psychological modalities are used in the treat¬
The DSM-5 uses the diagnosis of conversion disor- ment of dissociative disorders. It is important that patient
derQ (functional neurological symptom disorder) spe¬
is not encouraged to assume a "sick-role" and it must be
cifically for this category and classifies it along with the
emphasized that the patient is normal. The secondary
somatoform disorders. In ICD-10, the term conversion
and tertiary gains should not be allowed otherwise the
disorder is synonymous with dissociative disorder. In
symptoms tend to become persistent. The treatment moda¬
the questions given at the end of this chapter, the
lities include behavioral therapy, abreaction*3 (in abreac-
diagnosis of "conversion disorder" refers to the DSM-5
tion, attempt is made to bring the unconscious memories
diagnosis. La belle indifferenceQ is a phrase used to
and emotions, into conscious awareness using hypnosis,
describe the feeling of indifference which patients of
medications and other techniques) and psychoanalysis.
conversion disorders have towards their symptoms.
The use of drugs is limited. Benzodiazepines, thiopen¬
For example, if a person suddenly has a sensory
tone and amytal have been used for abreaction.
loss, say loss of vision, he would be expected to get
extremely concerned about it, however the patient of
conversion disorder looks completely unconcerned
SOMATOFORM DISORDERS
and this unconcern/indifference towards their symp¬ The patients with somato¬
toms is called "la belle indifference"Q. form disorders typically The DSM-5 has replaced the dia¬
F. Depersonalization/derealization disorder. In dep¬ present with physical gnosis of somatoform disorders with
"somatic symptom and related dis¬
ersonalization patient has a feeling of unreality of symptoms which cannot orders".
self. He feels "as if" he has changed. The patients fre¬ be explained by any
quently report that they feel as if they have detached known medical condition. These patients persistently
from their body and are watching themselves like in request for investigations despite repeated negative
a movie. The depersonalization is often accompanied
findings and reassurances by doctors. The symptoms
by derealization, which is a feeling of unreality of the
are significantly distressing to the patient and cause
external world, as if the world is unreal.
impairment in social and occupational functioning.
G. Dissociative identity disorder (multiple personality dis¬
There are various types of somatoform disorders:
order): Here, two or more distinct personalities exist
A. Somatization disorder: The main feature is presence
within an individual, with only one of them being evi¬
dent at any particular time. The different personali¬ of multiple physical symptoms for which no medical
ties are known as "alters" and the personalities are cause can be ascer- (S-
in DSM-5, the diagnosis of soma¬
unaware of each others existence. tained. According tization disorder has been removed
H. Other dissociative disorders: This category includes to DSM-IV, for a and these patients are given the
diagnosis of "somatic symptom disor¬
Ganser'sQ syndrome. The characteristic symptom is diagnosis of somati¬ der". The criterion of somatic symp¬
approximate answers'3 (vorbeigehen). The approxi¬ zation disorder, the toms disorder include the presence
of somatic symptoms and also exces¬
mate answer are the answers which are not cor¬ following symptoms sive thoughts, feelings and behaviors
rect, but bear an obvious relation to the question, should be present: related to these somatic symptoms.
Neurotic, Stress Related and Somatoform Disorders 49 |
. Four pain symptoms (pain at four different sites)
Two gastrointestinal symptoms (such as nausea,
The location of the
imagined defect is In DSM-5, the diagnosis of body
• dysmorphic disorder has been reor¬
vomiting, belching) usually hair, nose dered. It is now considered as a type
• One sexual symptoms (such as erectile or ejacula- and skin. of OCD and related disorders and has
been removed from the category of
dysfunction)
tory DSM-IV consid¬ somatoform disorders (somatic symp¬
• One pseudoneurological symptom (such as weak¬ ers body dysmor¬ toms and related disorders).

ness, visual disturbance, etc). phic disorder as a


The patient usually refuses to accept the advice or separate entity and a type of somatoform disorder.
reassurance of the doctors that there is no physical However ICD-10 clubs this diagnosis under the
cause of the symptoms. The onset and progression of broader diagnosis of hypochondriasis.
symptoms usually bears a close relationship to unplea¬
E. Somatoform autonomic dysfunction: This type is
sant life events and psychological stressors.
characterized by specific and unexplained autonomic
symptoms such as palpitations, tremors, sweating,
Treatment usually involves psychotherapy. The
belching, etc.
patient should be made aware that the physical
F. Persistent somatoform pain disorder: Here, the
symptoms are expression of underlying emotions
main complaint is persistent and unexplained pain.
and should be helped to cope with the symptoms and
In DSM-5, this category has been subsumed under the
underlying emotions.
new diagnosis of "somatic symptom disorder"
B. Undifferentiated somatoform disorder: This diag¬
G. Pseudocyesis: It is a type of somatoform disorder in
nosis is used when
which the patient has a false belief that she is preg¬
multiple physical
In DSM-5, the diagnosis of undif¬ nant0. There are associated objective signs of preg¬
symptoms are pre¬ ferentiated somatoform disorder has nancy like abdominal enlargement (although umbili¬
sent but full picture been removed and these patients
would also get the broader diagnosis cus does not become everted), reduced menstrual
of somatization dis¬
of "somatic symptom disorder". flow or amenorrhea, subjective sense of fetal move¬
order is not fulfilled.
ments, breast engorgement and labour pains at the
C. Hypochondriasis: This disorder is characterized by
a preoccupation with the fear of having, or the idea expected date of delivery. Some endocrine changes
that one has one or may also be present.
more serious phys¬ In DSM-5, the diagnosis of hypo¬
ical illnesses0. The chondriasis has been replaced by "ill¬ OTHER NEUROTIC DISORDERS_
ness anxiety disorder".
preoccupation per¬ A. Neurasthenia: This disorder is characterized by com¬
sists despite normal investigation0 results as well as
plaints of increased mental and physical fatigue after
doctors reassurances.
mild efforts. The patient is often concerned about
It is important to differentiate hypochondriasis
lowered physical and mental efficiency. Associated
from somatization disorder. The emphasis in hypo¬
symptoms include muscular aches and pain, sleep
chondriasis is on the diagnosis whereas the emphasis
disturbances, irritability, dyspepsia, headache and
in somatization disorder is on the symptoms.
inability to relax. The ICD-10 includes the diagnosis
Also, hypochondriasis must be differentiated from
of fatigue syndromes under the category of neuras¬
delusional disorder (somatic type). In patients with
hypochondriasis the belief is not as fixed as it is in thenia.
delusional disorder. The patient with hypochondriasis B. Chronic fatigue syndrome (myalgic encephalo¬
may doubt his belief atleast for short-term, after a nor¬ myelitis): This syndrome is frequently diagnosed in
mal investigation or medical reassurance. In contrast western countries. The symptoms are severe, debili¬
in delusional disorder, the belief is fixed and totally tating fatigue, malaise, headaches, pharyngitis, low
unshakeable. grade fever, cognitive complaints, gastrointestinal
D. Body dysmorphic disorder: It is characterized by symptoms and tender lymph nodes. The diagnosis
the preoccupation with an imagined defect0 in body of chronic fatigue syndrome is not covered by either
appearance. In case a slight physical anomaly is pre¬ ICD-10 or DSM-5, however the symptoms have some
sent, patient's concern for the same is exaggerated. resemblance to neurasthenia.
50 Review of Psychiatry

C. Culture bound syndrome: These are limited to a Stage 3, the stage of exhaustion: If the stress continues,
particular culture and are not seen world wide. It is the resistance of body gradually decreases and finally col¬
believed that local cultural beliefs and patterns of lapses.
behavior have strong influence on the presentation of Almost all the organ systems may be involved in psy¬
these syndromes. Few common culture bound syn¬ chosomatic disorders. The important ones include:
dromes are: A. Gastrointestinal system: A large number of GI disor¬
ders such as peptic ulcers, Crohn's disease, ulcerative
• Dhat syndrome°: It is prevalent in Indian subcon¬
tinent. The patient has a belief that he is passing colitis are affected by psychological causes. Irritable
semen in urine and this is resulting in physical and
bowel syndrome, which is characterized by symptoms
such as abdominal pain, cramps, alteration of bowel
mental weakness.
habits (diarrhea or constipation) is a well known
• ICoroQ: The patient has a fear that his penis will example of psychosomatic disorder.
retract into the abdomen and would result in death
B. Respiratory system: Asthma, COPD and hyperventila¬
• Latah : This is characterized by automatic obedi¬
tion syndrome are known to have psychological com¬
ence, echolalia and echopraxia. ponent. Hyperventilation syndrome is characterized
D. Factitious disorder (munchausen syndromeQ): Facti¬ by rapid and deep breathing for several minutes and
tious disorder (also known as hospital addiction) is accompanying symptoms of suffocation, giddiness,
a disorder in which patients produces fake symp¬ paraesthesia and syncope due to falling PC02 levels
toms with the sole aim of obtaining medical attention in blood.
(hence called professional patients0). Unlike malin¬ C. Cardiovascular system: Cardiovascular disorders such
gering, in which the motive is usually financial gains as hypertension, coronary artery diseases, cardiac
or avoidance of duty, the patients with factitious dis¬ arrhythmias are known to be affected by psychological
orders have no recognizable motives apart from wish causes. Of particular interest is the association of so
to get medical attention. These patients distort the called type A personality with coronary artery dis¬
history and make stories (pseudologia fantastica) to ease0. The type A personality is characterized by eas¬
convince the doctors. The patients are often from the ily aroused anger, impatience, aggression, competitive
medical and related fields0 and have basic under¬ striving and hostility. Type A pattern is associated with
standing of symptoms/signs of various disorders. a nearly two fold risk of MI and CAD related mortality.
In comparison type B-personality is characterized by
calmness, relaxed attitude, low competitiveness and
PSYCHOLOGICAL FACTORS AFFECTING
lesser chances of coronary artery diseases.
OTHER MEDICAL CONDITIONS_ D. Musculoskeletal system: Disorders like rheumatoid
The concept of psychosomatic disorders (physical disor¬ arthritis, systemic lupus erythematosus are known to
ders caused by or aggravated by psychological factors) have psychological components. Of particular note
has been known for a long time. It is clear that stress is fibromyalgia, a disease characterized by pain and
stiffness of soft tissues such as muscle and ligaments.
can result in many somatic symptoms. Stress is described
The patient often reports of local areas of tenderness,
as any circumstance, that disturbs or is likely to disturb,
also known as "trigger points'! There might be associ¬
the normal physiological or psychological functioning of
ated symptoms such as anxiety, fatigue and inability
an individual.
to sleep.
Hans Selye described a model of stress that is known
E. Other disorders such as endocrinological disorders,
as general adaptation syndrome0. According to this
skin disorders, headaches also have psychological
model, body reacts to stress in three stages.
contributions.
Stage 1, the alarm reaction: This is the immediate response
Treatment: Patients with all forms of somatoform disor¬
characterized by fight or flight response.
ders usually resist psychiatric treatment. The treatment
Stage 2, the stage of resistance: This is also known as is usually focused on helping the patient understand the
stage of adaptation. Here, the body adapts to the stress. effect of psychological factors in the genesis of symptoms
For example, if the stress is starvation, body reduces the while acknowledging that the symptoms are real and dis¬
energy consumption and decreases physical activity. tressing to the patient. Psychotherapeutic techniques like
Neurotic, Stress Related and Somatoform Disorders 51

group psychotherapy, insight oriented psychotherapy, Grief, Bereavement and Mourning


behavior therapy, cognitive therapy and hypnosis may
be useful. Relaxation techniques and stress management Although these terms have been used interchangeably,
training may also be required. they have specific meanings. Bereavement means the
state of being deprived of someone due to death. Grief

DEATH AND DYING_ reaction is the psychological feeling precipitated by the


death of a loved one. Mourning is the process through
When an individual is informed about his impending which grief is resolved. Mourning involves societal prac¬
death, he usually goes through a series of responses. These tices like funerals, burial and memorial services.
stages of death and dying, were proposed by Elizabeth
Kubler-RossQ. Complicated Bereavement (Complicated Grief
Stage 1:Denial and shock— This is characterized by refusal Reactions)
to accept thediagnosis and a reaction of shock.
Complicated bereavement includes prolonged grief
Stage 2: Anger— In this stage patients become irritable and reactions (chronic grief) or extraordinarily intense grief
angry at family members, friends , doctors and even God.
reactions (hypertrophic grief) or delayed grief reactions
Stage 3: Bargaining— In this stage patient try to bargain (delayed grief). Traumatic Bereavement refers to grief
with family members and even God. For example, they that is both chronic and hypertrophic.
may pledge to god that they will regularly go to temples
if god cures them. Bereavement and Depression
Stage 4: Depression— The patient now start showing symp¬ Grief is a complex experience in which both positive emo¬
toms of depression such as sadness of mood, withdrawal tions (happy memories of the deceased) and negative
and suicidal thoughts emotions (sadness) coexist and alternate. In depression,
Stage 5: Acceptance— Finally patient accepts that death the negative emotions predominate and do not change.
is inevitable and their feelings may change to neutral or Also the symptoms in depression are severe and cause
even happiness. significant dysfunction.

QUESTIONS AND ANSWERS

QUESTIONS _ C. Pattern of psychological response to stress


D. Pattern of autonomic nervous system (ANS) and
General physiological response when we are aroused by
a stressful situation
1. All of the following are seen in anxiety except:
(Kerala 1996) 4. Which of the following is the most common psy¬
A. Decreased sweating B. Hyperventilation chiatric disorder? (DNB NEET 2014-15)
C. Cold extremities D. Palpitations A. Anxiety disorder
E. Pupillary dilatation B. Schizophrenia
2. General adaptation syndrome (GAS) is seen in: C. Depression
(AI2012) D. Mania
A. Panic attacks B. Depression
C. Anxiety D. Stressful situations Generalized Anxiety Disorders
3. General adaptation syndrome relates to :(AI2012) 5. All are seen in generalized anxiety disorder
A. How we achieve homeostasis except: (DNB June 2009)
B. How well we adapt to new situations A. Muscle tension
52 Review of Psychiatry

B. Irritability C. Short-term benzodiazepine plus SSRI plus CBT


C. Fear of impending doom D. Long-term benzodiazepine plus venlafaxine
D. Restlessness
6. Drugof choice for generalizedanxiety disorder is: Phobic Anxiety Disorders
(DNBNEET2014-15) 12. Agoraphobia is: (SGPGI2003)
A. Alprazolam B. Buspirone A. Fear of getting caught in places from where
C. Venlafaxine D. Beta-blockers escape would be difficult
7. A 25-year-old lady presentedwith sadness, palpi¬ B. Fear of heights
tation, loss of appetite and insomnia. There is no C. Fear of animals
complaint of hopelessness, suicidal thoughts and D. Fear of closed spaces
there is no past history of any precipitating event. 13. True about social phobia is: (UP 2001)
She is remarkably well in other areas of life. She
A. Fear of closed spaces
is doing her office job normally and her social life
B. Irrational fear of situation
is also normal.What is the probable diagnosis in
C. Irrational fear of activities
this case? (AI2010)
D. Irrational fear of specified objects
A. Generalized anxiety disorder
B. Mixed anxiety depression 14. A middle aged personreportedto psychiatric OPD
C. Adjustment disorder with the complaints of fear of leaving home, fear
D. Mild depressive episode of travelling alone and fear of being in a crowd.
He develops marked anxiety with palpitations
Panic Disorder and sweating if he is in these situations. He often
avoids public transport to go to his place of work.
8. A 30-year-old lady presents with sudden onset
The most likely diagnosis is: (AIIMS May 2006)
breathlessness, anxiety, palpitation and feeling
A. Generalized anxiety disorder
of impending doom. Physical examination does
B. Schizophrenia
not reveal any abnormality. What is the probable
C. Personality disorder
diagnosis in this case? (AIIMS Nov 2010)
D. Agoraphobia
A. Panic attack B. Anxiety disorder
C. Conversion disorder D. Acute psychosis 15. A medical student could not deliver semi¬
nar, fearing his seniors despite knowing that
9. Panic attack is associated with a disturbance inall
they are supportive. He further has difficulty
of the following neurotransmitters except:
(AIIMS Nov 2011) speaking in front of others and also avoids
A. Serotonin
going to the parties. The most likely diagnosis is:
B. GABA (AIIMS 1999, MH2008)
C. Glutamate A. Agoraphobia B. Claustrophobia
D. Dopamine, CCK, pentagastrin C. Social phobia D. Acrophobia

10. Differential diagnosis of panic disorder are: 16. A fifty-year- oldmale feels uncomfortable inusing
(PGIJune 2004) lift, being in crowded places and traveling. The
A. Pheochromocytoma most appropriate line of treatment is:
B. Myocardial infarction (AIIMS Nov 2005)
C. Mitralvalve prolapse A. Counseling
D. Depression B. Relaxation therapy
E. Carcinoid syndrome C. Exposure and response prevention
D. Covert sensitization
11. Which of the following is the most appropriate
treatment for panic disorder? (AIIMS 2009) 17. Treatment of choice in phobic disorder is :
A. Buspirone plus benzodiazepines (AIIMS 1994), (Assam 1999)
B. Benzodiazepines plus supportive therapy A. Psychotherapy
Neurotic, Stress Related and Somatoform Disorders 53

B. Benzodiazepines 24. True about obsessive compulsive disorders are


C. Behavior therapy all except. (DNB NEET 2014-15)
D. 5-HT reuptake inhibitor A. Obsessions are ego-alien
B. Patient tries to resist against obsessions and
18. Agoraphobia is treated with: (PGIDec 2007)
compulsions
A. Systematic desensitization
C. Obsessions are egosyntonic
B. Psychodynamic therapy
D. Insight is present
C. Exposure therapy
D. Relaxation therapy 25. Most common major symptom in obsessive com¬
E. Behavior therapy pulsive disorder is: (DNB NEET 2014-15)
A. Compulsive washing of hand
Obsessive Compulsive Disorder B. Obsessive thoughts of contamination
C. Compulsive checking
19. Feeling of uncertainty and excessive sense of
D. Obsessive precision
responsibility is seen in: (AIIMS May 2015)
A. Generalized anxiety disorder 26. Transmitters mainly involved in OCD is:
B. OCD (AIIMS 1995)
C. Phobia A. GABA B. Norepinephrine
D. Personality disorder C. Dopamine D. Serotonin

20. Abnormal thought possessionis found in: (AI1994) 27. A 15-year-oldboy feels that the dirt has hung onto
A. Organic brain syndrome himwhenever he passes through the dirty street.
B. Hysteria This repetitive thought causes much distress
C. Obsessive compulsive disorder and anxiety. He knows that there is actually no
D. Neurasthenia such thing after he has cleaned once but he is not
21. True statements about obsession: (PGI2003) satisfied and is compelled to think so. This has
A. It is a repetitive thought or image led to social withdrawal. He spends much of his
B. Patient believes that the images or thoughts are time thinking about the dirt and contamination.
imposed by others This has affected his studies also. The most likely
C. Content of obsession are about sex or God diagnosis is: (AI2003)
D. Patient gets disturbed when unable to remove A. Obsessive compulsive disorder
the ideas or thoughts B. Conduct disorder
C. Agoraphobia
22. Which of the following statements differentiates D. Adjustment disorder
obsessional idea from delusion:
(DNB NEET 2014-15, AIIMS Nov 2005) 28. An obsessive compulsive neurosis patient is likely
Obsessional idea is not a conventional belief to develop: (AIIMS 1993)
A.
B. Obsessional idea is held in spite of evidence to
A. Hallucination B. Depression
the contrary C. Delusion D. Schizophrenia
C. Obsessional idea is regarded as senseless by 29. Drug of choice for OCD is: (DNB lune 2009)
patient A. Clomipramine B. Fluoxetine
D. Obsessional idea is held on inadequate ground C. Carbamazepine D. Chlorpromazine
23. Following are the major symptoms of obsessive 30. In obsessive-compulsive disorder, which is not
compulsive disorder: (DNB NEET 2014-15) given: (DNB 2002, Jharkhand 2006)
A. Doubts of contamination A. Clomipramine
B. Pathological doubts B. Haloperidol
C. Intrusive thoughts C. Sertraline
D. All of the above D. Carbamazepine
54 Review of Psychiatry

31. All drugs are used for treatment of OCD except: Post-traumatic Stress Disorder
(ONB2009, PCI1999), (JIPMEr2002) (MAHE2003)
38. Most common disorder(s) after trauma is:
A. Carbamazepine B. Lithium
(PGIMay 2015)
C. Fluoxetine D. Diazepam
A. Major depression B. Mania
32. Drug used for long-term treatment of OCD C. Schizophrenia D. PTSD
includes: (PGIMay 2013) E. Acute stress reaction
A. Clomipramine B. Fluoxetine
39. Which of the following is not a clinical feature of
C. Fluvoxamine D. Citalopram
post-traumatic stress disorder (PTSD)?(AI2008)
E. Trifluperidol
A. Flashbacks B. Hyperarousal
33. Treatment of obsessive-compulsive disorder C. Hallucinations D. Emotional numbing
includes: (PGIDec 2008) 40. Post-traumatic stress-disorder is associated with
A. Exposure and response prevention (PGI2000)
all except:
B. Flooding
A. Flashback
C. Psychoanalytic therapy
B. Severe traumatic injury
D. Supportive psychotherapy involving family C. Re-experiencing of stressful event
members D. Anhedonia
E. Systematic desensitization E. It does not develop after 6 months of stress
34. Treatment of choice for OCD is: 41. False statement about post-traumatic stress dis¬
(DNB 2004, MP 2006) order: (DNB NEET 2014-15)
A. Behavior therapy A. Symptoms develop immediately after the event
B. Drug therapy B. Symptoms include insomnia, poor concentration
C. Psychosurgery C. It is the response to an exceptionally stressful or
D. Combination of behavior and drug therapy catastrophic stimuli
D. Anxiolytics are given only, if anxiety develops
35. A 35-year-female has been diagnosed with
obsessive compulsive disorder and she washes 42. True for PTSD are all except: (PGI2001)
her hands many times a day. Which would be the A. Patients have past history of psychiatric illness
best CBT technique for her treatment? (AI 2012) B. Women are more predisposed
A. Thought stopping B. Responseprevention C. Occur in intellectuals
C. Relaxation D. Exposure D. Feeling of numbness
E. Feeling of detachment
36. Exposure and response prevention technique is/
43. Post-traumatic stress disorder (PTSD) is differ¬
are used in: (PGIMay 2015)
entiated from other disorders by presence of:
A. Schizophrenia B. OCD
C. Phobia D. Mania (AIIMS May 2012)
A. Nightmares about events
E. Depression
B. Autonomic arousal and anxiety
37. A woman comes to psychiatrist with history of C. Recall of events and avoidance of similar experi¬
spending a lot of time in washing her hands. She ences
is distressed about it but says that she is not able to D. Depression
stop washing. This has started to affect her social
44. All are true for PTSD except: (PGI2002)
life as well. What is the best mode of treatment for
A. Hippocampus and amygdala are the brain areas
her? (AIIMS May 2015) involved in PTSD
A. Cognitive behavioral therapy B. Anhedonia
B. Exposure and response prevention C. Depression and guilt
C. Systematic desensitization D. Insomnia and poor concentration
D. Pharmacological agents E. Anxiolytics are the treatment of choice
Neurotic, Stress Related and Somatoform Disorders 55 1
45. Three years back a woman suffered during an This however, provoked anxiety and sadness of
earthquake and she was successfully saved. After mood when she would remain preoccupied with
recovery she has nightmares about the episode thoughts about him. She should be treated with:
and she also gets up inthe night and feels terrified. (AIIMS May 2003)
The most probable diagnosis is: A. Clomipramine
(AIIMS May 2002) B. Alprazolam
A. Major depression C. Electroconvulsive therapy
B. Post-traumatic stress disorder D. Haloperidol.
C. Mania
D. Schizophrenia 51. Ms. B. a 27-year-old nurse had extracurricular
interests in trekking and painting. She broke up
46. Whichofthefollowingisthemosteffectivetreatment relationshipwith her boyfriend. Two months later
modality for post-traumatic stress disorder she lost interest inher hobbies and was convinced
(PTSD)? (AIIMS Nov 2014)
that she would not be able to work again. She
A. Cognitive behavioral therapy
thought life was not worth living and consumed
B. Eye movement desensitization and reprocessing
60 tablets of phenobarbitone to end her life. She
C. Hypnosis
is most likely suffering from: (A 12004)
D. Rational and emotive therapy
A. Adjustment disorder
47. SSR1 are first line treatment for: (PGI2010) B. Acute stress disorder
A. OCD B. Panic disorder C. Depressive disorder
C. Social phobia D. Post-traumatic stress D. Post-traumatic stress disorder
disorder
E. Adjustment disorder 52. Two monthsafter knowingthathissonwas suffering
from leukemia, a 45-year-old father presents with
Grief and Adjustment Disorder sleep deprivation, lethargy, headache, and low
mood. He interacts reasonably well with others,
48. A man coming from mountainwhose wife died 6 but has absented himself from work. The most
months prior says that his wife appeared to him probable diagnosis is: (AI2008)
and asked him to join her.The diagnosis is: A. Depression
(AIIMS 2000) B. Psychogenic headache
A. Normal grief C. Adjustment disorder
B. Grief psychosis D. Somatization disorder
C. Bereavement reaction
D. Supernatural phenomenon Somatoform and Factitious Disorders
49. Which of the following is not a part of Kubler-
53. Which of the following is/are the form/subtype of
Ross's stages of impeding death? (DNB Dec 2010)
somatoform disorder(s)? (PGIMay 2012)
A. Depression B. Bargain
A. Post-traumatic stress disorder
C. Aggression D. Anger
B. Depersonalization
50. An elderly house wife lost her husband who died C. Somatic passivity
suddenly of myocardial infarction couple of years D. Conversion disorder
ago. They had been staying alone for almost a E. Hypochondriasis
decade with infrequent visits from her son and
grandchildren. About a week after the death she 54. Which of the following is not a specific somatoform
heard his voice clearly talking to her as he would disorder? (AIIMS Nov 2011)
in a routine manner from the next room. She A. Somatization disorder
went to check but saw nothing. Subsequently she B. Chronic fatigue syndrome
often heard his voice conversing with her and she C. Irritable bowel syndrome
would also discuss her daily matters with him. D. Fibromyalgia
1 56 Review of Psychiatry

55. A 45-year-old male presents with history of head¬ A. Carcinoma lung


ache, and vague body pains, off and on diarrhea B. Hypochondriacal disorder
and constipation, impotence and tingling and C. Delusional disorder
paresthesia in glove stocking pattern. The most D. Malingering
probable diagnosis is:
60. A young 20-year-old girl presents with complaints
(AI2K, JIPMER 2002, DNB 2004)
of pain in legs, intermittent vomiting, and head¬
A. Hypochondriasis
ache since 2 months. Her physical examination
B. Somatization disorder
was normal. What is the most possible diagnosis?
C. Conversion disorder
(AIIMS Nov 2009)
D. Factitious disorder
A. Generalized anxiety disorder
56. A 41-year-old married female presented with B. Conversion disorder
headache for the last 6 months. She had several C. Somatoform pain disorder
consultations.All her investigations were found to D. Somatization disorder
be within normallimits. She stillinsiststhat there
61. A 40-year-old male is admittedwith complaints of
is somethingwrong inher head andseeks another
consultation. The most likely diagnosis is: abdominal pain and headache. General physical
examination revealed six scars on the abdomen
(AI2003)
A. Phobia from previous surgeries. He seems to maintain
B. Psychogenic headache a sick role and seeks attention from the nurses.
C. Hypochondriasis He demands multiple diagnostic tests including
D. Depression a liver biopsy. The treating team failed to diag¬
nose any major physical illness in the patient.
57. Hypochondriasis is: (AI 1994)
His mental status examination did not reveal any
A. Normal preoccupation with abnormal body
major psychopathology. One of the treating staff
function
recognizedhimto have appeared in severalother
B. Abnormal preoccupation with abnormal body
hospitals with abdominal pain and some other
function
vague complaints. He is most likely suffering from:
C. Normal preoccupation with normal body func¬
(AIIMS Nov 2003)
tion
A. Schizophrenia B. Malingering
D. Abnormal preoccupation with normal body
C. Somatization disorder D. Factitious disorder
function
62. A 30-year-old lady presented to physician with
58. A man with intermittent hiccups feels that he is
complaints of hematuria. On evaluation RBCs
about to die because he is suffering from gastric were found in urine but no cause was found. On
cancer. All his radiological investigations prove further enquiry it was found that she has gone to
the contrary. Which is the most probable diagno¬ many doctors with the same complaints andwould
sis? (DNB NEET 2014-15) demand inpatient care. She would prick her finger
A. Somatization disorder B. Hypochondriasis and mix blood in urine sample. Her diagnosis is:
C. Conversion disorder D. Delusionaldisorder (Karnataka 2011)
59. A 35-year-old male,with premorbid anxious traits A. Malingering
and heavy smoker believes that hehas been suffer¬ B. Factitious illness
ingfrom Tung carcinoma' for a year. No significant C. Dissociative disorder
clinical finding is detected on examination and D. Hypochondriasis
relevant investigations. In the process, he has 63. Munchausen's syndrome by proxy involves:
spent a huge amount of money, time and energy (MH2011)
ingetting himself unduly investigated. He is most A. Drug abuse
likely suffering from: (AIIMS Nov 2004) B. Toxin mediated neuropsychiatric disorder
Neurotic, Stress Related and Somatoform Disorders 57 1
C. Illness caused by care giver .
7 1 Which of the following can differentiate hysterical
D. All of the above fits from epileptic fits? (DPG 2009, Calcutta 2002)
A. Occur in sleep B. Injuries to person
64. Maintainingsickrolebyanymeansisacharacteristic D. Occur when people
C. Incontinence
feature of:
are watching
(JIPMER 2002, Make 2004, Rohtak 2002, DNB 2003)
A. Hypochondriasis 72. The most common form of dissociative hysteria
B. Somatization disorder is: (MH2000)
C. Conversion disorder A. Fugue B. Amnesia
D. Factitious disorder C. Multiple personality D. Somnambulism

73. Psychogenic amnesia is characterized by:


Dissociative Disorders (AIIMS 1997)
(Conversion Disorders) A. Anterograde amnesia
65. La belle indifference is seen in: B. Retrograde amnesia
(DNB NEET 2014-15, AIIMS 1998) C. Both with confabulation
A. Conversion disorder B. Schizophrenia D. Patchy impairment of personal memories
C. Mania D. Depression
74. A person missing from home is found wander¬
66. In conversion disorder, all of the following state¬ ing purposefully. He is well-groomed and denies
ments are true except: (DNB NEET 2014-15) remembering how he reached at the new place.
A. Autonomic nervous system is involved Most likely diagnosis is: (AI2001)
B. There is primary and secondary gain A. Dementia B. Dissociativeamnesia
C. La belle indifference is a feature C. Dissociative fugue D. Schizophrenia
D. Patient does not intentionally produce symptoms
75. An 18-year-old boy came to psychiatry OPD with
67. All are true about conversion disorder except: a complaint of feeling as, if he is changed from
(DNB June 2011) inside. He reports feeling strange as, if he is dif¬
A. Presence of secondary gain ferent from his normal self. He was very tense and
B. Onset in late age anxious yet could not point out the precise change
C. Patient does not consciously produce symptom inhim. This phenomena is best called as:
D. Relation with stress
(AT 2005)
68. Which of the following is a conversion disorder? A. Delusional mood
(DNB June 2009) B. Depersonalization
A. Hysterical fits B. Derealization C. Autochthonous delusion
C. Depersonalization D. Amnesia D. Overvalued idea

69. Inconversion disorders, all are found except: 76. Regarding Ganser's syndrome, which of the fol¬
(DNB NEET 2014-15) lowing statement is true: (AIIMS 1998)
A. Jealousy B. Paralysis A. Repeated lying
C. Anesthesia D. Abnormal gait B. Approximate answers
C. Unconscious episodes
70. Following are included in dissociative disorder:
(PGIJune 2007, 2003)
D. Malingering
A. Multiple personality disorder 77. All are true about Ganser's syndrome except:
B. Fugue (JIPMER/UP 2K, PGI 1999, DNB 1998)
C. Hypochondriasis A. Approximate answer
D. Somatization disorder B. Apparent clouding of consciousness
E. Obsession C. Only found in prisoners
F. Borderline personality D. Hallucinations
!?Ngg»«gB»B8gaiBB3BaJ'Ui ... 1LWIII III MillUIIIIUJ 1

58 Review of Psychiatry

78. Ganser syndrome is a type of: (DNB NEET2014-15) expected that there would be at least some distur¬
A. Dementia B. Malingering bance in professional and social life. Further her
C. Dissociative disorder D. Personality disorder symptoms are not enough to make a diagnosis of
depression. In view of the above, the diagnosis
79. All is true about pseudocyesis except:
of mild depression cannot be made.This patient
(DNB NEET 2014-15)
doesn't have any history of precipitating event
A. Abdominal enlargement
hence the diagnosis of adjustment disorder can
B. Patient is pregnant
be easily ruled out. Few guides are giving the
C. Labor pains at expected date of delivery
answer as generalized anxiety disorder which
D. Amenorrhea
does not make any sense as the only anxiety
80. The difference between malingering and hysteria symptom mentioned here is palpitation. The
is: (AI1994, DNB 2006) core feature of generalized anxiety disorder i.e
A. Hypnosis "generalized and persistent anxiety" is not there.
B. Malingering has poor prognosis Hence, we are left with mixed anxiety depression.
C. Hysteria is more common in females The diagnosis of mixed anxiety depression is
D. Conscious motive in malingering made when there are "symptoms of both anxiety
and depression, but neither set of symptoms is
81. Differential diagnosis of premenstrual tension
severe enough to make an independent anxiety or
includes all of the following except:
depressive disorder diagnosis" This description
(AIIMS Nov 2002)
suits best to the clinical scenario provided here.
A. Psychiatric depressive disorder
8. A.
B. Panic disorder 9. C. Serotonin, norepinephrine and GABA are the
C. Generalized anxiety disorder major neurotransmitters involved. Cholecys-
D. Chronic fatigue syndrome tokinin and pentagastrin (which acts on CCK
receptors) are known to cause panic attacks.
ANSWERS _ 10. A, B, C, E.
1. A. 11. C.
2. D. 12. A.
3. D. 13. B. Social phobia is defined as irrational fear of
4. A. As a group anxiety disorders are the most com¬ social situations. Though it can be said that
mon psychiatric disorders. it also includes certain activities, however
5. C. Fear of impending doom is typically seen inpanic please remember that it's the context (situa¬
attacks. tion) that is central to social phobia and not the
6. A. Benzodiazepines are the drug of choice for gen¬ activity. For example, many patients with social
eralized anxiety disorder. However, it must be phobia have difficulty eating in a restaurant.
remembered that benzodiazepines can cause However, they have no problem doing the same
dependence. The other drugs which can beused activity (i.e eating) when alone. It's the situation
include SSRIs, buspirone and venla-faxine. (i.e. the restaurant) that produces anxiety.
7. B. In this patient the best answer would be 14. D
"mixed anxiety depression". This patient has 15. C, Here the fear is performing in public and there is
some depressive symptoms (sadness, loss of also avoidance of social situations (i.e. avoidance
appetite and insomnia), however the ques¬ of parties).
tion explicitly mentions that there is no hope¬ 16. C. The diagnosis here is agoraphobia as this gen¬
lessness, no suicidal thoughts and that her tleman is uncomfortable with closed places
job and social life is normal. The question (lift), crowded places and also travelling alone.
goes on to add that "she is doing remarkable The best treatment option here is exposure and
well in other areas of life" Please remember response prevention.
that even in a patient with mild depression, it is 17. C.
Neurotic, Stress Related and Somatoform Disorders 59 H
18. A, B, C, D, E. dence is for haloperidol, risperidone, quetiapine
19. B. The cognitive theory of OCD says that the typi¬ and olanzapine. Incomparison, carbamazepine
cal abnormalities in OCD include, "excessive is rarely used in OCD and has very weak evidence
or inflated sense of responsibility" "feeling of in comparison to haloperidol. Hence, the best
uncertainty" and "overestimation of threat" Few answer here would be carbamazepine.
books are giving the answer as "generalized anxi¬ 31. D. Again, we have to choose between carbamaz¬
ety disorder" which is incorrect. epine and diazepam. Now, diazepam is a benzo¬
20. C. Obsessions are disturbances of thought posses¬ diazepine and can improve anxiety temporarily
sion. however it doesn't act at core symptoms of OCD.
21. A, C, D. Whereas, carbamazepine, though has minimal
Obsessions are repetitive thoughts, images evidence, but it has been found to act on core
or impulses. Often the content of thoughts is symptoms of OCD.
about sex or god and patient tries to stop these 32. A, B, C, D.
SSRIs and clomipramine are first line agents.
anxiety provoking thoughts unsuccessfully.
Please remember, that a patient with obsession Trifluperidol doesn't have any evidence in man¬
agement of OCD.
identifies the repetitive thought as his "own
33. A, B, C, D and E.
thought" and not something that is imposed
34. D. A combination of pharmacotherapy and psycho¬
by others. Also remember, if the patient indeed
therapy hasthe best evidence inthe management
believes that the thought has been imposed by
of OCD.
others, it would then be diagnosed as "thought
35. B. The technique is actually exposure and response
insertion" and not an obsession.
prevention. In OCD, the primary aim is to stop
22. C. Obsessions are considered senseless by the
the compulsions; hence response prevention is
patient whereas patient has full belief in the
the better answer here.
delusions. For example, a patient who gets
36. B, C.
obsessive thoughts that "his hands are unclean"
37. B. According to american psychiatric association
understands that his thought is not true and guidelines "The first line treatments for OCD
gets bothered by this repetitive thought whereas are cognitive behavioral therapy that relies on
a patient with "delusion of infidelity" actually behavioral technique of exposure and response
believes that his wife is cheating on him and prevention and serotonin reuptake inhibitors".
continues to believe so irrespective of what Now, this question is just mentioning pharma¬
others say. cological agents without specifying anything
23. D. about which agent. Also few studies have found,
24. C. Obsessions are ego dystonic and not ego syn¬ that exposure and response prevention has more
tonic. lasting effect than pharmacological agents. Said
25. B. that, the choice of treatment between ERP and
26. D. If we have to choose one, it would be serotonin. pharmacological agents depends on patients
27. A. characteristics, which have not been provided,
28. B. Most of the patients with OCD, develop secon¬ hence its tough to choose. However in this case,
dary depression. exposure and response prevention appears to be
29. B. Both SSRIs and clomipramine are considered the best answer.
first line treatment, however due to better side 38. A, D, E.
effect profile, SSRIs are preferred. 39. C.
30. D. Manyguides are giving the answer as haloperidol 40. E. PTSD may have a delayed onset, i.e after 6 months
but that is not the right answer here. The Ameri¬ of trauma.
can Psychiatric Association guidelines clearly 41. A.
state that, if patients do not respond to SSRIs and 42. C. The patients who have a past history of psychiatric
clomipramine, one of the treatment strategy is illness are more predisposed and so are women.
augmentation with antipsychotics. The best evi¬ Thorp is no such correlation with intellect.
60 Review of Psychiatry

43. C. Recall of events and avoidance is quite typical remember patient can develop all kind of psychi¬
of PTSD. Nightmares, autonomic arousal and atric disorders like depression, anxiety, PTSD in
depressive symptoms can be seen in other dis¬ association with grief and if the symptoms are
orders also. severe enough, they should receive separate
44. E. The treatment of choice is cognitive behavioral diagnosis. This patient has psychotic symptoms
therapy. All other statements are correct. (i.e. hallucinations) and should be treated with
45. B. There is history of a traumatic event followed an antipsychotic, haloperidol. The treatment
by intrusion symptoms (nightmares). The most depends on symptoms, in case of occasional
likely diagnosis is post-traumatic stress disorder. anxiety, alprazolam could have been used.
46. A. In case of significant depressive symptoms
47. A, B, C, D, E. antidepressant could have been used, but
48. A. The answer here is debatable. First of all, lets since the psychotic symptoms are prominent,
review some facts. There is no clear cut duration we must use an antipsychotic.
in which grief should get resolved. The most 51. C. This question has been answered wrongly by
accepted duration for grief is 6-12 months. most of the guides. Please remember few basic
However, every single textbook says that grief things about adjustment disorder and depres¬
usually continues beyond that period. Second, sion. Adjustment disorder is always seen after
brief hallucinations can be a part of normal grief, a stressful event, which is usually a negative life
however continuous hallucinations are not seen. event. The symptoms of adjustment disorder
In this case the history is that the man reported are quite similar to depression and include
that wife asked him to join her.The question has depressed mood, anxiety, worry, a feeling of
not mentioned if it was an auditory perception inability to cope and some degree of disturbance
(i.e. he heard voice of wife) or visual percep¬ inindividuals daily functioning. Now, a negative
tion (i.e. he saw his wife) , what was the state life event can also precipitate the depressive
of consciousness (whether he was awake or episode. So, the presence of a stressor cannot
sleeping). In view of above its difficult to even be used to differentiate between adjustment
call this phenomenon as a hallucination. Even disorder and depression. If a patient has the
if we accept it as a hallucination, it appears to be symptoms severe enough to qualify the diagnosis
a single episode. There is no history of any other of depression, depression would always be diag¬
associated symptoms. Hence, the better answer nosedahead of adjustment disorder, irrespective
here would be normal grief. Also, please remem¬ of whether there was a stressor or not. In this
ber that grief and bereavement are often used case patient has severe symptoms such as loss
interchangeably, however strictly speaking, of interest, ideas of hopelessness (patient is con¬
bereavement is a state of loss, whereas grief is vinced that she won't be able to work again) and
the emotional and behavioral response to loss. most importantly suicide attempt, all of which
The question is talking about the behavioral and are highly suggestive of depression. Hence, the
emotional response here. All in all, its a poorly diagnosis would be depressive disorder.
framed and incomplete question. 52. C. Here, the diagnosis is adjustment disorder. The
49. C. symptoms are not severe enough to qualify for
50. D. In this case, death happened "couple of years the diagnosis of depression and there is a clear
ago". The first time she had auditory hallucina¬ history of a stressor (diagnosis of leukemia
tions was after a week of his death and since in son).
then it has been happening. Now, in grief "brief 53. D, E.
hallucinations" can occur however here the According to DSM-IV, the following are the
hallucinations are often and patient is even dis¬ somatoform disorders (1) somatization disorder
cussing the daily matters with the "voice" This (2) conversion disorder (3) hypochondriasis (4)
clearly shows presence of psychotic symptoms body dysmorphic disorder (5) pain disorder
which should be diagnosed separately. Please (6) undifferentiated somatoform disorder (7)
Neurotic, Stress Related and Somatoform Disorders 61

somatoform disorder, not otherwise specified. tious disorders frequently fake symptoms to get
Please remember that inDSM-5, the somatoform the "sick role".
disorders are now referred as somatic symptom 65. A. La belle indifference is a phrase used to describe
and related disorders. the feeling of indifference which patients of con¬
54. A. The classification of somatoform disorders version disorders have towards their symptoms.
mentioned in the preceding answer is fre¬ 66. A. Sensory and motor system are involved and not
quently not used by nonpsychiatrist practition¬ the autonomic nervous system.
ers. These practitioners use other diagnoses, 67. B. The onset of conversion disorder is usually seen in
which are frequently referred to as functional late childhood to early adulthood and is rare after
somatic syndromes. These includechronic fatigue 35 years of age.
syndrome, fibromyalgia and irritable bowel 68. A. The term "hysterical fits" is no longer used in
syndrome. Somatization disorder is not a part modern terminology. The current classificatory
of functional somatic syndromes. system will use the diagnosis of conversion
55. B. This patient has pain symptoms, gastrointestinal disorder with seizure The DSM diagnosis of
symptoms, sexual symptoms and pseudoneuro- conversion disorder can present with either motor
logical symptoms. symptoms, sensory symptoms or convulsions.
56. C. The patient had multiple normal investigations
Please remember that in ICD-10 , conversion
but continues to believe that there is something disorder is another name for dissociative disorders.
So, if we follow ICD-10, all four options are true.
wrong in her head and continues to seek mul¬
But usually, inexams the term conversion disorder
tiple consultations. The most likely diagnosis is
refers to the DSM diagnosis and not the ICD.
hypochondriasis.
69. A. Jealousy is not a neurological sign, the rest three
57. D.
are.
58. B.
70. A, B.
59. B.
71. D. Hysterical fits or dissociative convulsions/sei¬
60. D. The best answer here is somatization disorder.
zures or conversion disorders with convulsions/
The patient has pain symptoms and gastroin¬
seizures do not occur in sleep, are not associated
testinal symptoms. Going by strict definition with any injuries, are not associated with any
of DSM-IV, there should be 4 pain symptoms, 2 incontinence and there is no postseizure amne¬
GI symptoms, 1sexual symptom and 1pseudo- sia or confusion. They usually occur when others
neurological symptoms. However the ICD-10, are watching.
simply says that there should be "multiple and 72. B. Dissociative amnesia is the most common type
variable physical symptoms for which no ade¬ of dissociative disorder.
quate explanation has been found" The other 73. D. In psychogenic amnesia (or dissociative amne¬
plausible option is somatoform pain disorder sia), usually memory is lost for events which have
however it is characterized by only pain symp¬ some personal significance, whereas memories
toms whereas in this patient intermittentvomit¬ for neutral events (e.g. national events) is intact.
ing is also present. Hence, the memory loss is patchy and mostly for
61. D. The history of multiple scars from previous personal memories.
surgeries, seeking attention from nurses, main¬ 74. C.
tenance of sick role, demands for multiple 75. B.
diagnostic tests and identification by a staff all 76. B.
suggest a factitious disorder. 77. C. Though Ganser syndrome is usually seen in
62. B. prisoners but it is not exclusive to them.
63. C. 78. C.
64. D. Sick role means that the patient wants others to 79. B. Patient is not pregnant is pseudocyesis. Though
accept him as "sick" and treat him accordingly she falsely believes that and there are also associ¬
by giving attention and care. Patients with facti¬ ated changes suggestive of pregnancy.
62 Review of Psychiatry

80. D. The symptoms is malingering are produced con¬ their resolution after the onset of menses or
sciously for some conscious motive (e.g. mon¬ within few days of onset of menses. These symp¬
etary gain). In hysteria (dissociative disorders) toms are not present during the other period of
the symptoms are produces unconsciously and menstrual cycles. If the depressive and anxiety
the motive is also unconscious (e.g. attention or symptoms are present throughout the cycle the
love from others). differential diagnose is depression, anxiety dis¬
81. D. Premenstrualtension or Premenstrual syndrome orders like generalized anxiety disorder, panic
is characterized by depressive and anxiety symp¬ disorder. Chronic fatigue syndrome is not a dif¬
toms one week before the onset of menses, and ferential here.
Substance Related
5 and Addictive Disorders

The substance related disorders encompass 10 separate B. Harmful use: It is a state where substance use is caus¬
classes of drugs which includes alcohol, caffeine, canna¬ ing harm but still criterion of dependence are not met.
bis, hallucinogens, inhalants, opioids, sedatives and hyp¬ According to ICD-10, the harmful use is defined as a
notics, stimulants, tobacco and other substances. pattern of substance use which is causing damage to
physical health (e.g. hepatitis due to alcohol use) or
Terminology mental health (e.g. episode of depression secondary
to heavy alcohol consumption).
A. Dependence: It is defined as a pattern in which the
C .Abuse: The DSM-TV, does not use the concept of
use of a substance or a class of substances takes on a
"harmful use" It instead uses the concept of "abuse"
much higher priority for a given individual than other
which is defined as a pattern of substance use that
behaviors that once had a greater value. It encom¬
leads to one or more of the following (1) failure to ful¬
passes behavioral dependence (substance seeking
fil obligations at work, school or home (2) substance
behaviors), physical dependence (physiological
use in situations in which it is physically hazardous
effects of multiple episodes of substance use) and
(such as while driving) (3) legal problems and (4)
psychological dependence (continuous or intermit¬
social or interpersonal problems.
tent craving).
D. Intoxication: A transient condition that develops
According to ICD-10, the presence of three or more
following administration of a substance, in which
of the following in past one year is required for diag¬
various mental functions such as consciousness,
nosis of dependence on a substance:
thinking, perception or behavior are altered.
• Strong desire or sense of compulsion to take a sub¬ E. Withdrawal: Specific symptoms that occur after stop¬
stance (craving)
ping or reducing the amount of substance that has
• Difficulty in controlling substance taking behavior been used regularly over a prolonged period.
in terms of its onset, termination or levels of use
• Withdrawal symptoms (typical physiological Etiology
symptoms that develop when substance use is DSM-5 Update: In DSM-5, the cate¬
gories of "dependence" and "abuse"
reduced or stopped) The development of have been removed and clubbed
• Tolerance (increased doses of substance is required substance use disor¬ under a single diagnostic category of
"substance use disorders".
to achieve the effects originally produced by lower ders is best explained
doses) by a biopsychosocial
• Progressive neglect of alternative pleasures or model. It means that DSM-5 Update: Pathological gam¬
interests because of substance use there is an interaction bling has been included along with
substance related disorders under
• Persistence with substance use despite clear evi¬ of biological factors, the diagnostic entity of "gambling
dence of harmful consequences. psychological factors disorder".
64 Review of Psychiatry

and social factors which results in development of sub¬ Table 1:Absolute alcohol concentration in various preparations.
stance use disorders (dependence, harmful use or abuse).
Concentration of alcohol by
The drugs act on particular receptors and brain path¬ Preparation volume (% ABV)
ways and these receptors and pathways have been found Spirits (whiskey, rum, gin, 40
to play a central role in development of substance use
vodka, brandy, etc.)
disorders. Of particular importance are the dopaminergic Arrack 33
neutrons in the ventral tegmental area which project to Fortified wines 14-20
cortical and limbic regions, especially the nucleus accum- Wines 5—13
bens. This pathway is involved in the sensation of reward Beer (strong) 8-1 1
(or pleasure) and is believed to be the major mediator Beer (standard) 3-4
of effects of substances. This pathway is also known as
"brain reward pathway".
The major neurotransmitters involved in development depending on whether the alcohol was ingested on
of substance used disorders include opioids, catechola¬
an empty stomach (absorption is faster) or with food
mines (particularly dopamine) and y-aminobutyric acid
(absorption is slower).
(GABA).
Mellanby effect?: Studies have shown that intoxicating
The evidence from studies of twin, adoptees and sib¬
effects of alcohol are greater at a given blood alcohol level
lings has also suggested the role of genetic factors in
when BAC (blood alcohol concentration) is increasing than
development of substance abuse.
for the same BAC when the blood alcohol level is falling.
Apart from biological factors, learning and condi¬
tioning is also known to contribute to development Reverse tolerance: This refers to the phenomenon where
of the substance use disorder. The use of substance the intoxicating effects of alcohol are seen progressively
can result in an intense sense of euphoria, it also with lower dosages0. A patient may report that he
frequently alleviates the negative emotions (such as sad¬ gets intoxicated with much smaller amounts of alcohol
ness, anxiety). This results in reinforcement of substance now in comparison to the past. It is believed to be
taking behavior. Other factors like peer pressure, social secondary to decreasing levels of alcohol metabolizing
acceptance, easy availability and the personality type
enzymes secondary to progressive liver dysfunction. A
of the individual also contribute to the development of similar concept of "sensitization" is seen in cocaine,
substance use disorders. amphetamines, opioids and cannabis where in aug¬
mented stimulant response is observed with repeated,
ALCOHOL _ intermittent exposure to a specific drug. It is believed to
be due to changes in the brain reward pathways.
Ethyl alcohol is the active ingredient of alcoholic drinks. Metabolism: About 90% of absorbed alcohol is
The concentration of ethyl alcohol (ethanol) varies across metabolized through oxidation in the liver, the remaining
the preparations. The standard drink or a unit of alcohol 10% is excreted unchanged by the kidneys and the
corresponds to 10 mL of absolute alcohol or 7.8 gram of lungs. The alcohol in alveolar air is in equilibrium with
absolute alcohol (specific gravity of alcohol = 0.78). alcohol in blood passing through pulmonary capillaries,
One standard drink = 1peg (30 mL) of spirits = 1glass hence determining the alcohol levels in breath by breath
(125 mL) of wine = 1 glass (60 mL) of fortified wine = 1/2 analyzer gives a good estimate of blood alcohol levels.
packet of arrack = 1/2 bottle of standard beer = 1/4 bottle The rate of oxidation by the liver is constant and is
of strong beer. around 7-10 gram an hour (which equals to amount of
Arrack is the country made liquor. Fortified wines are alcohol in one standard drink). Alcohol is converted by
prepared by adding brandy to wine. activity of enzyme alcohol dehydrogenase into acetalde-
Absorption: About 10% of alcohol is absorbed from sto¬ hyde, which is further oxidized by aldehyde dehydroge¬
mach and remainder from small intestine0. Peak blood nase into acetate. Acetate is converted to carbon dioxide
alcohol concentration is reached in 30-90 minutes, and water.
Substance Related and Addictive Disorders 65 1
Acute Intoxication include disturbances of consciousness, disorientation to
time, place and person, hallucinations (most commonly
Alcohol is a depressant of the central nervous system. The visual) coarse tremors and autonomic hyperactivity.
excitement that follows alcohol use is due to decrease in
conscious self control. The symptoms and signs of alcohol Alcohol Induced Disorders
intoxication depends on the blood alcohol concentration.
Following symptoms develop: The use of alcohol may be associated with development
Blood levels Symptoms of various mental disorders. Usually alcohol induced dis¬
20-30 mg/dL: Slowness of motor performance and orders, resolve within one month of cessation of alcohol
decreased thinking ability. 30 mg/dLQ is intake. If the symptoms of mental disorder persist beyond
the legal limit for driving in India that, the possibility of an independent mental disorder
30-80 mg/dL: Worsening of motor performance and should be entertained. The following disorders have been
further decrease in thinking ability described:
80-200 mg/dL: Incoordination, judgment errors, mood 1. Alcohol induced psychotic disorders
2. Alcohol induced bipolar disorders
lability
3. Alcohol induced depressive disorders
200-300 mg/dL: Nystagmus, slurring of speech, alcoholic
4. Alcohol induced anxiety disorders, alcohol induced
blackoutsQ
sleep disorder
>300 mg/dL: Impaired vital signs and possible death
5. Alcohol induced sexual dysfunction
Alcoholic blackout: It refers to anterograde amnesia*3 6. Alcohol induced neurocognitive disorders.
seen during intoxication. The person is unable to recall
Alcohol induced neurocognitive disorders: Long-term
the events that happened when his blood alcohol levels
alcohol use can cause amnestic disorders characterized
were between 200-300 mg/dL.
by disturbances in short-term memory. The classic names
for alcohol induced amnestic disorders are Wernicke's
Alcohol Withdrawal encephalopathy and Korsalcoffs syndrome.
It refers to the symptoms which develop after cessation A. Wernicke's encephalopathy: It is the acute neurolo¬
of alcohol intake. In most patients the following sequence gical complication characterized by the following
is seen, though all symptoms do not necessarily occur in symptoms (pneumonic GOA):
all patients. G: Global confusion*3
O: Ophthalmoplegia,*3 usually 6th nerve palsy (sec¬
After6-8 hours: The classic and most common sign of
ond most common is 3rd nerve palsy) causing,
alcohol withdrawal is tremulousness (coarse tremors)*3.
horizontal nystagmus and gaze palsy)
Other symptoms include gastrointestinal symptoms (like
A: Ataxia*3
nausea and vomiting), sympathetic autonomic hyper¬
Although Wernicke's encephalopathy can be com¬
activity including arousal, anxiety, sweating, hypertension,
pletely reversed with treatment, often residual ataxia*3
mydriasis and tachycardia.
and horizontal nystagmus remain despite treatment.
After 12-24 hours: Alcoholic hallucinosis*3. It refers to Wernicke's encephalopathy may clear spontaneously in
hallucinations in the absence of any disturbances of cons¬ days to weeks or progress to Korsakoff's syndrome.
ciousness. Usually auditory hallucinations are present. B. Korsakoff's syndrome: It is the chronic neurological
After 24-48 hours: Alcohol withdrawal seizures. The complication of long-term alcohol use. It is charac¬
seizures are usually generalized and tonic-clonic. Usually terized by impaired recent memory, anterograde
patients have more than one seizures in a span of 3-6 amnesia*3 (inability to form new memory), retro¬
hours, hence often the term cluster seizures is used for grade amnesia*3 (inability to recall old memories)
alcohol withdrawal seizures. and confabulations*3 (making of false stories to fill
After 48-72 hours: Delirium tremens. Alcohol withdrawal memory gaps, which is unintentional). The antero¬
delirium is a medical emergency and if untreated the grade amnesia is much more prominent than the
mortality rate is around 20%. The symptoms and signs retrograde amnesia.
66 Review of Psychiatry

The pathophysiology for both Wernicke's syndrome Blood alcohol concentration is usually measured
and Korsakoff's syndrome is thiamine deficiency. The using breath analyzers. It can also be estimated by
neuropathological lesions are usually symmetrical and using Widmark formula, if the amount of alcohol
involve mammillary bodiesQ. Other sites of lesion include consumed and body weight is known.
thalamus, hypothalamus, midbrain, pons, medulla, fornix • Carbohydrate deficit transferrin (CDT'): The most
and cerebellum. sensitive and specific laboratory test for the identi¬
The treatment of Wernicke's encephalopathy is high fication of heavy drinking is elevated blood levels
dose of parenteral thiamine. Treatment of Korsakoff of carbohydrate deficit transferrin.
syndrome is oral thiamine for 3-12 months. Only around
• Gamma-glutamyl transferase (GGT): Elevated levels
20% of patients with Korsakoff syndrome recover.
of GGT are again suggestive of heavy drinking. The
C. Marchiafava bignami disease: It is a rare neurologi¬
levels of both CDT and GGT return towards normal
cal complication of long-term alcohol use. It is charac¬
within days to weeks of stopping drinking.
terized by epilepsy, ataxia, dysarthria, hallucinations
and intellectual deterioration. The pathophysiology is • Mean corpuscular volume: MCV is frequently ele¬
vated in individuals who indulge in heavy drinking.
demyelination of corpus callosum, optic tracts and
cerebellar peduncles. • Other test include elevated levels of ALT (alanine
aminotransferase) and alkaline phosphatase, which
indicate liver injury secondary to heavy drinking.
Evaluation
A. Screening test: One of the most commonly used Treatment
screening test is CAGE questionnaire*2, which
The treatment of alcohol dependence is done in the
includes the following four questions:
following phases.
• Have you ever felt that you should Cut down on A. Detoxification: It is the first phase of treatment which
your drinking? involves management of withdrawal symptoms.
• Have people Annoyed you by criticizing your drink¬ The usual duration of detoxification is 7-14 days.
ing? Benzodiazepines*2 are the drugs of choice (particu¬
• Have you ever felt bad or Guilty about your drink¬ larly chlordiazepoxide*2) for all the withdrawal symp¬
ing? toms ranging from common ones like tremors and
• Have you ever had a drink first thing in the morning nausea to severe withdrawal symptoms like alcohol
to steady your nerves or to get rid of hangover (Eye withdrawal seizures and delirium tremens. In addi¬
opener)? tion vitamins (particularly thiamine) must be given as
A positive response on two or more than two of the patients usually are deficient in vitamins.
above questions, is suggestive of alcohol use disorder. Carbamazepine can also be used in place of benzo¬
Another commonly used screening test is AUDIT diazepines however other anticonvulsants do not have
(alcohol use disorders identification test). Others tests any role. The antipsychotics can be used in patients
such as SADQ (severity of alcohol dependence ques¬ with delirium tremens and alcoholic hallucinosis.
tionnaire) are used to determine the severity of depen¬ B. Maintenance of abstinence: After the completion of
dence. detoxification, the next phase involves long-term
treatment to maintain the abstinence. It involves both
B. Diagnostic markers: Apart from the screening tests,
pharmacological and nonpharmacological treatment.
the blood test may also help in the identification of
heavy drinkers who are susceptible to development • Pharmacological treatment: The drugs used are of
two types:
of alcohol use disorders.
a. Deterrent agents: The most commonly used
• Bloodalcohol concentration: It can be used to judge deterrent agent is disulfiram*2. It is an irre¬
tolerance to alcohol. For example, if a person has versible inhibitor of aldehyde dehydrogenase,
high blood alcohol concentration without showing the enzyme which metabolites acetaldehyde.
any signs of intoxication, it indicates the presence Acetaldehyde is the first breakdown product of
of tolerance and high chances of presence of alco¬ alcohol. If a patient who is on disulfiram, con¬
hol use disorders. sumes alcohol, it results in accumulation of toxic
Substance Related and Addictive Disorders 67

levels of acetaldehyde and causes a number progress to subcutaneous administration, once he is not
of unpleasant signs and symptoms, termed as able to find any patent vein. The subcutaneous route is
disulfiram ethanol reaction (DER). known as "skin popping".
Other deterrent agents include citrated cal¬
cium carbimide and metronidazole. Intoxication
b. Anticraving agents: These agent reduce craving,
which is an important reason for relapse. The Opioids when taken (especially intravenously) produce a
anticraving agents include naltrexone0, acam- feeling of intense euphoria. The other symptoms include
prosate0, topiramate, serotonergic agents like a feeling of warmth, heaviness of extremities and facial
fluoxetine and baclofen. flushing. This initial euphoria is followed by a period of
• Nonpharmacological treatment: These are psycho¬ sedation (known as "nodding off").
social treatment methods and include: Opioids overdose can be lethal due to respiratory
a. Cognitive behavioral therapies: A large number depression. The symptoms of overdose include coma,
of therapies have been found to have efficacy in slow respiration, hypothermia, hypotension0, bradycar¬
maintaining abstinence. These include motiva¬ dia, pin point pupils, cyanosis.
tional enhancement therapy, relapse preven¬
tion model and cognitive therapy. Withdrawal Symptoms
b. Alcoholic anonymous: It is a self help group,
The sudden stopping of opioids after prolonged use or
which follows 12 steps to quit alcohol use. The
intake of opioid antagonists like naltrexone can produce
members include patients who have recovered
withdrawal symptoms. The short-term use of opioids
from alcoholism, current alcohol users and also
decreases the activity of noradrenergic neurons and the
volunteers. long-term use results in compensatory hyperactivity.
c. Family therapy When opioids are suddenly stopped, there are symptoms
d. Group therapy of rebound noradrenergic hyperactivity. This hypothe¬
sis also explains the mechanism of action of clonidine
OPIOIDS _ (alpha-2 adrenergic receptor agonist, which decreases
The term opiates is used to describe the psychoactive norepinephrine release) in management of opioid with¬
alkaloids (like morphine and codeine) which are present drawal.
in opium (derived from papaver somniferum, the poppy The withdrawal symptoms usually appear around
plant). The term opioids is a broader term which also 6-8 hours0 after the last dose, peak during the second
includes synthetic compounds like heroin and metha¬ or third day and subside during the next 7-10 days. The
done, which share the action and effects of opiates. withdrawal from opioids produces a flu-like syndrome0
Heroin (diacetyl morphine) is the most commonly0 with the following symptoms.
abused opioid. Since, it is more lipid soluble than mor¬ 1. Lacrimation0, rhinorrhea0, sweating, diarrhea0
phine, it crosses blood brain barrier faster and has a more 2. Yawning and piloerection0
rapid onset of action. Heroin was initially used as a treat¬ 3. Pupillary dilation0
ment for morphine addiction, however, it was realized 4. Muscle cramps and generalized bodyache
that dependence forming potential of heroin is higher 5. Insomnia0, anxiety, hypertension and tachycardia
than morphine. The street names of heroin includes 6. Nausea, vomiting and anorexia.
"smack" and "brown sugar" amongst others. The street
forms are often impure and have adulterants like starch Treatment
(fructose and sucrose), quinine, chalk powder, paraceta¬
mol and talcum powder, etc. A. Detoxification: In this stage, the main focus is on
Opioids can be taken orally, snorted intranasally the management of withdrawal symptoms. The
(also called chasing the dragon), and injected intrave¬ medications used are usually long acting opioids like
nously or subcutaneously. The intravenous users tend methadone0 or buprenorphine. Both medications,
to gradually shift from peripheral veins to larger veins in view of their agonist action at opioid receptors,
(a phenomenon called mainlining0). The user may suppress the withdrawal symptoms. Other opioids
68 Review of Psychiatry

like dextropropoxyphene can also be used. Usually CANNABIS _


detoxification medicines are required for 2-3 weeks. Cannabis is derived from the hemp plant, cannabis
Another method is use of clonidineQ for detoxification. sativa. The plant has several varieties named after the
However, clonidine provides considerably less reduction regions where it is found (e.g. cannabis sativa indica in
in symptoms in comparison to buprenorphine or metha¬ India, cannabis sativa americana in USA). Cannabis is the
done. Clonidine is thus mostly used as an adjunct to most commonly used illegal drug5 in the world and in
methadone or buprenorphine during detoxification. India. The street names include joints, marijuana, grass,
Accelerated detoxification: In this method, initially low pot, weed, etc.
doses of naltrexone is given to patient. Naltrexone being The active ingredient, which is responsible for the
an opioid antagonist, produces severe withdrawal symp¬ psychoactive effects of cannabis is 5-9 tetrahydrocan¬
toms. After that, clonidine is used to control the symp¬ nabinol (THC)5. The various preparations of cannabis
toms. This method reduces the detoxification period to includes.
4-5 days. Table 2: THC concentration in various cannabis preparations.
B. Maintenance treatment. It follows the detoxification Cannabis preparation THC content (%)
and the aim is to prevent the relapse. There are two Bhang (derived from dried leaves) 1
different pharmacological approaches for mainte¬ Ganja (derived from inflorescence) 1-2
nance phase. Hashish/Charas (derived from resinous 8-14
• Opioid substitution therapy: In this method, the exudates)
illicit, parenterally administered and short acting Hash oil (lipid soluble plant extract) 15-40
opioids (like heroin) are replaced by medically safe,
The cannabis can be ingested orally or is more com¬
orally taken and long acting opioids. The long act¬
monly smoked. Itis unsuitable for intravenous use because
ing opioids such as methadone, buprenorphine are of poor solubility in water and risk of anaphylaxis due to
mostly used. Levo alpha acetylmethadol was also undissolved particulate matter.
used in past, however it has since been stopped as
it is known to cause torsades de pointes. Intoxication
These orally used opioids are given at govern¬
It is characterized by euphoria, subjective sense of
ment approved centres. Though the patient conti¬
slowing of time, sense of floating in air, reddening of
nues to remain dependent, however he is protected
conjunctiva5 (due to dilatation of conjunctival blood
from medical consequence of parenteral opioids vessels), increased appetite and dryness of mouth.
(like HBV, HIV infection) and does not need to Other symptoms include depersonalization, derealiza¬
indulge in criminal activities to fund the illicit opi¬ tion, synesthesia5 (cross over of sensory perceptions. For
oid use. example, patient may report that he is "seeing" music and
• Opioid antagonist treatment: Naltrexone5 can be "hearing" lights).
given to the patient after detoxification is complete. Sometimes, after consumption of cannabis, the per¬
The rationale is that naltrexone will block the opi¬ son might feel restless, fearful, extremely anxious (similar
oid receptors and any opioid use would fail to pro¬ to panic attack) and may feel that he is about to go crazy.
duce the euphoric response and hence would not This unpleasant experience is known as "bad trip"5.
be repeated.
• Nonpharmacological approaches like cognitive Withdrawal Symptoms
behavioral therapy, narcotic anonymous (12 step It was earlier believed that cannabis doesn't cause physi¬
self help groups), family therapy and group therapy cal dependence and produces no withdrawal symptoms,
are also useful. however recent studies have shown that there are mild
C. Overdose treatment: The opioids are lethal in over¬ withdrawal symptoms within 1-2 weeks of cessation and
dose. The drug of choice for treatment of opioid over¬ include insomnia, anxiety, decreased appetite, irritability,
dose is i.v. naloxone5 (short acting opioid antagonist). etc.
Substance Related and Addictive Disorders 69

Cannabis Related Disorders Withdrawal Symptoms


1. Cannabis induced psychotic disorder: It is also some¬ Hallucinogens do not cause any physical dependence,
times referred to as "hemp insanity". The patient has hence tolerance and withdrawal symptoms are not seen.
psychotic symptoms such as delusions and hallucina¬ The use of hallucinogens like LSD can be associated
tions. with flashback phenomenon0 which refers to recurrence
2. Cannabis induced anxiety disorders. of LSD use experience in the absence of current LSD use.
3. Flash back phenomenonQ: It is characterized by a
recurrence of cannabis use experience in the absence Treatment
of current cannabis use.
Mostly psychotherapeutic techniques are used to prevent
4. Running amok0: It is described as development of
rage following cannabis use, in which person may
relapse.
hurt or even kill others in an indiscriminate fashion.
5. Amotivational syndrome°: It is characterized by an STIMULANTS __
unwillingness to persist in any task, whether at school
or at work. The patient appears uninterested, lethargic Cocaine
and apathetic. Cocaine is derived from the plant erythroxylum coca.
Sigmund Freud0 had studied its pharmacological effects
Treatment and is also believed to be addicted to cocaine for a long
time. Coca cola used to contain cocaine till 1903 after
As withdrawal symptoms are mild, no medications are
which it ceased to be an ingredient.
usually used. If required, benzodiazepines can be used Cocaine was initially used as a local anesthetic0 and
for short-term. still is used in eye, nose and throat surgery0. The local
Long-term treatment usually involves the psycho¬ anesthetic effect is mediated by blockade of fast, sodium
therapeutic approach and patient maybe offered cognitive channels.
behavioral therapy, family therapy or group therapies. Cocaine acts primarily by blocking dopamine recep¬
tors0 (D1 and D2) and increasing dopamine concentra¬
HALLUCINOGENS _ tion in synaptic cleft. It is also an inhibitor of uptake of
norepinephrine and hence has significant sympathomi¬
This class includes a variety of drugs like LSD (Lysergic metic effect. It causes marked vasoconstriction of peri¬
acid diethylamide), mescaline, psilocybin, methylene- pheral arteries, which results in hypertension0, further,
dioxyamphetamine (MDMA, also called ecstasy), phen- vasoconstriction of the epicardial coronary arteries, can
cyclidine (angel dust) and ketamine. lead to ischemic myocardialinjury. Cocaine use can also
cause seizures. Cocaine (most common) and ampheta¬
Intoxication mines (second most common) are the substances mostly
associated with seizures.
The characteristic symptoms of LSD (and other halluci¬
Cocaine is usually inhaled (known as snorting). Due
nogens) intoxication are depersonalization, derealization, to its vasoconstrictive properties nasal inhalation of
synesthesia0 (also called as reflex hallucinations wherein cocaine causes nasal congestion and can even result in
patient may report cross over of sensory perceptions), nasal septal perforation0. Long-term use can also cause
illusions and hallucinations, autonomic hyperactivity fea¬ jet black pigmentation of tongue0.
tures such as pupillary dilatation, tachycardia, sweating, Other methods of intake are smoking (known as free-
palpitations, tremors, etc. basing0) and subcutaneous or intravenous injections.
Similar to cannabis,at times, patient may become Freebasing involves mixing street cocaine (which usu¬
restless, fearful and may develop panic reaction (bad ally has procaine or sugar as adulterants) with freebase
trip)0. Usually patient can be calmed down by reassur¬ (chemically extracted pure cocaine). A particular potent
ance. However in cases with extreme agitation, benzodia¬ way is consumption of cocaine and heroin (called speed-
zepines or antipsychotics may be required. ball) together.
I 70 Review of Psychiatry

Crack, is a freebase form of cocaine which is smoked. which includes smoking, chewing, applying, sucking
It is extremely potent and even a single use can cause and gargling. Beedi smoking is the most common form
intense craving. followed by cigarette smoking. The active ingredient of
Intoxication: The intoxication is characterized by eupho¬ tobacco, which causes addiction is nicotine. The constitu¬
ria, pupillary dilatation, tachycardia, hypertension and ents responsible for cardiovascular disorders are nicotine
sweating. Acute intoxication with moderate to high and carbon monoxide.
dose of cocaine may be associated with paranoid idea¬ Nicotine has a stimulant action and improves the
tions, auditory hallucinations0 and visual illusions. The attention, learning, reaction time and problem solving
patients also occasionally report of tactile hallucinations ability.
(feeling of insects crawling under the skin), also known as The withdrawal symptoms can develop within two
cocaine bugs, (also known as formication and magnan hours of smoking the last cigarette and peak in 24-48
phenomenon0). hours. These symptoms include craving for nicotine, irri¬
tability, anxiety, difficulty concentrating, bradycardia0,
Withdrawal symptoms: Cocaine causes strong psycho¬
drowsiness and paradoxical trouble sleeping, increased
logical dependence0 however physiological dependence
appetite and weight gain.
(tolerance and withdrawal symptoms) is mild0 in
comparison. The withdrawal symptoms includes feeling
low, exhaustion, lethargy, fatigue, insatiable hunger. The Treatment
most severe withdrawal symptom is depression, which
Pharmacotherapy
can be associated with suicidal ideation.
1. Nicotine replacement therapy: It is used to relieve
Cocaine inducedpsychotic disorder. It is most commonly
seen with intravenous use and crack users. The hallmark the withdrawal symptoms by substituting nicotine in
is paranoid delusions (delusion of persecution) and tobacco with nicotine in safer forms as they do not
contain other harmful constituents present in tobacco.
auditory hallucinations0. Visual andtactile hallucinations
(cocaine bugs) can also be present. The disorder is quite The various preparations include nicotine gums, nico¬
tine lozenges, nicotine patches, nicotine inhalers and
similar to paranoid schizophrenia0 in its presentation.
nicotine spray).
Treatment: The withdrawal symptoms are usually mild 2. Medications which can be used include bupropion
and no specific pharmacological agents reduces the (first line0) and clonidine and nortriptyline (second
intensity of withdrawal. Treatment mostly relies on line). Varenicline is a new medicationwhich has been
psychotherapeutic interventions like cognitive behavioral approved for use in tobacco dependence. Varenicline
therapy, group therapy, and support groups such as acts as an agonist at a 7 nicotinic acetylcholine recep¬
narcotic anonymous. tors and partial agonist on a4(32 receptors.

Apart from medications behavioral therapy is also


Amphetamines considered beneficial.
The major amphetamines include dextroampheta¬
mine, methamphetamine. Methylphenidate is also an
amphetamine like compound. Amphetamines are used
OTHER DRUGS _
1. Inhalants or volatile solvents: These include gaso¬
to increase performance and induce a euphoric feeling.
line (petrol), glues, thinners, industrial solvents. These
Long-term use can result in amphetamine induced psy¬
chotic disorder, whose hallmark is presence of paranoid solvents are soaked in a cloth and than are sniffed
delusions (delusion of persecution0) and auditory hal¬ (vapors are inhaled). It is more common seen in
lucinations0. children and adolescents. Long-term use may cause
irreversible damage to livers and kidneys, peripheral
TOBACCO _ neuropathy and brain damage.
2. Benzodiazepines and other sedative hypnotics:
It is the most commonly used substance in India (caf¬ Benzodiazepines can produces physical and psy¬
feine not considered) and is used in a variety of ways chological dependence. The withdrawal symptoms
Substance Related and Addictive Disorders 71 1
usually include anxiety, irritability, insomnia and in with feeling of improved efficiency, increased energy
some cases seizures. The treatment usually involves levels and concentration. Excessive use can produce
anxiety, restlessness, irritability. Caffeine can also
slow tapering and then stopping of benzodiazepines
produce physiological dependence and withdrawal
along with supportive measures.
symptoms include anxiety,irritability, mid depressive
3. Caffeine: Caffeine is the most widely used psycho¬
active substance worldwide. Caffeine use is associated symptoms, nausea and vomiting.

QUESTIONS AND ANSWERS

QUESTIONS _ Alcohol
Substance Use Disorders 6. Irresistible urge to drink alcohol is known as:
(DNB June 2011)
.
1 Which of the following is not an important factor A. Kleptomania B. Pyromania
in development of substance dependence? C. Dipsomania D. Trichotillomania
(AIIMS Nov 2009)
A. Personality B. Family history 7. All of the following statements are true about
blackouts except: (AIIMS May 2014)
C. Peer pressure D. Intelligence
A. The person appears confused to the onlookers
2. Not included in definition of substance abuse B. Remote memory is relatively intact during the
syndrome: (PGIMay 2011) blackout
A. Withdrawal symptom C. It is a discrete episode of anterograde amnesia
B. Use despite knowing that it can cause physical/ D. It is associated with alcohol intoxication.
mental harm
C. Tolerance to drug 8. A patient taking 120 mL alcohol everyday sincelast
D. Recurrent substance abuse 12 years is brought to the hospitalby his wife and is
E. Use despite substance related legal problems diagnosed to have alcohol dependence syndrome.
Which of the following drug should be avoided in
3. All of the following are criteria for substance
the management? (AIIMS Nov 2014)
dependence except: (AI2012)
A. Phenytoin B. Disulfiram
A. Repeated unsuccessful attempts to quit the sub¬
C. Naltrexone D. Acamprosate
stance
B. Recurrent substance related legal problems/use 9. All of the following are true about alcohol depen¬
of illegal substances dence syndrome except: (DNB NEET 2014-15)
C. Characteristic withdrawal symptoms; substance A. No tolerance
taken to relieve withdrawal B. Withdrawal symptoms
D. Substance taken in larger amount and for longer C. CAGE questionnaire
than intended D. Physical dependence
4. Symptomatic treatment is not required in with¬ 10. First symptom to appear in alcoholwithdrawal is:
drawal of: (AI 1998) (AIIMS May 2015)
A. Cannabis B. Morphine A. Visual hallucinations
C. Alcohol D. Cocaine B. Sleep disturbance
5. Drugs which cause both physical and psychologi¬ C. Tremors
cal dependence are: (DNB NEET 2014-15) D. Delirium
A. Opioids B. Alcohol 11. Most common symptom of alcohol withdrawal is:
C. Nicotine D. All of the above (DNB NEET 2014-15, AI 2007)
I 72 Review of Psychiatry

A. Bodyache B. Tremor 20. Wernicke's encephalopathy is due to deficiency of:


C. Diarrhea D. Rhinorrhea (DNBNEET 2014-15)
A. Folic acid B. Thiamine
12. Which of the following is characteristic of alcohol
C. Ascorbic acid D. Pyridoxine
withdrawal? (AIIMS 1991)
A. Hallucination B. Illusion 21. Which of the following is included inthe classical
C. Delusion D. Drowsiness triad of Wernicke's encephalopathy?
A. Peripheral neuropathy (DNB NEET 2014-15)
13. Widmark formula is used for: (AIIMS 1993)
B. Autonomic dysfunction
A. Opium B. Cannabis
C. Alcohol C. Ataxia
D. Amphetamine
D. Abdominal pain
14. Male started drinking alcohol at age of 20 years,
22. Not affected in Wernicke's disease:
presently taking 3 quarters daily over 30 years,
(DNB NEET2014-15)
complains that now he gets the kick in 1quarter.
A. Hypothalamus B. Thalamus
Probable diagnosis is: (AIIMS Nov 2012)
A. Withdrawal C. Hippocampus D. Mammillary bodies
B. Mellanby phenomenon 23. An alcoholic patient comes to your office, he
C. Reverse tolerance can't tell his name. There is gross incoordination
D. Cross tolerance in walking, and his eyes are deviated to one side.
15. Psychiatric complications of alcohol dependence What is the probable diagnosis? (Bihar 2006)
are: (PGI2001) A. Wernicke's encephalopathy
A. Anxiety B. Suicide B. Korsakoff's psychosis
C. Depression D. Schizophrenia C. Alcoholic hallucinosis
E. Mania D. Delirium tremens

16. Not a feature of delirium tremens is: (AI2011) 24. Feature(s) of Korsakoff psychosis:
A. Confusion (clouding of consciousness) (PGINOV2014)
B. Visual hallucinations A. Confabulation B. Retrograde amnesia
C. Coarse tremors C. Ophthalmoplegia D. Delirium
D. Oculomotor nerve palsy (ophthalmoplegia)
25. Korsakoff syndrome true is/are:
17. True about delirium tremens: (PGIJune 2005) (DNB NEET2014-15)
A. Clouding of consciousness A. Can be seen in chronic alcoholics
B. Coarse tremors B. Absence of intellectual decline
C. Chronic delirious behavior C. Chronic amnestic syndrome
D. Hallucination
D. All of the above
E. Autonomic dysfunction
26. All are relatively normal in Korsakoff's psychosis
18. Wernicke's encephalopathy involves which part
of central nervous system: (PGI2000) except: (MAHE 2003, KA 2003; J&K 2000)
A. Mammillary body B. Thalamus A. Implicit memory B. Intelligence
C. Frontal lobe D. Arcuate fasciculus C. Language D. Learning

19. A 45-year male with a history of alcohol depen¬ 27 . True statement about Korsakoff's psychosis is:
dence presents with confusion, nystagmus and (Rohtak 2000; JIPMER 1999) (UP 1999; PGI1997)
ataxia. Examination reveals 6th cranial nerve A. Severe anterograde + Mild retrograde memory
weakness. He is most likely to be suffering from: defect
A. Korsakoff's psychosis. (AI2005) B. Mild anterograde + severe retrograde memory
B. Wernicke encephalopathy. defect
C. De Clerambault syndrome. C. Only anterograde memory defect
D. Delirium tremens. D. Only retrograde memory defect
Substance Related and Addictive Disorders 73 1
28. InKorsakoff psychosis all are seen except: day he had GTCS followed by another episode of
A. Loss of remote memory (JIPMER 1998) GTCS after few hours. Drugwhich should be given
B. Loss of intellectual function but preservation of to control the symptoms: (AIIMS May 2013)
memory A. Sodium valproate B. Phenytoin
C. Lack of insight, unable to understand the dis¬ C. Diazepam D. Clonidine
ability 33. In alcoholwithdrawal drug of choice is:
D. Reversible state (DNB NEET 2014-15, PGIJune 2007, AIIMS 1990)
29. A 35-year-old male comes with h/o 10-years of A. Haloperidol B. Chlordiazepoxide
alcoholismand past history of ataxiawith bilateral C. Naltrexone D. Disulfiram
rectus palsy. He was admitted and treated. What
34. Drugs used for treatment of delirium tremens is/
changes can be expected to be seen insuchcondi¬ are: (DNB NEET 2014-15, MCIscreening)
tion? (PGIJune 2008) A. Diazepam B. Quetiapine
A. Progression to Korasakoff's psychosis D. Both A and C
C. Chlordiazepoxide
B. Residual ataxia in 50% of patients
C. Extraocular palsy disappears in hours 35. All of the following agents are used in the treat¬
D. Immediate relief from symptoms ment of alcohol dependence except'.
(DNB NEET 2014-15, AI2011)
30. A 30-year-oldmalewith historyof alcohol abuse for A. Flumazenil B. Acamprosate
15 years is brought to the hospital emergency with C. Naltrexone D. Disulfiram
'complaints of fearfulness, misrecognition, talk¬
36. Inpatients of substance-abuse, drugs used are:
ingto self, aggressive behavior, tremulousness and
(PGI2002)
seeing snakes and reptiles that are not visible to
A. Naltrexone B. Naloxone
others aroundhim.There is history of last drinking
C. Clonidine D. Lithium
alcohol two days prior to the onset of the present
E. Disulfiram
complaints. He is most likely suffering from:
(AIIMS Nov 2003) 37. All are anticraving agent for alcohol except'.
A. Delirium tremens (AIIMS May 2009)
B. Alcoholic hallucinosis A. Lorazepam B. Naltrexone
C. Schizophrenia C. Topiramate D. Acamprosate
D. Seizure disorder 38. Which of the following is not used in delirium?
31. A 40-year-old man presents to casualty with his¬ (PGIDec 2005)
tory of regular and heavy use of alcohol for ten A. Haloperidol B. Lithium
years and morningdrinking for one year. The last C. Diazepam D. Olanzapine
alcohol intake was three days back. There is no E. Risperidone
history of head injury or seizures. On examina¬
tion, there is no icterus, sign of hepatic encepha¬ Opioid
lopathy or focal neurological sign. The patient
39. Which of the following is not an opioid peptide?
had coarse tremors, visual hallucinations and
(AIIMS May 2005)
haddisorientation to time. Which of the following
is the best medicine to be prescribed for such a A. Endorphins B. Epinephrine
patient? (AI2004) C. Leu-enkephalins D. Met-enkephalins
A. Diazepam B. Haloperidol 40. All are seen in morphine poisoning except:
C. Imipramine D. Naltrexone (AI1997)
A. Cyanosis
32. A chronic alcoholic patient stopped alcohol intake B. Pinpoint pupil
for 2 days due to religious reasons, developed C. Hypertension
symptoms of withdrawal on first day. On second D. Respiratory depression
74 Review of Psychiatry

41. Opioids can cause which of the following? C. Prevent relapse


A. Physical dependence (DNB NEET2014-15) D. Has addiction potential; used for detoxification
B. Psychological dependence of opioid
C. Both A and B
D. None of the above CannsblS
42. Usual sign of morphine withdrawal are all except: 49. After use of some drug, a person develops episodes
(PGIMay 2013, 1999, 1993) of rage in which he runs about and indiscrimi¬
A. Dryness of secretion natelyinjures apersonwho is encountered inway.
B. Constipation He is probably addict of: (AIIMS 199 7)
C. Miosis A. Alcohol B. Cannabis
D. Lacrimation, diarrhea, rhinorrhea C. Opium D. Cocaine
E. Generally occur after 6-8 hours of last use
50. Which of the following substances is associated
43. Withdrawal of which of the following causes yawn¬ with flashback phenomenon? (KA 1999)
ing and piloerection ? (DNB NEET 2014-15) A. Cannabis
A. Morphine B. Cannabis B. LSD
C. Smoking D. Alcohol C. Psilocybin
D. All of the above
44. A boy ishavingdiarrhea, rhinorrhea, sweating and
lacrimation.What is the most probable diagnosis? .
5 1 Amotivational syndrome is seen in:
A. Cocaine withdrawal (AIIMS Nov 2010) (DNB NEET 2014-15, MH2010, TN 1999)
B. Heroin withdrawal A. Cannabis B. Cocaine
C. Alcohol withdrawal C. Amphetamine D. Heroin
D. LSD withdrawal
52. Which of the following substance intoxication
45. Treatment of opioid dependence includes: causes conjunctival congestion, increased appe¬
(PGIMay 2011) tite, dry mouth, tachycardia and synesthesia?
A. Naloxone B. Naltrexone (MH2009)
C. Acamprosate D. Buprenorphine A. Cannabis B. Caffeine
E. Topiramate C. Cocaine D. Codeine

46. Which drug is most commonly usedworldwide in 53. Bad trip is seen with which of the following drugs:
maintenance treatment for opioid dependence? (DNB NEET 2014-15)
(AT 2011) A. Cocaine B. Cannabis
A. Naltrexone B. Methadone C. LSD D. Heroin
C. Imipramine D. Disulfiram
Others
47. Which of the following is an alternative to
methadone for maintenance treatment of opiate 54. Correct statement about cocaine abuse:
dependence? (AIIMS May 2005) (PGIMay 2011)
A. Diazepam A. Block uptake of dopamine in CNS
B. Chlordiazepoxide B. Strong physical dependence
C. Buprenorphine C. Increased BP
D. Dextropropoxyphene D. Severe tolerance
48. Naltrexone is used in opioid addiction because: E. Cause impairment of nerve conduction
(AIIMS May 2010, 2007, 2006, AI2007) 55. Paranoid delusions are associated with use of:
A. To treat withdrawal symptoms (AI2012)
B. To treat overdose of opioids and prevent respira¬ A. Cocaine B. Heroine
tory depression C. Cannabis D. GHB
Substance Related and Addictive Disorders 75

56. Jet black pigmentation of tongue with tactile hal- ANSWERS


lucination and visual hallucinations is a feature
1. D. The personality, family history and peer pressure
of which substance use: (RJ 1998)
all play a role in development of dependence.
A. Cocaine B. Cannabis
There is no correlation between intelligence and
C. Heroin D. LSD
substance use.
57. Paranoid psychosis observed with cocaine abuse 2. A, C.
can be explained by: (AI 2011, 2012) The DSM-IV, diagnosis of substance abuse
A. Tolerance B. Intoxication includes the following four criterion (1) recurrent
C. Reverse tolerance D. Withdrawal use resulting in failure to fulfil major obligations
58. Formicationanddelusion of persecution,bothare at work, school or home. (2) recurrent use in

together seen in: (AIIMS May 2011, 2009) situations in which it is physically hazardous
A. LSD psychosis (such as while driving) (3) substance use causing
B. Amphetamine psychosis legal problems and (4) substance use causing
C. Cocaine psychosis social or interpersonal problems (e.g. fights
D. Cannabis psychosis with spouse). Withdrawal and tolerance are a
criterion for "substance dependence" but not
59. A 16-year-oldboy suffering from drug abuse pre¬ "substance abuse" Please remember in DSM-5,
sents with crossover of sensory perceptions, such both these diagnosis of "substance dependence"
that, sounds can be seen and colors can beheard. and "substance abuse" have been removed and
Which of the following is the most likely agents replaced by "substance use disorders"
responsible for drug abuse? (AI2012) 3. B. Neither presence of legal problems related to
A. Cocaine B. LSD substance use nor use of illegal substances, is a
C. Marijuana D. PCP(phencyclidine) criterion for substance dependence.
60. Psychosis resulting due to chronic amphetamine 4. A. Since cannabis causes very mild withdrawal
intake most commonly resembles: (Orissa 1999) symptoms hence, no symptomatic treatment
A. Delirium is required. LSD and other hallucinogens also
B. Mania do not cause any withdrawal symptoms or tole¬
C. Paranoid schizophrenia rance.
D. Dissociative disorder 5. D.
6. C. Dipsomania is compulsive drinking or an irresis¬
61. Used for averting tobacco dependence is:
tible urge to drink alcohol.
(DPG 2008)
7. A. Inalcoholic balackouts, which is an anterograde
A. Buspirone B. Methadone
amnesia, the person later doesn't remember,
C. Bupropion D. Buprenorphine
however at that time he appears to be totally
62. Most common substance of abuse in India is: in control and his behavior appears purpose¬
(DNB NEET 2014-15, AIIMS May 2010, May 2007, ful to others. He doesn't look confused to the
AI 2007) onlookers.
A. Tobacco B. Cannabis 8. B. Since this patient, has been taking alcohol
C. Alcohol D. Opium every day, at the time of presentation, disulfiram
63. Which is not a feature of nicotine withdrawal?
should be avoided as it may precipitate a severe
(DNB December 2011) disulfiram like reaction. Disulfiram should not be
A. Depression B. Headache
useduntil person has abstained from alcohol for
C. Tachycardia D. Anxiety atleast 12 hours. Also, please remember that phe-
nytoin doesn't have any role in the management
64. Which is not a feature of caffeine withdrawal? of alcohol dependence. However, this question
(DNB December 2011) is specifically asking for the drug that should be
A. Headache B. Hallucination avoided and hence disulfiram is the best answer.
C. Depression D. Weight gain 9. a. Alcohol does produce tolerance.
76 Review of Psychiatry

10. C. Tremors usually appear 6-8 hours after last The diagnosis in this patient is Wernicke's
alcohol intake. encephalopathy. The patients when treated
11. B. Tremor is the most common withdrawal symp¬ adequately have the following course (1) Oph¬
tom (excluding the hangover). thalmoplegia starts to resolve within hours,
12. A. Alcoholic hallucinosis is a characteristic with¬ though horizontal nystagmus often persists (2)
drawal symptom of alcohol. Delusion of infidelity Ataxia begins to improve within first week how¬
(morbidjealousy) is also seen in chronic alcoho¬ ever around 50% of patient will be left with some
lism but it is not related to withdrawal state. residual abnormalities. (3) Global confusion
13. C. begins to recover within 2-3 weeks and would
14. C. Reverse tolerance refers to the phenomenon usually clear completely in 1-2 months. Despite
where the intoxicating effects of alcohol are seen treatment, patient can progress to Korsakoff
progressively with lower dosages. syndrome.
15. A, B, C, E. 30. A. The onset of symptoms is after 2 days of last
See the list of alcohol induced disorders in the intake. There is history of chronic alcohol use.
text. There is history of disorientation (misrecogni-
16. D. Oculomotor nerve plays causing ophthalmople¬ tion), visual hallucination (seeing snakes and
gia is a feature of Wernicke's encephalopathy and reptiles), hyperactivity. All these put together is
not delirium tremens. suggestive of delirium tremens.
17. A, B, D, E. 31. A. The diagnosis is delirium tremens and the drug
Deliriumtremens is usually not a chronic condi¬ of choice is benzodiazepines like diazepam.
tion. 32. C. The diagnosis is alcohol withdrawal seizures
18. A, B. and the drug of choice is benzodiazepines like
Kindly see text. diazepam.
19. B. 33. B. Benzodiazepines are the drug of choice inalcohol
20. B. withdrawal. If the question asks you to chose a
21. C. specific benzodiazepine, the best choice would
22. C. Kindly see text. be chlordiazepoxide.
23. A. Here, there is history of ataxia (incoordination) 34. D. The best answer here is both diazepam and
and ophthalmoplegia. The inability to tell name chlordiazepoxide as the benzodiazepines are
might be because of confusional state. The likely the drugs of choice. However, please remember
diagnosis is Wernicke's encephalopathy. antipsychotics can also be used if patient is hav¬
24. A, B. ing excessive hallucinations or is excessively
25. D. Korsakoff syndrome is due to thiamine defi¬ agitated and these symptoms are not responding
ciency. Apart from alcoholism, malnutrition can to benzodiazepines alone.
also cause it. Also it presents with amnesia and 35. A. Flumazenilhas no role. It is used in benzodiaz¬
confabulations. epine overdose.
26. D. InKorsakoff psychosis, there is prominent antero¬ 36. A, B, C, E.
grade amnesia. Whenever there is anterograde Naltrexone is used in alcohol as well as opioid
amnesia (i.e. new memories cannot be made), dependence. Naloxone is used in opioid over¬
learning would be severely affected. dose. Clonidine can be used in opioid withdrawal
27. A. and disulfiram in alcohol dependence.
28. A, B, D. 37. A. See text.
There is some mistake inthe language of question 38. B. As explained above, benzodiazepines and anti¬
as only option C is correct and all other options psychotics can be used in delirium.
are wrong statement. In Korsakoff, both remote 39. B. Epinephrine is not an opioid peptide. The endo¬
memory and intellect remains preserved and the genous opioid peptides include p endorphins,
patient doesn't have insight into his symptoms. Met and Leu enkephalins and Dynorphins.
29. A, B, C. 40. C. Hypotension is a feature and not hypertension.
Substance Related and Addictive Disorders 77

41. C. and auditory hallucinations and it resembles


42. A, B, C. paranoid schizophrenia.
43. A. 61. C.
44. B. 62. C. This is a controversial question. Now, if the ques¬
45. B, D. tion was simply, most commonly used substance
46. B. Methadone is used as methadone maintenance in India, the answer would have been tobacco
treatment, in long-term treatment of opioid without any controversy. Since the question
dependence. mentions most common "substance abuse" the
47. C. Methadone, buprenorphine, levo alpha acetyl- controversy arises. According to DSM-IV, there
methadol can be usedfor maintenance treatment can be two types of substance use disorders (1)
of opiate dependence. substance dependence (2) substance abuse.
48. C. The only indication for naltrexone in opioid These two can be considered as two different
dependence is relapse prevention in highly levels of addiction, substance abuse is a lower
motivated patients. For opioid overdose nalox¬ level and substance dependence is higher level.
one is used and not naltrexone. Now, tobacco can cause dependence but not
49. B. The description is suggestive of run amok which abuse i.e. DSM-IV provides criterion for tobacco
is seen with cannabis use. dependence but says that "substance abuse" is
50. D. Cannabis and hallucinogens cancause flash back not applicable for tobacco. Whereas for alco¬
phenomenon. hol which is the second most commonly used
51. A. substance in India, both "alcohol dependence"
52. A. and "alcohol abuse" has been described.If this
53. B and C. question is interpreted strictly in terms of DSM-
54. A, C and E. IV diagnoses, of substance abuse, the answer
Cocaine causes strong psychological dependence becomes alcohol, as there is no diagnosis of
however physiological dependence (tolerance "tobacco abuse" However if the term abuse is
and withdrawal symptoms) is mild in compari¬ used literally, the answer becomes tobacco.
son. Cocaine blocks dopamine and norepineph¬ The book published by AIIMS, on substance
rine uptake and hence causes hypertension. It use disorders, says that "alcohol is the most
blocks nerve conduction and is also used as an frequently used substance as seen in the NHS
anesthetic agent. and DAMS! Here "NHS" and "DAMS" refers to
55. A. Cocaine. name of surveys which were conducted by Indian
56. A. government. Further, a table titled "major drugs
57. B. The delusion of persecution and auditory hal¬ of abuse inIndia" again mentions alcohol on the
lucinations can be seen in cocaine intoxication. top and doesn't mention anything about tobacco.
58. C. Since, this question has been asked by AIIMS, its
59. B. The sign here is synesthesia (sounds can be seen likely that they will follow their own book. So, my
and colors can be heard) which is in with LSD advise is mark alcohol as the answer.
and cannabis intoxication. 63. C. Bradycardia is a symptom of nicotine withdrawal
60. C. The symptoms of amphetamine induced psy¬ and not tachycardia.
chotic disorder include delusion of persecution 64. B. See text.
Chapter

0 Organic Mental Disorders

Organic mental disorders are caused by either a demon¬ eyes closed. Various other terms such as "confusional
strable cerebral disease, brain injury or other insults state" "clouding of consciousness" and "altered sen-
leading to cerebral dysfunction. Following are the com¬ sorium" are used to describe the disturbances of con¬
mon symptoms seen in organic mental disorders: sciousness in delirium.
A. Cognitive impairment: The term "cognition" is used C. Hallucinations : These patients most commonly have
to describe all the mental processes that are utilized visual hallucinations9 although auditory, olfactory,
to gain knowledge. These processes include memory, gustatory and tactile hallucinations can also be pre¬
language, orientation, judgment, performing actions sent.
(praxis) and problem solving. At times the term "cog¬ D. Delusions : The delusions are usually transient9.
nition" is used to describe the thoughts. In organic Complex delusions are rare9.
mental disorders one or more of cognitive functions The organic mental disorders are classified in the fol¬
are impaired. Frequendy patient presents with diso¬ lowing groups:
rientation (to time, place and person), impaired A. Delirium
attention and concentration, disturbances in memory B. Dementia
(especially recent memory resulting in anterograde C. Amnestic disorders
amnesia), etc. As organic mental disorders commonly
have disturbances of cognition, they are also known DELIRIUM
as cognitive disorders.
_ _
It is the most common9 organic mental disorder. It is
B. Disturbances of consciousness: The consciousness
characterized by an acute onset9 of symptoms and a
has different levels ranging from alertness to coma.
fluctuating course9. It is most commonly seen in elderly
Usually the term "alertness" is used when one is aware
population. The patients who have been hospitalized
of the internal and external stimuli and can respond for medical and
surgical disorders frequendy develop
to them. The patients with organic mental disorders delirium. The
patients with hip fractures9, open heart
usually have disturbances of consciousness which surgeries9, severe burns9, pneumonia9,
postoperative
can be of varying severity. The term "somnolence or patients9 and critically ill patients have high
prevalence
lethargy" is used when patient tends to drift off to of delirium. The history of a medical disorder followed by
sleep when not actively stimulated. The next level is sudden development of disturbances of consciousness,
"obtundation" in which patient is difficult to arouse cognition and psychiatric symptoms such as hallucina¬
and when aroused appears confused. The next level tions and delusions is strongly suggestive of delirium. The
is "stupor or semicoma" in which patient is mute and other causes includes use of multiple medications (espe¬
immobile. When stimulated persistently and vigor¬ cially those with anticholinergic actions). Withdrawal of
ously he may groan or mumble. Finally, in "coma" , psychoactive substances (such as alcohol and sedadves/
patient is totally unarousable and remain with their hypnotics) is another common cause. Delirium can
Organic Mental Disorders 79 1
develop in older patients wearing eye patches after cata¬ Delirium versus dementia: The acute presentation and fluc¬
ract surgery (due to sensory deprivation), also known as tuations of symptoms is suggestive of delirium. Dementia
black-patch deliriumQ. develops slowly and usually the symptoms are stable over
time. Further, a patient with delirium presents with distur¬
Symptoms bances of consciousness whereas a patient with demen¬
tia doesn't have any consciousness disturbances. In some
The clinical features of delirium are: cases, a patient of dementia may develop superimposed
• Disturbances of consciousness0 (ranging from som¬ delirium, a condition called as "beclouded dementia'.'
nolence to coma)
Delirium versus schizophrenia: A patient of delirium
• Impairment of attention
may have pronounced hallucinations and delusion and
• Disorientation to time, place and person
may resemble schizophrenia. However, in delirium the hal¬
• Memory disturbances (impairment of immediate
and recent memory with relatively intact remote lucinations are not constant and delusions are transient
memory0) and not systematized (not organized) whereas in schizo¬
phrenia the hallucination are more constant and delusions
• Perceptual disturbances like illusions and hallucina¬
tions (most commonly visual0) and transient delu¬ are also better organized. Further, the patient of delirium
sions has disturbances of attention and disturbed consciousness
which is not seen in patient with schizophrenia.
• Hyperactivity or hypoactivity, agitation
• Autonomic disturbances
• Disturbances of sleep wake cycle (insomnia or rever¬ Treatment
sal of sleep wake cycle) A. Treat the underlying cause.
• Sundowning: It refers to diurnal variation of symp¬ B. Antipsychotics can be used for management of delu¬
toms with worsening of symptoms in the evening (i.e. sions, hallucinations and agitation seen in delirium.
with downing of sun) C. Benzodiazepines are used for insomnia and are the
• Floccillations (or carphologia): Aimless picking beha¬ drugs of choice in alcohol withdrawal delirium (delir¬
vior, where patient appears to be picking at his ium tremens).
clothes/bed
• Occupational delirium: Patient behaves as if he is still
on his job, despite being in hospital (e.g. a tailor may
DEMENTIA __
ask for clothes and scissors, while lying on the bed of" Dementia is defined as a
the hospital). progressive impairment DSM-5 Update: The DSM-4 diagno¬
sis of dementia and amnestic disor¬
The neurotransmitter involved in delirium is acetyl¬ of cognitive functions in der are sub-sumed under the newly
choline and the neuroanatomical area involved is the the absence of any dis¬ named entity major neurocognitive
disorders (NCD).
reticular formation (kindly remember reticular ascend¬ turbances of conscious¬
ing system is responsible for arousal in a person). ness0. The prevalence of
dementia increases with DSM-5 Update: In DSM-5, a new
age, with prevalence of diagnostic category of mild neuro¬
Diagnosis cognitive disorders (NCD) has been
around 5% in the popu¬ added, for the patients who present
The diagnosis of delirium is made clinically0, on the basis lation older than 65 with milder cognitive impairment
(which is not sever enough of diag¬
of above mentioned symptoms. The sudden onset and
years and prevalence of nosis of dementia or major neurocog¬
fluctuations in symptoms are important pointers towards nitive disorder).
20-40% in the popula¬
the diagnosis. Bedside examinations such as mini mental
tion older than 85 years. The underlying cause of dementia
status examination (MMSE)° and mental status exami¬
can be permanent or reversible.
nation (MSE) are used to provide a measure of cognitive
impairment.
Generalized slowing0 on EEG is a common finding Symptoms
in patients with delirium, however delirium caused by The following are the symptoms of dementia:
alcohol or sedative-hypnotic withdrawal has low voltage A. Cognitive impairment The cognitive impairment is charac¬
fast activity on EEG. terized by 4 A's: amnesia, aphasia, apraxia and agnosia.
80 Review of Psychiatry

• Amnesia refers to the memory impairment. may result in an emotional outburst in a patient
Initially the loss is of recent memory followed by of dementia. This is known as "catastrophic
immediate memory and lastly the remote mem¬ reaction"5.
ory. Another way of describing memory impair¬ C. Focal neurological signs and symptoms: These are usu¬
ment is in terms of episodic (memory for events), ally seen in vascular dementia (multi-infarct demen¬
semantic memory (memory for facts such as rules, tia) and correspond to the site of vascular insults.
words and language) and visuospatial deficits. In These include exaggerated tendon reflexes, extensor
episodic memory, there is a gradient of loss with plantar response, gait abnormalities, etc.
more recent events being lost before remote events.
Semantic memory is preserved in the early course Types
of disease and is gradually lost as the disease pro¬
The dementia can be divided in to reversible and irrever¬
gresses. Visuospatial skills deficits manifests with
sible dementias. It is extremely important to do detailed
symptoms of disorientation in strange environ¬
ments and later, wandering and getting lost in even
work up of a patient of dementia as around 15% of cases
familiar environments. are reversible. The reversible causes of dementia5 are:
Aphasia refers to the disturbances of language A. Neurosurgical conditions (subdural hematoma, nor¬

function. The initial disturbance is usually "word- mal pressure hydrocephalus, intracranial tumors,
finding difficulties" which gradually progresses to intracranial abscess).
more severe abnormalities. B. Infectious causes (meningitis, encephalitis, neuro¬
• Apraxia is inability to perform learned motor syphilis, lyme disease).
functions. For example, patient may start having C. Metabolic causes (vitamin B12 or folate deficiency,
difficulties in functions like buttoning the shirt or niacin deficiency, hypo and hyperthyroidism, hypo
combing the hair. and hyperparathyroidism).
• Agnosia is inability to interpret a sensory D. Others (drugs and toxins, alcohol abuse, autoimmune
stimulus. One of the common disturbance is encephalitis).
"prosopagnosia"5 which is inability to identify the Dementia can also be classified into cortical and sub¬
face. At times patient may be unable to identify his cortical types depending on the area of brain which is
own face , a condition known as "autoprosopag- affected first by the dementing process.
nosia".
Cortical dementias: These disorders are characterized by
• Apart from the 4 A's, disturbances in executive early involvement of cortical structures and hence early
functioning (i.e. planning, organizing, sequencing
and abstracting) is another important cognitive appearance of cortical dysfunction. These disorders have
impairment. early and severe presentation of the As: amnesia, apraxia,
B . Behavioral and psychological symptoms: These may aphasia, agnosia and acalculia (impaired mathematical
include: skills) indicating cortical involvement. Alzheimer's
• Personality changes: There might be a significant disease5 is the prototype of cortical dementia. Others
change in the personality. Patient may become include Creutzfeldt-Jakob disease, Pick's disease and
introvert and seem to be unconcerned about other frontotemporal dementias.
others or patients may become hostile. The per¬
Subcortical dementia: These disorders are characterized
sonality changes are mostly seen in patients with
by early involvement of subcortical structures like basal
frontal and temporal lobe involvement.
ganglia, brain stem nuclei and cerebellum. These dis¬
• Hallucinations and delusions: Delusion mosdy seen
is delusion of persecution and delusion of theft. orders are characterized by early presentation of motor
symptoms (abnormal movements like tics, chorea, dysar¬
• Depression, manic and anxiety symptoms.
• Apathy, agitation, aggression, wandering and circa- thria, etc), significant disturbances of executive functioning
dian rhythm disturbances. and prominent behavioral and psychological symptoms
• Catastrophic reaction: The subjective awareness of like apathy, depression, bradyphrenia (slowness of think¬
intellectual deficits while in a stressful situation ing). The examples include Parkinson's disease, Wilson's
Organic Mental Disorders 81 1
disease, Huntington's disease, multiple sclerosis, progres¬ in tangles is in a highly phosphorylated form and has
sive supra nuclear palsy, normal pressure hydrocephalus. abnormal functioning. Normally, tau protein binds
Some dementias such as vascular dementia, dementia and stabilizes microtubules, which are essential for
with lewy body have mixed presentation. axonal transport, however in Alzheimer's this func¬
tion is deranged. The neurofibrillary tangles are widely
Alzheimer's Disease (Dementia of distributed in cortical structures and hippocampus,
Alzheimer's Type) but always spare cerebellum*3. Multiple studies have
established that amount and distribution of NFTs
It is the most common13 cause of dementia. The preva¬ correlates with the duration and severity of dementia*3.
lence of Alzheimers disease increases with age, the rates Both senile plaques and neurofibrillary tangles can
are around 5% for all those aged 65 years and older, be present in elderlies without any dementia. However
increasing to around 20-30% for all those aged above 85 in patients with dementia, these findings are extensive
years. The Alzheimers disease can be divided into early and wide spread. The neuropathological diagnosis of
onset (presenile), if the age of onset is 65 years or ear¬ Alzheimer disease requires extensive presence of both
lier; or late onset (senile), if the age of onset is after 65 senile plaques (extracellular deposits) and neurofibrillary
years. At all ages, females outnumber males by a ratio of tangles (intracellular inclusions).
2 or 3:1 except in early onset familial forms (inherited as Granulovacuolar degeneration (GVD)*3 and Hirano
autosomal dominant disorder) in which sex ratio is 1. The bodies*3 (eosinophilic inclusions) are abnormalities seen
onset is usually insidious and progression is gradual. The in the cytoplasm of hippocampal neurons in patients with
insightQ (awareness of illness) is lost relatively early in the Alzheimer disease. Both of them are present in elderlies
course of illness. In the initial phase symptoms include
without dementia, however they are much more severe
and widespread in Alzheimers disease.
memory disturbances, gradually apraxia, agnosia, apha¬
sia and acalculia develop and executive functions are lost. Amyloid cascade hypothesis: According to this hypo¬
In the later stages neurological disabilities like tremors, thesis, mutation in APP gene near cleavage site favor the
rigidity and spasticity may develop. cleavage by [3 and y secretase, resulting in the produc¬
tion of Ap. The Ap peptides form Ap oligomers which in
Pathophysiology: The classical gross neuroanatomical turn induce tau phosphorylation, producing neurofibril¬
finding in Alzheimers disease is diffuse atrophy with flat¬ lary tangles. The tau protein in this highly phosphorylated
tened cortical sulci and enlarged cerebral ventricles. form is not able to stabilize microtubules, resulting in
The classical microscopic findings are neuritic granulovascular degeneration of neurons, neuronal loss
(senile) plaquesQ and neurofibrillary tanglesQ. Senile and synaptic loss.
plaques, also referred to as amyloid plaques are com¬ Neurochemistry: Alzheimer's disease is predominantly a
posed of a particular protein Ap. This protein is derived disorder of cholinergic neurons'3 and loss of cholinergic
from amyloid precursor protein (APP) by the action of neurons in nucleus basalis of meynert is a consistent find¬
|3- and y-secretase enzymes. The AP protein combines to ing. Apart from acetylcholine, norepinephrine and sero¬
form fibrils. The senile plaques are extracellular deposits tonin have also been implicated in some cases.
of Ap and are found in all cortical areas and also in striatum
Genetics: Alzheimer's disease has shown linkage to
and cerebellum. The amyloid-P peptide not only deposits chromosome 1,14 and 21. A small number of cases of
in the brain parenchyma in the form of amyloid plaques Alzheimer disease are early onset and familial and are
but also in the vessel walls in the form of cerebral amy¬ inherited in autosomal dominant fashion. Mutations
loid angiopathy (CAA)Q. in three genes, amyloid precursor protein*3 (chromo¬
The senile plaques can also be seen in elderlies who some 21), presenilin-lQ (chromosome 14) andpresenilin-2*3
do not have Alzheimer's and their number increases with (chromosome 1) have been found in most cases with
age. Hence senile plaques are not specific for Alzheimer familial Alzheimer's disease. The majority of cases
disease. The amyloid plaques are not correlated with the are however sporadic and late onset. ApO E4 gene*3 is
severity of dementia. associated with the risk of development of Alzheimers
The neurofibrillary tangles (NFTs) are intraneu- disease, however its testing is not recommended as it is
ronal aggregates of tau protein. The tau protein present neither sensitive nor specific for Alzheimer's disease.
I 82 Review of Psychiatry

The patients with Down's syndromeQ have signifi¬ subcortical type with more motor abnormalities and less
cantly higher risk for development of Alzheimer's disease. of amnesia, apraxia, aphasia and agnosia.
The gene for APP (amyloid precursor protein) is located
on chromosome 21. HIV Related Dementia
Risk factors: Age is the most important risk factors. The diagnosis of HIV dementia (AIDS dementia complex)
Other risk factors include head injury, hypertension, is made by lab evidence of systemic HIV infection, cogni¬
insulin resistance, depression. Few studies have claimed tive deficits, presence of motor abnormalities or persona¬
that smoking0 is a protective factor against Alzheimer's lity changes. Personality changes are characterized by
but this finding has been contradicted by other studies. apathy, emotional lability or disinhibition.
High education levels and remaining physically and men¬
tally active till late in life are protective factors against Head Trauma Related Dementia
Alzheimer's disease. Dementia can develop as a sequelae of head trauma.
Dementia pugilistica (punch drunk syndrome) can
Vascular Dementia or Multi-infarct Dementia develop in boxers after repeated head trauma.
This is the second most common type of dementia.
Occurrence of multiple cerebral infarctions (caused by Frontotemporal Dementia (FTD)
occlusion of cerebral vessels by arteriosclerotic plaques Frontotemporal dementias are a group which have simi¬
or thromboemboli) results in progressive deterioration of lar presentation but may be caused by a variety of neuro-
brain functions, finally resulting in dementia. There are pathological substrates. Pick's disease0 is one pathological
acute exacerbations which correspond to the new infarcts, variant of FTD, and is characterized by presence of pick's
and result is stepwise deterioration of symptoms (step- bodies. The frontotemporal dementia's have an earlier
ladder pattern). The general symptoms of dementia onset0, around 45-65 years and mainly present with beha¬
are present. In addition patient has focal neurological vioral symptoms and change in personality with relative
deficits which correspond to site of infarction. There is preservation of memory. Three distinctive forms of FTD
usually history of previous stroke or transient ischemic have been described on the basis of clinical presentation.
attacks. The patients usually have hypertension and A. Frontal variant FTD: The symptoms are primarily of
other cardiovascular risk factors. The treatment involves loss of frontal lobe function. The classical feature is
management of risk factors and cholinesterase inhibitors. stereotyped behavior, disinhibition and apathy.
B. Semantic dementia: The symptoms are primarily of
Binswanger's diseaseQ: It is also known as subcortical
loss of temporal lobe functions and is characterized
arteriosclerotic encephalopathy, and is characterized by
by complaints of loss of memory for words.
multiple small white matter infarctions and can produce
C. Progressive nonfluent aphasia: It presents with
symptoms of subcortical dementia.
speech dysfluency and word finding difficulties.

Lewy Body Disease (Dementia with Lewy Body) Pseudodementia


The clinical signs and symptoms are similar to Alzheimer The depression in elderly patients may mimic symptoms
disease. Apart these patients also have fluctuating levels of dementia and hence is known as pseudodementia0.
of attention and alertness, recurrent visual hallucinations A depressed patient may get a low score on MMSE, as
and parkinsonian features (tremors, rigidity and bradyki- depressed individual lacks motivation to solve the ques¬
nesia). Antipsychotic medications should be avoided as tions. Hence low score on MMSE should be carefully
these patients are extremely sensitive to antipsychotics interpreted, if depression is suspected.
and can develop drug induced parkinsonism.
Management of Dementia
Huntington's Disease, Parkinson's Disease,
The evaluation of cognitive functions is usually done using
Wilson's Disease and Multiple Sclerosis the screening test of mini mental status examination
These predominantly motor diseases are associated with (MMSE)0. A score of less than 24 (out of a maximum 30) is
the development of dementia. The dementia seen is of suggestive of dementia. In accordance with the cholinergic
Organic Mental Disorders 83 1
hypothesis, cholinesterase inhibitors are widely used Amnestic syndrome is characterized by inability to form
for treatment of cognitive deficits in Alzheimer's disease. new memories (anterograde amnesia) and the inability
Donepezil, rivastigmine, galantamine and tacrine are few to recall previously remembered knowledge (retrograde
of the drugs belonging to this category. amnesia). Short-term and recent memory are usually
Memantine, a NMDA receptor antagonist has also impaired with preservation of remote and immediate
been approved for the treatment. For behavioral and memory. The major causes'3 of amnestic disorders are:
psychological symptoms of dementia, symptomatic treat¬ A. Thiamine deficiency (Korsakoff syndrome)
ment is used and may include antidepressants, antipsy¬
B. Hypoglycemia
chotics and benzodiazepines. C. Primary brain conditions (head trauma, seizures, cere¬
bral tumors, cerebrovascular disease, hypoxia, elec¬
AMNESTIC DISORDERS _ troconvulsive therapy, multiple sclerosis)
Amnestic disorder is a broad category that includes a vari¬ D. Substance related disorders (alcohol, benzodiaz¬
ety of conditions which present with amnestic syndrome. epines).

QUESTIONS AND ANSWERS

QUESTIONS_ 5. Disorientation occurs in: (AI 1993)


A. Schizophrenia
Organic Mental Disorders B. Organic brain syndrome
C. Depression
1. Which of the following behavioral problems
D. Mania
would suggest an organic brain lesion?
(SGPGI2005, DNB2006) 6. Which of the following suggest a psychotic rather
A. Formal thought disorder than an organic disorder? (DNB June 2009)
B. Auditory hallucinations A. Confusion
C. Visual hallucinations B. Complex delusions
D. Depression C. Impairment of consciousness
D. Lack of insight
2. Organic mental disease is indicated by:
(AIIMS 1991, DNB 1993) 7. Feature(s) suggestive of schizophrenia rather than
A. Incoherence organic psychosis is/are: (PGIJune 2009)
B. Delusion A. Third person hallucination
C. Flight of idea B. Split personality
D. Perseveration of speech C. Visual hallucination
D. Altered sensorium
3. Mini mental status examination is:
E. Systematized delusion
(DNB 2004, JIPMER 2002)
A. Method to investigate common psychiatric 8. InIndia psychiatric disorder in people above 60-
problem year of age is mostly due to:
B. 30 point program to evaluate cognitive functions (DNB 2003, Calcutta 2K)
C. To evaluate schizophrenia A. Depression B. Dementia
D. Instrument to measure delirium C. Hysteria D. Schizophrenia

4. Cognitive disorders are: (PGIJune 2006, 2007)


Delirium
A. Intellectualization B. Depersonalization
C. Dementia D. Delirium 9. Most important feature of delirium is:
E. Hallucination F. Secondary gain (DNB NEET 2014-15)
84 Review of Psychiatry

A. Impaired attention Amnestic Syndrome


B. Anxiety 16. Anterograde amnesia is seen in:(AIIMS Nov 2010)
C. Hyperactivity A. Head injury
D. Clouding of consciousness B. Stroke
10. Delirium is defined as: (DNB NEET 2014-15) C. Spinal cord injury (traumatic paraplegia)
A. Acute onset of disturbed consciousness D. Alzheimer's disease
B. Chronic onset of disturbed consciousness
17. Cause of organic amnestic syndrome include(s):
C. Progressive generalized impairment of intellec¬
(PGIMay 2013)
tual functions and memory without impairment
A. Multiple sclerosis B. Hypoglycemia
of consciousness.
C. Hyperglycemia D. Hypoxia
D. Disorientation without clouding of conscious¬
ness E. Hypercapnia
18. Not diagnostic/defining criteria for amnestic dis¬
11. Features of delirium: (PGINov 2010, June 2008)
order: (PGINov 2009)
A. Deficit of attention (attention deficit)
B. Autonomic instability (dysfunction) A. Visual hallucination
C. Altered sleep wake pattern B. Transient delusion
D. Visual hallucination and clouding of conscious¬ C. Impaired concentration/attention
ness D. Good recall of recent events
E. Delirium cannot be diagnosed clinically E. Ability to form new memories

12. Deliriumand schizophrenia differ from each other 19. All are true except: (PGIFeb 2008)
by: (DNB 2003, WB 2001, KA 2004) A. Procedural learning is from past experiences
A. Change in mood B. Implicit learningis procedural skill acquirement
B. Clouding of consciousness C. Amnestic syndromes lose semantic memory
C. Tangential thinking D. Implicit memory is declarative
D. All of the above E. Anterograde amnesia affects long-term memory
more in amnestic syndrome
13. Slow waves in EEG activity are seen m:(PGI 1998)
A. Depression B. Delirium Dementia
C. Schizophrenia D. Mania
20. Deliriumand dementia can be differentiated by?
14. A patient with pneumonia for 5 days is admitted (DNB June 2010)
to the hospital inalteredsensorium. He suddenly A. Loss of memory B. Apraxia
ceases to recognize the doctor and staff. He thinks C. Delusion D. Altered sensorium
that he is injail and complains of scorpion attack¬
21. Most common cause of dementia is:
inghim. His probable diagnosis is: (AI2001)
A. Acute dementia B. Acute delirium (DNB NEET2014-15)
A. Alzheimer's disease B. Vascular dementia
C. Acute schizophrenia D. Acute paranoia
C. Wilson's disease D. Pick's disease
15. A 60-year man had undergone cardiac bypass
22. True about dementia is all except: (AI1994)
surgery 2 days back. Now he started forgetting
A. Often irreversible
things andwas not able to recall names and phone
B. Hallucinations are not common
numbers of his relatives. What is the probable
diagnosis? C. Clouding of consciousness is common
(AI2010)
A. Depression D. Nootropics have limited role
B. Post-traumatic psychosis 23. Catastrophic reaction is a feature of: (MH2011)
C. Cognitive dysfunction A. Dementia B. Delirium
D. Alzheimer's disease C. Schizophrenia D. Anxiety
Organic Mental Disorders 85

24. All are causes of subcortical dementia except: A. Down's syndrome


(AIIMS May 2009) B. Head trauma
A. Alzheimer's disease C. Smoking
B. Parkinson's disease D. Low education group
C. Supranuclear palsy
33. Dementia of Alzheimer's type is not associated
D. HIV associated dementia
with one of the following: (AIIMS Nov 2005)
25. Dementia is/are present in all except: A. Depressive symptoms
A. Alzheimer's disease B. Pick's disease B. Delusions
C. Lewybody D. Binswanger'sdisease C. Apraxia and aphasia
E. Gansers syndrome D. Cerebral infarcts
26. Reversible causes of dementia: (PGIJune 2004) 34. All the following are features of Alzheimer's dis¬
A. Hypothyroidism ease except: (DNB 1994, WB 2002)
B. Alzheimer's disease A. Cerebellar atrophy
C. Vitamin B12 deficiency B. Common in 5th and 6th decade
D. Vitamin A deficiency C. Atrophied gyri widened sulci
27. Treatable causes of dementia are: (PGI2001) D. Progressive dementia
A. Alzheimer's disease 35. InAlzheimer's disease (AD) which of the following
B. Hypothyroidism is not seen: (AIIMS Nov 2011)
C. Multi-infarct dementia A. Aphasia B. Acalculia
D. subdural hematoma (SDH) C. Agnosia D. Apraxia
E. Hydrocephalus
36. False regarding Alzheimer's disease (AD) is:
28. Vascular dementia is characterized by:(PGI2003) A. Number of senile neural plaques correlates
A. Disorientation B. Memory deficit (increases) with age
C. Emotional lability D. Visual hallucination B. Presence of tau protein suggest neurodegenera-
E. Personality deterioration tion
29. A 65-year-old male is brought to the outpatient C. Number of neurofibrillary tangles is associated
clinic with one year illness characterized by with the severity of dementia
marked forgetfulness, visual hallucinations, sus¬ D. Extracellular inclusions (lesions) can occur in
piciousness, personality decline, poor self care the absence of intracellular inclusions to make
and progressive deterioration inhis condition. His pathological diagnosis of AD
MiniMental Status Examination (MMSE) score is 37. Area of brain resistant to neurofibrillary tangles
21. His most likely diagnosis is: (AIIMS Nov 2002) inAlzheimer's disease: (AI2012)
A. Dementia B. Schizophrenia A. Visual association area
C. Mania D. Depression B. Entorhinal cortex
30. Which of the following neurotransmitters are C. Lateral geniculate body
decreased inAlzheimer's disease? D. Cuneal gyrus area Vl/temporal lobe
(DNBNEET 2014-15)
38. Regarding Alzheimer's disease which is/are not
A. Acetylcholine B. Norepinephrine
true: (PGIDec 2008, June 2009) (AIIMS Nov 2011)
C. Corticotropin D. All of the above
A. Initial loss of long-term memory
31. Protein involved in Alzheimer's disease: B. Delayed loss of short-term memory
(NIMHANS 2001, DNB 2002) C. Step ladder pattern
A. APOE4gene B. Presenilin-1 D. Cognitive impairment
C. Amyloid protein D. All of the above E. ludgment impaired
32. Following are predispositions to Alzheimer's dis¬ 39. All are true regardingAlzheimer's disease except:
ease except: (DNB 1996, AI 1999) A. Gradually progressive (PGIFeb 2008)
86 Review of Psychiatry

B. Abrupt onset and acute exacerbations C. Dementia


C. Episodic memory can be affected D. Anxiety
D. Frontotemporal disorder 47. The psychiatric disorder most commonly associ¬
E. Ubiquitin Lewy bodies
ated with myxedema:
40. Frontotemporal dementias include all except: A. Depression B. Mania
(DNB2003, UP 2007) C. Phobia D. Psychosis
A. Pick's disease
48. Myxedemamadness includes:(DAffiNEET2014-15)
B, Nonfluent aphasia
A. Auditory hallucinations and paranoia
C. Semantic dementia
B. Visual hallucinations and depression
D. Alzheimer's disease
C. Auditory hallucinations and depression
41. A 70-year-old man presents with h/o prosopag¬ D. Paranoia and depression
nosia, loss of memory, 3rd person hallucinations
since 1 month. On examination deep tendon ANSWERS
reflexes are increased, mini-mental examination
score is 20/30. What is most likely diagnosis? 1. C. If a patient presents with prominent visual hal¬
(AIIMS 2001) lucinations, organic mental disorders (organic
A. Dissociated dementia brain lesions) should always be looked for.
B. Schizophrenia 2. D. Perseveration of speech is suggestive of organic
C. Alzheimer's disease mental disorders. Few books are giving the
D. Psychotic disorder answer as delusion which is completely wrong.
3. B. Mini mental status examination is used to evalu¬
42. Not a feature of Alzheimer's disease:
ate cognitive functions in illnesses like dementia
(PGIMay 2013)
and delirium.
A. Hirano bodies
4. C, D.
B. Amyloid angiopathy
As organic mental disorders commonly have
C. Granulovacuolar degeneration of neurons
disturbances of cognition, they are also known
D. Senile plaque
as cognitive disorders.
E. Cerebellar atrophy
5. B. Presence of disturbances of consciousness and
43. Rivastigmine and Donepezil are drugs used pre¬ disorientation is suggestive of organic mental
dominantly in the management of: (A12006) disorders.
A. Depression B. Dissociation 6. B. The complex delusions are frequently seen in
C. Delusion D. Dementia psychotic disorder. In organic mental disor¬
44. True regarding FTD are all except: ders, the delusions are usually transient and
(AIIMS 2011, NEET 2013) fragmented. Presence of complex delusions in
A. Semantic dementia organic mental disorder is very rare. The lack of
B. Nonfluent aphasia insight is a feature of bothwhereas confusion and
C. Apathetic, disinhibited personality impairment of consciousness is seen in organic
D. Rapid onset static course mental disorders.
7. A, E.
45. All are true regarding frontotemporal dementia:
Third person hallucinations are quite suggestive
(AIIMS Nov 2012)
of schizophrenia. Also systematized delusions
A. Stereotypic behavior B. Insight present
(elaborate delusions) are much more likely in
C. Age less than 65 years D. Affective symptoms
schizophrenia. Please remember that schizo¬
46. The following are the psychiatric sequelae after phrenia is not a disorder of personalty and hence
stroke inelderly: (PGI2003) there is no "split personality" in schizophrenia.
A, Depression Visual hallucinations and altered sensorium
B. Post-traumatic stress disorder are more suggestive of organic mental disorders
Organic Mental Disorders 87 ]
although visual hallucinations can also be seen whereas implicit memory (nondeclarative
in schizophrenia. memory) doesn't involve awareness. For exam¬
8. B. In older age (>60 years) dementia is the most ple, if you have to chose the correct option for a
common psychiatric disorder followed by particular MCQ, you first try to remember the
depression. correct answer, i.e you try to bring the memory
9. D. Please remember that the hallmark symptom of associated with MCQ into awareness , hence its
delirium is clouding of consciousness, which is an example of explicit memory. However, when
associated with impairment of global cognitive you drive a car, you don't really try to remember
functions, most importantly attention. everything every time. Changing clutches, press¬
10. A. ing breaks and accelerator happens automati¬
11. A, B, C, D.
cally and you don't have to remember anything,
12. B. Delirium presents with clouding of conscious¬
its an example of implicit memory.
ness whereas in schizophrenia consciousness is
Explicit memory is further divided into epi¬
intact. The mood changes and tangential think¬
sodic memory for events (e.g. the memory of
ing cannot be used for differentiation.
your first day in medical college) and semantic
13. B.
memory for facts (e.g. memory for the most
14. B. History of a medical disorder (pneumonia ) fol¬
lowed by disturbances in consciousness (altered common , least common type of questions).
sensorium), disorientation (failure to recognize Procedural memory (for procedures like driv¬
doctor and staff and thinking that he is injail) and ing) is a type of implicit memory. Now, looking
hallucinations (scorpions attacking) is suggestive at options. Option A is true, procedural learning
of delirium. depends on past experience. Initially we have to
15. C. The history of cardiac surgery 2 days prior fol¬ remember every detail about how to use clutch,
lowed by behavioral changes is suggestive of break and accelerator however with repeated
delirium. The question here is stressing on experience it becomes implicit. Option B is also
"disturbances of memory" which can be seen correct as procedure learning is a type of impli¬
in delirium, however are usually restricted to cit memory. Option C is wrong, in amnestic
short- term memory loss. The other important syndrome, episodic memory is lost more and
features such as clouding of consciousness and not the semantic memory. Option D is wrong as
attention impairment has not been provided. implicit memory is nondeclarative. Option E is
Nonetheless, the most likely diagnosis appears to also wrong, in amnestic syndrome short-term
be delirium. As delirium has prominent cognitive and recent memory are more affected and not
dysfunction, that is the correct answer. Alzheimer the long-term memory.
disease does not have such sudden onset. 20. D. Please remember that the hallmark of delirium
16. B. Anterograde amnesia is seen in stroke.
is disturbance of consciousness (altered senso¬
17. A, B,D.
rium) whereas in dementia, there is no distur¬
See text.
bance of consciousness.
18. A, B, C, D, E.
21. A.
None of the options are included in diagnostic
22. C. There is no disturbance of consciousness in
criterion for amnestic disorder. Amnestic syn¬
drome is characterized by inability to form new dementia. It is often irreversible. The halluci¬
memories (anterograde amnesia) and the inabi¬ nations can be present but are not common.
lity to recall previously remembered knowledge Nootropics (or cognitive enhancers) have very
(retrograde amnesia). Short-term and recent limited role in the management of dementia.
memory are usually impaired with preservation 23. A. See text.
of remote and immediate memory. 24. A. Alzheimer's disease is a cortical dementia.
19. C, D, E. 25. E. Ganser's syndrome is a type of dissociative disor¬
Explicit memory (declarative memory) is the der. The other options are examples of dementia.
memory which is associated with awareness, 26. A, C.
88 Review of Psychiatry

27. B, D, E. 38. A, B, C.
Perhaps the use of word "treatable" is inappro¬ Short-term memory is lost first, long-term mem¬
priate here since all the types of dementia can be ory gets lost only inthe later stages of illness. Step
"treated". The examiner most likely wants to ask ladder pattern is typical of vascular dementia.
the types which can be "reversed" or "cured" 39. B, D, E.
28. A, B, C, D. Alzheimer's has an insidious onset and gradual
Vascular dementia presents with memory loss,
progression. Later in the course of disease epi¬
mood changes (depression, irritability, emo¬
sodic memory does get disturbed. Alzheimer's
tional lability), delusions and hallucinations,
disease primarily involves parietal and temporal
confusion and disorientation.
lobe.
29. A. Old age with history suggestive of a progressive
40. D.
impairment in memory, presence of behavioral
and psychological symptoms (hallucinations, 41. C. The presence of loss of memory, prosopagnosia
suspiciousness), poor self care and personality (difficulty in identifying face) in a 70-year-old
decline and a MMSE score <24, are all suggestive man is quite suggestive of Alzheimer's disease.
of dementia. Third person auditory hallucinations are usu¬
30. A. ally seen in schizophrenia, however they can be
31. D. present in Alzheimer's disease too. Further on
32. C. Smoking is considered to be one of the protec¬ examination, deep tendon reflexes are increased,
tive factors in Alzheimer's disease however this which again can be seen in late stages of Alzhei¬
finding hasbeen inconsistent across the studies. mer'sdisease. Finally MMSE score below 24 seals
33. D. Cerebral infarcts are a feature of vascular the diagnosis.
dementia and not dementia of Alzheimer's type 42. E. See text.
(Alzheimer's disease). 43. D.
34. A. InAlzheimer's ,the disease process usually spares 44. D. The frontotemporal dementias have a progressive
cerebellum. Especially neurofibrillary tangles are course and not static course.
never seen in cerebellum. 45. B. Insight is usually lost.
35. B. The best answer here is B. In reality, all four
46. A, C, D.
options given here are seen inAlzheimer's how¬
The psychiatric sequelae of stroke includes
ever, the DSM criterion for Alzheimer's disease
dementia, depression, mania, apathy, psychosis,
does not include acalculia as a symptom, while
emotional instability.
other three, aphasia, apraxia and agnosia have
47. A. The most common psychiatric disorder associ¬
been included.
36. D. Please remember that the neuropathological ated with hypothyroidism is cognitive slowing
diagnosis of Alzheimer's disease requires followed by depression.
extensive presence of bothsenile plaques (extra¬ 48. A. Myxedematous madness has been described in
cellular deposits) and neurofibrillary tangles a small number of patients with hypothyroidism.
(intracellular inclusions). The characteristic symptoms include auditory
37. C. hallucinations and paranoia (persecutory ideas).
Chapter

A7
«£mMbhmHhhbBH
Personality Disorders

Personality is defined as the dynamic organization within In other words, the individual with a personality disorder
the individual that determines his/her unique adjustment doesn't find anything wrong with himself and hence is
to his/her environment. The personality can be described often unwilling to take any treatment. DSM-5 has classi¬
under five broad dimensions. These five dimensions, also fied the personality disorders into three clusters.
called personality traitsQ can be remembered with the
pneumonic, OCEAN. Cluster A Personality Disorders
1. Openness to experience: It reflects the curiosity, nov¬
The following personality disorders are included in clus¬
elty seeking13 and desire to have new experiences.
ter A:
Individuals with high openness to experience may
A. Paranoid personality disorder: The characteristic
indulge in activities such as skydiving, bungee jump¬ feature is excessive suspiciousness and distrust of
ing, gambling, etc. others. These patients may be excessively sensitive*3
2. Conscientiousness: It reflects the tendency to be and may be quick to react angrily. They give excessive
organized, disciplined and dutiful. importance to themselves and believe in conspiracy
3. Extraversion-. It reflects the sociability, talkativeness theories. Psychotherapy is the treatment of choice.
and preference for group activities over solitary activi¬ Medications like benzodiazepines and antipsychotics
ties. may be used for agitation and paranoia (excessive sus¬
4 . Agreeableness: It reflects compassion and coopera¬ piciousness).
tion for others and a trusting and helpful nature. B. Schizoid personality disorder: These patients are
5. Neuroticism: It reflects the tendency to experience detached*3 from social relationships and prefer soli¬
unpleasant emotions easily. It also refers to the degree tary activities.They are emotionally cold*3 and are
of emotional stability. indifferent to praise or criticism. They appear self
If the personality of an individual deviates from social absorbed and lost in day dreams and may be preoc¬
norms and is a cause of unhappiness and impairment, cupied with fantasies. Since they are uncomfortable
the individual is diagnosed with a personality disorder. with human interaction, they have little interest in
Personality disorder is defined as presence of abnor¬ sexual activities. The management revolves around
mal behavior and subjective experiences which causes psychotherapy. The medications which are occasion¬
significant impairment. The prevalence of personality ally used include antipsychotics, antidepressants and
disorder is around 10-20% in the general population. benzodiazepines.
The onset is in adolescence or early adulthood*3, the C. Schizotypal personality disorder: These patients
symptoms remain stable throughout the adult life and have disturbances of thinking and communication.
maturing*3 occurs by around 40 years. Maturing means They frequently exhibit odd beliefs or magical think¬
the resolution of abnormal patterns of behavior. The per¬ ing*3 (e.g. superstitiousness, belief in telepathy or "sixth
sonality disorder are "ego syntonic"*3 (agreeable to self). sense"). Their inner world may be like that of a child,
i 90 Review of Psychiatry

filled with fears and frequently get involved in unlawful behaviors such
fantasies. They may In ICD-10, schizotypal disorder is as theft, lying, truancy and conning. They have a lack
not considered as a personality dis¬
have strange ways order, instead it is classified as a of remorse or guilt for their actions. Substance use
of communication psychotic disorder along with schizo¬ disorders are frequently present in these patients.
phrenia.
making it diffi¬ Treatment usually is psychotherapy. Medications like
cult to understand. carbamazepine, beta blockers are occasionally used.
They may also report illusions and other perceptual D. Borderline personality disorder: These patients are
disturbances. They usually don't have any close rela¬ almost always in a state of crisis. They have significant
tionships and appear "odd and eccentric" to others. mood swings. They may start feeling angry, anxious
When in severe stress, they may decompensate and or frustrated without any reason. Their interpersonal
have psychotic symptoms, but these are usually brief. relationships are intense and tumultuous. They swing
The management revolves around psychotherapy. from being excessively dependent to being hostile to
The medications which are occasionally used include persons close to them. Hence, they have a history of
antipsychotics, antidepressants and benzodiazepines. unstable relationships0. Another characteristic fea¬
The "cluster A" personality disorders (especially schi¬ ture is the repetitive self destructive acts0 such as
zotypal personality disorder) are considered to be on a slashing of wrists, or overdosage of medications. The
"schizophrenia continuum" which means that they lie patients indulge in these behaviors to elicit help from
somewhere in between the "normal" and "schizophrenia" others, to express the anger or just to numb them¬
selves to the overwhelming painful feelings they have.
Cluster B Personality Disorders These patients are also impulsive0 in areas such as
spending, sex and substance use. Finally, these patient
The following personality disorders are included in clus¬
excessively use the defense mechanism of splitting
ter B:
(wherein they consider each person to be either "all
A. Histrionic personality disorder. These patients are
good" or "all bad"). Management involves psycho¬
excitable and overtly emotional and behave in a
therapy. "Dialectical behavior therapy" is a therapy
dramatic and extroverted way. They want to be the
which has been designed for treatment of borderline
center of attention and exaggerate everything, mak¬
personality disorder. Medications used include anti¬
ing it sound more important than it really is. They
psychotics, antidepressant and mood stabilizers like
tend to behave in a sexually seductive manner and
carbamazepine. In ICD-10, the borderline personality
use physical appearance to draw attention towards
disorder has been described as a subtype of a broader
self. Management usually involves psychotherapy.
diagnosis of "emotionally unstable personality dis¬
Medications like antidepressants are occasionally
order"
useful.
B. Narcissistic personality disorder: These patients
have a heightened sense of self importance0. They
Cluster C Personality Disorders
believe that they are special and very talented. The following personality disorders are included in clus¬
They are preoccupied with fantasies of unlimited ter C:
success and power. They want to be admired by A. Avoidant personality disorder: These patients are
others. If condemned, they may become very angry excessively sensitive to rejection. They are afraid that
or they may show complete indifference to criticism. they would be criticized or rejected in social situa¬
They have a fragile self esteem and are susceptible to tions. Hence, they tend to remain socially withdrawn.
development of depression, when faced with rejec¬ These persons are usually unwilling to enter into a
tion. Management usually involves psychotherapy. relationship unless they are given a strong guarantee
Medications like antidepressants are occasionally of uncritical acceptance. The ICD-10, uses the diagno¬
useful. sis of anxious personality disorder for such patients.
C. Antisocial personality disorder (dissocial persona¬ Management mostly involves psychotherapy. Beta
lity disorder): These patients don't have regard for blockers and selective serotonin reuptake inhibitors
rights of others and frequently violate them. They (SSRIs) are also useful.
Personality Disorders 91

B. Dependent personality disorder: These patients are In comparison individuals with Type B personality
dependent on others for everyday decisions. All the are easy going and relaxed, they are not excessively com¬
major decisions in their lives are taken by someone petitive and may focus more on enjoyment and less on
else. They ask for excessive amount of advice and reas¬ winning or losing. Recent studies have suggested a new
surance from others. They also have difficulty express¬ personality type, Type D personality0 which is charac¬
ing disagreement with others because of fear of loss terized by negative affectivity (a tendency to experience
of support. They get very uncomfortable and helpless negative emotions) and social inhibition (tendency to
when alone and fear that they wont be able to take inhibit expression of emotions). Individuals with Type D
care of themselves. Management usually involves psy¬ personality are predisposed to development of coronary
chotherapy. Benzodiazepines and SSRIs can be used heart disease0.
for symptomatic relief.
C. Obsessive compulsive personality disorder: These IMPULSE CONTROL DISORDERS_
patients are preoccupied with rules and regula¬ These disorders are characterized by irresistible impulses
tions. They give excessive importance to details and or temptations to perform a particular act which is harm¬
show perfectionism that interferes with task com¬ ful to self or others. Impulse is described by patients as a
pletion (since they want everything to be perfect, it feeling of increasing tension and arousal that leads to per¬
often results in significant delays). They are infle¬ formance of a certain behavior. The performance of the
xible and insist that others agree to their demands. behavior gives a sense of relief and also gratification. After
They are excessively devoted to work and may not some time, however the person feels guilty or remorseful.
The following are described as impulse control disorders.
have any time for leisure activities. They are for¬
All of them are preceded by the irresistible impulses:
mal and serious and often lack a sense of humor.
1. Pyromania: Recurrent and purposeful setting of fires.
The ICD-10 , used the diagnosis of "anankastic per¬ 2. Kleptomania°: Recurrent stealing of objects which
sonality disorder" for these patients. Management are not needed for personal use or are of no monetary
usually involves psychotherapy. value.
3. Intermittent explosive disorder: It is characterized by
Type A and B Personality episodes of aggression resulting in serious assault or
destruction of properties.
Another way of classifying personality is what is known
4. Pathological gambling: Recurrent episodes of gam¬
as Type A and Type B personality. Type A personality
bling which causes economic troubles and serious
is characterized by competitiveness, time urgency,
relationship problems.
hostility and anger. The people with Type A personality 5. Trichotillomania: Recurrent episodes of hair pulling.
are ambitious, impatient and hard working workaholics. 6. Others: These include, Oniomania or compulsive buy¬
Many studies have suggested that Type A personality ing: Recurrent episodes of buying or shopping despite
(especially the hostility and anger traits) is a risk factor the buying behavior causing significant monetary and
for coronary heart disease0. socio occupational distress.

mm
QUESTIONS AND ANSWERS

QUESTIONS _ 2. True about personality disorder: (PGIJune 2007)


A. Onset in early childhood and adolescence
.
1 Which of the following is not a personality trait? B. Matures around adulthood
(AIIMS Nov 2009) C. Not associated with social norms
A. Sensation seeking D. Direct result of disease or damage
B. Neuroticism 3. Characteristic disorder that appears in late child
C. Open to experience hood and continues in adulthood:
D. Problem solving (DNB NEET 2014-15)
92 Review of Psychiatry

A. Somatoform disorder 11. Antisocial personality is associated with:


B. Personality disorder (PGI1999)
C. Anxiety disorder A. Drug abuse
D. Mood disorder B. Paranoid schizophrenia
(PGI2003) C. Obsessive compulsive disorder
4. True about personality disorder:
early childhood and adoles¬ D. None
A. Typically onset at
cence 12. Features of borderline personality are:
B. Mature around at 30-40 years (PGIMay 2012)
C. Ego dystonic A. Impulsivity
D. Dramatic, emotional and erratic behavior in B. Recurrent suicidal behavior
paranoid PD C. Anger and anxiety
E. Pervasive and maladaptive behavior
D. Extreme suspiciousness
5. True about personality disorder: (PGIJune 2008) E. Pattern of unstable and intense interpersonal
A. Onset in early childhood and adolescence relationships
B. Matures around adulthood
13. Cardinal feature of antisocial personality disorder
C. Suspiciousness is seen in paranoid personality
is: (DNB NEET2014-15)
disorder
A. Violation of rules of society
D. Excessive preoccupation with fantasy is seen in
B. Attention-seeking behavior
schizoid personality disorder
C. Unstable interpersonal relationships
6. Oddities of speech, mannerism, odd clothingwith D. Grandiose behavior
magicalthinking is seen inwhich type of persona¬
lity disorder: (DNB 2003, JIPMER-2K) 14. A 16-year-oldgirlwasbroughttopsychiatricemer-
A. Schizoid B. Paranoid gency after she slashed her wrist in an attempt to
C. Schizotypal D. Borderline commit suicide. On enquiry her father revealed
that she had made several such attempts of wrist
7. Which personality disorder of DSM-IV is not
slashing inpast, mostly inresponse to trivial fights
classified as PD and is placed with schizophrenia
inherhouse. Further she hasmarked fluctuations
in ICD 10? (WB 1998, DNB 1997)
in her mood with a pervasive pattern of unstable
A. Schizoid B. Paranoid
interpersonal relationships. The most probable
C. Narcissistic D. Schizotypal
diagnosis is: (AIIMS Nov 2002)
8. Characteristic feature of schizoid personality A. Borderline personality disorder
disorder is: (AIIMS 1999) B. Major depression
A. Conversion reaction C. Histrionic personality disorder
B. Not concerned with disease D. Adjustment disorder
C. Check details of all things
D. Emotional coldness 15. Patients who are grandiose and require admira¬
tion from others, have which type of personality?
9. Which personality disorder can be consi¬
(DNB NEET 2014-15)
dered a part of autistic spectrum disorders?
A. Narcissistic B. Histrionic
(DNB NEET 2014-15)
C. Borderline D. Antisocial
A. Schizoid B. Schizotypal
C. Borderline D. All of the above 16. A young lady was admitted with h/o taking over¬
10. Markedly inappropriate sensitivity, self impor¬ dose of diazepam after broken affair. She has
tance and suspiciousness are clinical features of: history of slitting her wrist previously. Most likely
(DNBNEET2014-15, DNB2001, TN1999,AMU2002) diagnosis is: (AIIMS 2000)
A. Antisocial PD B. Histrionic PD A. Narcissistic PD B. Dependent PD
C. Schizoid PD D. Paranoid PD C. Borderline PD D. Histrionic PD
Personality Disorders 93

A. Coronary artery disease


17. A person has the habit of inflicting repeated inju¬
ries to self, what is the type of personality? B. Depression
(PGIJune 2004) C. Schizophrenia
A. Borderline B. Schizoid D. Mania
C. Histrionic D. Narcissistic
E. Depressive Impulse Control Disorder
18. Pervasive pattern of instability of interpersonal 25. Kleptomania is: (PGIMay 2011, 2007)
relationships, self image and affect, with marked A. Delusional disorder
impulsivity that begins at early adulthood and B. Obsession
present in varieties of context is characteristics C. Impulse disorder
of: (Bihar 2006)
D. Compulsion seclusion
A. Bipolar disorder
E. Hallucination
B. Schizoaffective disorder
C. Borderline personality disorder 26. One of the following is not a compulsive and habit
D. Schizotypal personality disorder forming disorder: (KA 1995)
19. A lady has changed multiple boyfriends in last 6 A. Kleptomania
months, she keeps breakingher relationships, and B. Pyromania
she also has attempted suicide many times. Most C. Nymphomania
likely diagnosis is: (MP 2006) D. Pathological gambling
A. Borderline personality disorder
B. Post-traumatic stress
C. Acute depression
ANSWERS _
1. D. Sensation seeking is a part of "openness to experi¬
D. Acute panic attack
ence" Problem solving is not a personality trait.
20. A person with shy, anxious avoidant personality 2. A,B,C.
comes under which cluster? (AIIMS May 2015) Personality disorders have onset in early child¬
A. Cluster A B. Cluster B hood and adolescence and maturing occur in
C. Cluster C D. Cluster D adulthood by 30-40 years of age. People with
21. Obsessive personality disorder is also called: personality disorders tend to have conflicts with
(DNBNEET2014-15) the societal norms (e.g. patients with antisocial
A. Anankastic personality disorder personality disorders tend to break societalrules
B. Dissocial personality disorder and regulations).
C. Eccentric personality disorder 3. B.
D. Histrionic personality disorder 4. A,B,E.
Personality disorders are "ego syntonic" and not
22. True about treatment of personality disorder:
(PGIMay 2010) "ego dystonic" Option D is description of histrio¬
A. Antipsychotics are used
nic personality disorder.
B. SSRI are used 5. A, B, C, D.
C. Behavior therapy is used See text
D. No need for treatment 6. C. Odd behavior including odd speech, manner¬
isms and magicalthinking is seen in schizotypal
23. False regarding Type A personality:
personality disorder.
(AIIMS Nov 2007)
7. D. InICD- 10, schizotypal PD is placed with schizo¬
A. Hostile B. Time pressure
phrenia spectrum and not in personality disor¬
C. Competitiveness D. Mood fluctuations
der.
24. Individual with Type D personality are recently 8. D. See text.
found to be at risk of developing: 9. A. The characteristic feature of autistic spectrum
(AIIMS Nov 2011) disorder (ASD) is impairment insocial interaction
94 Review of Psychiatry

andcommunication. These features are also seen 16. C. The repetitive episodes of self harming behavior
in schizoid personality disorder. There can be after stressors is suggestive of borderline perso¬
significant difficulty differentiating between nality disorder.
schizoid PD and milder forms of ASD. It must 17. A.
be remembered that patients with ASD have 18. C.
more severe social impairment and also have 19. A.
stereotypical behaviors and interests. 20. C.
10. D. See text. 21. A.
11. A. Antisocial PD is frequently associated with sub¬ 22. A, B, C.
stance use disorders.
The mainstay of treatment inpersonality disorders
is psychotherapy. Medications used include
12. A,B,C,E.
SSRIs, antipsychotics and mood stabilizers.
See text.
23. D.
13. A. People with antisocial PD characteristically
24. A.
disregards rights of others, don't follow norms 25. C. Kleptomania is an impulse control disorder in
of society and indulge in antisocial behaviors. which the patient has recurrent irresistible desire
14. A. This patient has history suggestive of self harming to steal objects, which he/she doesn't need for
behavior with mood fluctuations and pervasive personal use or for monetary value.
unstable pattern of interpersonal relationships, 26. C. Nymphomania is the condition of excessive
all of which are features of borderline PD. sexual desire in females. It is not an impulse
15. A. control disorder.
Chapter

8 Eating Disorders

ANOREXIA NERVOSA Subtypes


Anorexia nervosa is most commonly seen in adolescent Anorexia nervosa has the following two subtypes.
females. Initially, it was reported to be more common 1. Restricting type: This type is seen in around 50% of
in upper class, however recent data doesn't support that patients and is characterized by highly restricted food
fact. It must be noted that anorexia nervosa is a misnomer intake.
since the appetite of these patients is usually normalQ 2. Binge eating/purging subtype: It is seen in 25-50%
and hence there is no symptom of anorexia in anorexia of patients. In this type, patient alternates attempts at
nervosa. rigorous dieting with intermittent binging and purg¬
ing episodes. The binging involves intake of a large
It is characterized by the following signs and symp¬
amount of food in a short duration with an associated
toms:
feeling of lack of self control during binge episode. The
1. Disturbance of body image (patient perceives that she
purging is a compensatory mechanism wherein patient
is fat despite being quite thin in reality). tries to compensate for excess calories by self induced
2. Excessive fear of fatness and excessive emphasis on vomiting, laxative use, diuretic use or emetic use. The
thinness. repeated vomiting episodes may cause dental caries,
3. Restriction of energy intake resulting in a significantly parotitis, and hypokalemic alkalosis.
less weightQ than normal.
4. Medical symptoms secondary to starvation such as Treatment
amenorrhea0, lanugo (appearance of neonatal hairs),
hypothermia, dependent edema and bradycardia. The treatment may DSM-5 Update: In DSM-4, ameno¬
include hospitaliza¬ rrhea was a necessary symptom
The adolescent patients often have poor sexual deve¬ for diagnosis of anorexia nervosa,
tion to restore patients however in DSM-5 this criterion has
lopment0 whereas the adult patients usually report low0 nutritional status and been removed and anorexia nervosa
interest in sexual activities. Patients often exhibit pecu¬ can be diagnosed in the absence of
manage complications amenorrhea now.
liar behavior0 about food such as hiding food in the like dehydration and
house, trying to dispose food in napkins, cutting food into electrolyte imbalances. The treatment focusses on a com¬
very small pieces and rearranging the food repeatedly bination of behavioral management (praise for healthy
around the plate. These patients are preoccupied with eating habits, restriction of self induce vomiting), indi¬
the thoughts about food and may spend a large amount vidual psychotherapy and family education. Medications
of time collecting recipes or cooking food for others. such as cyproheptadine, tricyclic antidepressants (TCAs)
Patients are usually secretive and deny any symptoms and selective serotonin reuptake inhibitors (SSRIs) have
and refuse for treatment. been tried with varied success.
96 Review of Psychiatry

BULIMIA NERVOSA _ 4. Weight is usually normalQ, and is an important dif¬


ferentiating factor between bulimia nervosa and ano¬
Bulimia nervosa is characterized by episodes of binge rexia nervosa.
eating combined with inappropriate ways of prevent¬ The patients with bulimia nervosa usually tend to have
ing weight gain. Bulimia nervosa is more common than features secondary to purging such as enamel erosionQ
anorexia nervosa, is usually seen in females, and the age and dental cariesQ, salivary gland inflammations, callus
of onset is mostly late adolescence. The following are the on knucldesQ (as knuckles get injured against teeth during
clinical features: episodes of self induced vomiting). The patient may develop
1. Episodes of binge eating in which large amount of hypokalemia and hypochloremic alkalosis and rarely gas¬
food is usually consumed in a small duration with an tric and esophageal tear during forceful vomiting.
associated feeling of lack of self control during binge Patients have normal sexual functioning0 and are
episode. usually not secretive about their symptoms as p atients
2. Compensatory behavior after binge eating to prevent with anorexia nervosa.
weight gain. These measures usually include purg¬
ing behaviours like self induced vomiting, laxatives Treatment
or diuretics abuse, use of emetics and in few patients It is usually outpatient and involves psychotherapeutic
excessive exercising (hyperglycemia) and dieting. techniques like cognitive behavioural therapy (first line)
3. Like patients of anorexia nervosa, the patients with and dynamic psychotherapy. The medications mostly
bulimia nervosa too have a morbid fear of gaining used are antidepressants like selective serotonin reuptake
weight and give excessive emphasis to thinness. inhibitors.

pp

1 QUESTIONS AND ANSWERS

QUESTIONS _ 4. With regard to anorexia nervosa all of the follow¬


ing are true except:
.
1 Which of the following is not a common feature of
(DNB NEET 14-15, DNB 03, Kerala 2K)
anorexia nervosa? (DNB 2007, AI 2006)
A. Phobic avoidance of normal weight
A. Binge eating
B. Over perception of body image
B. Amenorrhea
C. Self induced vomiting
C. Self perception of being fat
D. Menorrhagia
D. Underweight
E. Excessive exercise
2. Anorexia nervosa can be differentiated from
5. A young lady presents with h/o repeated episodes
bulimia by: (AIIMS NOV 2008)
A. Intense fear of weight gain of over eating ( binge) followed by purgingusing
B. Disturbance of body image laxatives, she is probably suffering from:
C. Adolescent age (AI2002, UP 2004, AIIMS 10,07, DNB 2009)
D. Peculiar patterns of food handling A. Bulimia nervosa
B. Schizophrenia
3. Which of the following is not true about bulimia
C. Anorexia nervosa
nervosa? (UPSC2009)
D. Benign eating disorder
A. Recurrent bouts of binge eating
B. Lack of self control over eating during binge 6. Which of the following is not true about bulimia
C. Self induced vomiting or dieting after binge nervosa? (UPSC-1 08)
D. Weight gain A. Invariable weight loss with endocrine disorder
Eating Disorders 97

B. Occurrence of both binge eating and inappropri¬


ate compensatory behaviors at least twice weekly
ANSWERS __
1. A. Allthefouroptionsarefeaturesofanorexianervosa.
on an average for 3 months
However, if one has to chose, the best answer
C. Recurrent episodes of binge eating
D. Recurrent self induced vomiting would be binge eating. Though binge eating is
seen in almost 50% of patients with anorexia
7. False regarding anorexia nervosa:
nervosa, however its not a core symptom of
(DNB 2008, AI2006)
anorexia nervosa.
A. Evident psychosis
B. Vigor exceeding physical well-being 2. D. Unlike patients with bulimia, patients with ano¬
C. Weight loss rexia remain preoccupied with food and show
D. Decreased appetite peculiar behavior like hiding food in the house,
trying to dispose food in napkins, cutting food
8. False regarding anorexia nervosa:
into very small pieces and rearranging the food
(DNB NEET 2014-15)
repeatedly around the plate.
A. Psychiatric symptoms such as depression may
3. D. The patients with bulimia nervosa usually have
be associated
B. Excessive exercising can be a feature normal weight.
C. Weight loss is a feature 4. D. Amenorrhea and not menorrhagia is the men¬
D. Decreased appetite is a feature strual disturbance seen in anorexia.
5. A.
9. Following are true about bulimia nervosa except:
6. A. Weight loss and endocrine abnormality are seen
(DNB NEET 2014-15)
A. Uncontrolled eating episodes in anorexia not bulimia nervosa.
B. Overweight individuals 7. A. There are no psychotic symptoms in anorexia
C. Depressive symptoms are present nervosa.
D. Patients are sexually active 8. D. The appetite of patients with anorexia is normal
and as such there is no anorexia in anorexia
10. Not true about bulimia nervosa is:
(DNB NEET 2014-15) nervosa.
A. Onset is in late adolescence 9. B.
B. Dental caries/tooth decay is a finding 10. C. Presence of amenorrhea is a differentiating fea¬
C. Amenorrhea is a common finding ture between anorexia and bulimia. Itis seen only
D. Normal weight is usually seen in patients with anorexia.
9 Sleep Disorders

ELECTROENCEPHALOGRAM _ Stage 2, NREM: It is the stage with maximum



duration0. It is characterized by two typical find¬
It is the recording of electrical activity of the brain. It is
ings on electroencephalogram:
recorded by placing electrodes on the scalp and recording a. Sleep spindlesQ: These are bursts of regular
the potential difference between various electrodes. A waves (frequency of 13-15 Hz, 50 microvolt) and
normal EEG has following types of rhythm. b. K-complexesQ: These are high voltage spikes
which are seen intermittently.
Stages of Sleep • Stage 3, NREM: The sleep deepens and there is
appearance of delta waves.
Sleep can be divided into two stages:
• Stage 4, NREM: This is deep sleep and is characte¬
A. Nonrapid eye movement sleep (NREM) or slow wave
rized by predominance of delta waves on EEG.
sleep and
During the NREM sleep, there is pulsatile release of
B. Rapid eye movement (REM) sleep or paradoxical
gonadotropins and growth hormones. Further, the blood
sleep.
pressure, heart rate and respiratory rate also decreases.
A. Nonrapid eye movement sleep: It is further divided B. Rapid eye movement sleep: It follows the NREM
into following four stages: sleep. It is characterized by the following:
• Stage 1, NREM: It is the first stage and the sleep • The EEG shows increased activity similar to awake
is light (person can be easily aroused). The EEG state (beta activity) along with return of alpha
shows, loss of alpha waves (which predominate activity
when person has eyes closed but is still awake) and • Presence of rapid eye movements
predominance of theta waves. • There is generalized loss of muscle tone.

Table 1: EEG rhythms.


EEG rhythm Frequency (Hz) Amplitude (microvolt) Salient points Region
Alpha (a) 8-12 50-100 Seen when individual is awake, at rest, Present maximally in occipital
eyes closed and mind wandering and parieto-occipital area
Beta (P) 14-30 5-10 Normal awake pattern, when attention Predominantly in frontal area
is focussed beta waves appear
Theta (0) 4-7 10 Transition from wakefulness to sleep, Parietal region and temporal
early sleep region (hippocampus)
Delta (5) 1-4 20-200 Deep sleep
Sleep Disorders 99

. Increased rateQ of metabolism in brain contractions, however the bed partner frequently
• Penile erection0, autonomic hyperactivity (increase gets disturbed. The patient may report non restora¬
in pulse rate, respiratory rate and blood pressure) tive sleep and day time sleepiness. The treatment
• Dreams0, which can be recalled are seen during usually involves benzodiazepines.
REM sleep. • Restless leg syndrome (Ekbom syndrome): It is char¬
Ponto geniculo occipital spikes0 (large phasic poten¬ acterized by uncomfortable sensation in legs (such
tials that originate from cholinergic neurons in pons and as insect crawling on the skin) which get relieved by
pass rapidly to lateral geniculate body and then to occipi¬ moving the leg or walking around. This can cause
tal cortex) are a characteristic feature. difficulty in initiation of sleep as patient keeps on
REM sleep is called paradoxical sleep0 because moving the leg. The only approved drug for treat¬
though the EEG is quite similar to awake state, its quite ment is ropinirole0 (a dopamine agonist).
difficult to awaken the patient. B. Hypersomnia: Primary hypersomnia is diagnosed
In a 8 hour sleep, maximum time (around 6-6.5 hours)
when no cause can be found for excessive sleepi¬
is spent in NREM sleep and the rest (around 1.5 hours) in
ness which can present with either prolonged sleep
REM sleep. Most of the stage 4, NREM occurs in the first
episodes or excessive day time sleep episodes.
one-third of the night whereas most of REM sleep occurs
Few other disorders which can present with hyper¬
in the last one-third of the night. The REM sleep occurs
somnia include:
regularly after every 90-100 minutes with a total of around
• Narcolepsy: This disorder is characterized by the
4-5 REM sleeps in the entire night.
following symptoms:
a. Sleep attacks: The patient has irresistible urge
SLEEP DISORDERS _ for sleep which can occur at any time during
The various sleep disorders can be divided into two cate¬ the day.
gories: b. CataplexyP: It is sudden loss of muscle tone,
1. Dyssomnias due to which patient can even have a fall.
2. Parasomnias c. Hypnagogic hallucinations°: These are the
hallucinations, which occur while going to
Dyssomnias sleep. Patient may also have hypnopompic
hallucinations0 (hallucinations while getting
These disorders are characterized by abnormality in the
up from sleep).
duration or quality of sleep. They include:
d. Sleep paralysis: It usually occurs when the
A. Insomnia: Primary Insomnia is diagnosed when no
patient gets up in the morning. Though he has
cause can be found for decreased sleep and may
woken up, he is not able to move his body.
present with difficulty in initiation of sleep, difficulty
The hallmark of narcolepsy is reduced latency of
in maintenance of sleep (frequent awakening during
REM sleep0. Normally, it takes around 90 minutes to
night or early morning awakening) or nonrestora-
reach REM sleep (after crossing all the stages of NREM
tive sleep (not feeling refreshed in the morning due
to poor quality of sleep). The management usually
sleep) however in patients with narcolepsy, patient
involves use of benzodiazepines, Zolpidem and other reaches REM sleep much earlier.
hypnotics. The management includes a regimen of forced naps
Few other disorders which can present with insomnia at regular time. The medications used are modafinil and
include: other stimulants like amphetamines.
• Periodic limb movement disorder: It is charac¬ • Kleine-Levin syndrome: This is a rare disorder
terized by sudden contraction of muscle groups which is characterized by episodes of hypersom¬
(usually leg) while sleeping. This results in partial nia0, hyperphagia and hyper sexuality0 (increased
or complete awakening, repeatedly in the night. sexual activity). In between the episodes patient is
The patient is usually not aware of these sudden essentially asymptomatic.
| 100 Review of Psychiatry

Parasomnias of choice is bed alarms0, which start ringing,


as soon as child passes urine. The medications
These disorders are characterized by dysfunctional which can be used include tricyclic antidepres¬
events associated with the sleep. These include:
sants such as imipramine0, although their use
A. Stage 4, NREM sleep disorders: These disorders occur
is associated with severe side effects. Intranasal
during stage 4, NREM (also stage 3, NREM). Since
desmopressin0 is a better alternative.
most of the stage 4, NREM is present in first third
of the sleep, these disorders are also seen in the
• Bruxism (teeth grinding®): The patient grinds his
teeth making loud sounds and there may be dam¬
same period. Also, the patient is not able to recall
age to the enamel of teeth.
the events in the morning. These disorders are usu¬
ally seen in children and include:
• Sleep talking (somniloquy): Patient talks during
stage 3 and 4, NREM and is unable to recall the
• Night terror or sleep terror (pavor nocturnusQ): same in the morning.
The patient suddenly gets up screaming and has
In most cases these disorders do not require any
symptoms of intense anxiety such as tachycardia
treatment and the parents must be reassured. In some
and sweating. The patient is not able to recall any
cases, benzodiazepines0 are prescribed. As benzodia¬
dream or reasons for feeling scared.
zepines decrease the duration of stage 4, NREM, they
• Sleep walking (somnambulism°): The patients also decrease these episodes.
may carry out a range of activities for which he B. Other sleep disorders:
doesn't have any memory later on. It may include
• Nightmare: It occurs during REM sleep, wherein
leaving the bed and walking about and also activi¬ patient has a bad dream and gets up scared and
ties like dressing, moving around or even driving. has behavioral signs of anxiety such as tachycar¬
• Sleep related enuresis: The enuresis which is dia and hypertension. In contrast to night terror,
defined as voiding of urine at inappropriate in nightmare, the patient is able to recall the
places, is nocturnal in around 80% of cases. The dream. Agents that reduce duration of sleep, such
most common cause of bed wetting are psy¬ as tricyclic antidepressants can be used for treat¬
chosocial such as sibling rivalry. The treatment ment.

mmmw
QUESTIONS AND ANSWERS imMzm

QUESTIONS C. Brain shows increased metabolism


D. EEG shows decreased activity
.
1 Maximumduration of time spent is inwhich of the
following NREM stage? (NEET/DNB) 4. Slow wave in hippocampal area is: (MP 00)
A. I B. II A. Delta B. Theta
C. Ill D. IV C. Beta D. Alpha
5. Alpha-rhythm is seen in: (PGI1997)
2. A middle aged man complains of lack of sleep dur¬
A. Sleep with eyes closed with mind wandering
ing the night time. The duration of the time he is
B. Mental activity
truly asleep or awake can beascertained by which
C. Awake with eyes open
of the following? (AIIMS Nov 2012)
D. REM sleep
A. Barograph B. Kymograph
C. Actigraphy D. Plethysmography 6. Pontogeniculo occipital spike is characteristic of
which of the following sleep stage?
3. Not a feature of paradoxical sleep is: (PGI 1999) (DNBNEET2014-15)
A. Decreased muscle tone A. Stage INREM B. Stage 2 NREM
B. Rapid eye movements C. Stage 3 NREM D. REM
Sleep Disorders 101

7. The EEG recorded shown below is normally 14. Drug of choice for night terrors: (PGI 1998)
recordable during which stage of sleep: (AT2003) A. Meprobamate
B. Tricyclic antidepressant
K complex
C. Clonazepam
D. Diazepam
14 Hz
Sleep spindles
activity 15. Feature of narcolepsy include (s) all except:
(12-14 Hz) (PGIMay2013)
A. Disorder of REM sleep regulation
B. Disorder of NREM sleep regulation
C. Hypnagogic hallucination
D. Hypnopompic hallucinations
E. Cataplexy
16. Not true about narcolepsy: (PGIDec 2006)
A. Stage I B. Stage II
C. Stage III D. Stage IV A. Sudden sleep
B. Long duration (>3 hrs) of sleep
8. What are the EEG waves recorded for parieto C. Cataplexy
occipital region with subject awake and eyes D. Presents in Ilnd decade
closed? (Kerala 1997)
17. Modafinil is approved by FDA for treatment of all
A. Alpha waves B. Beta waves
except: (DNB 2006, AI2009)
C. Delta waves D. Theta waves
A. Obstructive sleep apnea syndrome (OSAS)
9. Which one of the following phenomenonis closely B. Shift work syndrome (SWS)
associated with slow wave sleep? C. Narcolepsy
(AIIMS Nov 2004) D. Lethargy in depression
A. Dreaming B. Sleepwalking 18. Following is true about ropinirole:
C. Atonia D. Irregular heart rate (DNB NEET2014-15)
A. Selective D2/3 receptor agonist
10. Not true about nocturnal penile tumescence is:
B. It is used in resdess leg syndrome
A. Totals about 100 min/night (AIIMS 1995)
C. Both A and B
B. Normal phenomenon D. None of the above
C. Occurs in NREM sleep
19. Regarding, Kleine-Levin syndrome which of the
D. Can be used to distinguish between psychologi¬
following is not true: (DNB NEET 2014-15)
cal or organic impotence
A. Hypersomnia
.
11 Which of the following conditions are seen during B. Hyposexulity
NREM sleep? (DNB NEET 2014-15) C. Spontaneous resolution
A. Teeth grinding B. Nightmares D. Also called sleeping beauty syndrome
C. Narcolepsy D. Sleep paralysis
ANSWERS
12. Pavor nocturnus is: (APPG 1997)
A. Sleep terror B. Sleep apnea l. B.
C. Sleep bruxism D. Somnambulism 2. C. Actigraphy is the procedure which is used for
studying the sleep patterns. It usually involves
13. Antidepressant drug used in nocturnal enuresis wearing a small sensor on the wrist, which
is: (AI2011) detects the movements. However, the gold
A. Imipramine B. Fluoxetine standard technique for studying sleep disorders
C. Trazodone D. Sertraline is polysomnography.
102 Review of Psychiatry

3. D. Inparadoxical sleep or REM sleep, the EEG shows 12. A.


increased activity, similar to awake state. 13. A. Rememberits not the drug of choice. Desmopres¬
4. B. sin is the drug of choice and bed alarms are the
5. B, D. treatment of choice.
Alpha rhythm is seen when a person is awake with 14. C,D.
eyes closed and his mind is wandering (having Benzodiazepines can be used in night terrors
mental activity) and not when a person is sleep¬ though usually no treatment is required.
ing with eyes closed. Also, alpha rhythm is seen 15. B.
in REM sleep. 16. B. The onset of narcolepsy is mostly in adolescence
6. D. See text. or young adulthood. There are sudden sleep
7. B. The sleep spindles and K complexes are seen in
attacks which last for 10-20 minutes (and not
stage II, NREM.
more than 3 hours) and cataplexy is a feature.
8. A.
17. D. Modafinil is FDA approved for narcolepsy, shift
9. B. Somnambulism is usually seen inNREMIIIand
IV (slow wave sleep). work sleep disorder and as an adjunct in obstruc¬
10. C. Nocturnal penile erections are a feature of REM tive sleep apnea.
sleep. 18. C. Ropinirole is a dopamine agonist (D2, D3 recep¬
11. A. Teeth grinding or bruxism is seen in NREM III tors) and is approved for restless leg syndrome.
and IV. 19. B. There is hypersexuality and not hyposexuality.
Chapter

\0 Sexual Disorders

Gender Identity Disorders there is no desire to permanently change the sexQ.


There is no sexual arousal0 associated with cross
Gender is the sense of being a male or a female. Mostly dressing. (Remember, in fetishistic transvestism,
the gender corresponds to the anatomical sex, (i.e. a man which is a type of paraphilia, the cross dressing is
with male body organs, also psychologically considers associated with sexual arousal).
himself as a male), however there might be a mismatch
resulting in gender identity disorder. The following are Treatment: In patients who insist for sex change, sex
types of gender identity disorder: reassignment surgery can be done. In a person born
A. Gender identity disorder of childhood: It usually mani¬ anatomically male, removal of penis, scrotum and tes¬
fests in preschool years. The child shows preoccupa¬ tes and construction of labia and vagina is done. In a
tion with the dress and activities of the opposite sex person born anatomically female, bilateral mastectomy,
(e.g. the male child insists on wearing skirts and frocks hysterectomy, removal
of ovaries and con- (30- --
and may play exclusively with dolls and reject the cars \~OdSM-5 Update: In DSM-5, the diag- 1
and other toys which are usually preferred by boys). struction of a neophal- nosis of "gender dysphoria" is used in
lus (penis) is done. The place of DSM-4 diagnosis of "gender
The child expresses the desire to be of the opposite sex , , identity disorder".
and rejects behaviors, attire and attributes of his ana¬ hormonal treatment ÿ
--
tomical sex. Usually, there is no feeling of rejection of usually accompanies.
the anatomical structures however in a small minority
it may be present (e.g. the male child may repeatedly Disorders of Sexual Orientation
assert that the penis and testicles are disgusting and
It must be remembered that homosexuality is not a psy¬
will disappear in due course of time). chiatric disorder (homosexuality is considered as a normal
B. Transsexualism: In adolescents and adults, the symp¬ variant, if it is ego syntonic, i.e. the individual accepts his
toms are quite similar to gender identity disorder of
sexual orientation) however ego dystonic homosexuality
childhood. The patients manifest a desire to liveQ (where in the individual doesn't accepts his sexual ori¬
and be treated as the other sex, usually accompanied entation and wants to change it) has been classified as
by a discomfort with one's anatomical sexQ and a a disorder.
desire to changeQ it with the help of a surgery or some
other form of treatment. The patient frequently uses Disorders of Sexual Response
the phrases like "I am a man trapped in body of
woman" The homosexual orientation is frequently Phases of Sexual Response Cycle
present. Normally sexual response has been divided into four
C. Dual-role transvestism: The patient wears the clothes phases.
of opposite sex, to enjoy the temporary feelingQ of A. Desire: It is characterized by a desire to have sex (hypo-
belonging to the other sex. Unlike transsexualism, active sexual desire disorder is a disorder of this phase).
ÿ 104 Review of Psychiatry

B. Excitement (arousal): This phase is characterized Treatment: The medications with best evidence
by penile erection and vaginal lubrication. Other include PDE-5 inhibitors0 (phosphodiesterase-5
changes such as nipple erection, enlargement of inhibitors like sildenafil, tadalafil and vardenafil,
size of testes and elevation of testes, engorgement which facilitate blood flow into penis and enhance
and thickening of labia minor and clitoris, and erection. The other medications which can be used
physiological changes like increased heart rate, include oral phentolamine (decreases sympathetic
blood pressure and respiratory rate are also seen. tone and relaxes smooth muscles of corpora caver¬
There is an associated subjective sense of pleasure nosa) and injectable and transurethral alprostadil.
(erectile dysfunction is a disorder of this phase). Alprostadil contains naturally occurring prosta¬
C. Orgasm: There is a peaking of sexual pleasure, glandin E and hence has vasodilator action. It can
followed by release of sexual tension and ejaculation be injected into corpora cavernosa or administered
of semen. In females, orgasm is characterized by intraurethrally.
involuntary contraction of lower third of vagina Apart from medications, psychotherapy also plays
and contractions from fundus downward to cervix. an important role. The most successful is dual-
(premature ejaculation and anorgasmia are disorders sex therapy0 (or simply sex therapy) which was
of this phase). developed by Masters and Johnson. This therapy
treats the "couple"0 and not the individual0. The
D. Resolution: The body goes back to the resting state.
couple is taught ways to improve their communi¬
There are disorders specific to each phase of sexual cation. The couple is also taught exercises which
cycle as described below: increases the sensory awareness. These exercises
A. Sexual desire disorders: It has been further subdivided are called, sensate focus exercises. Initially, the
into two categories: hypoactive sexual desire disorder, couple is asked to touch, rub, kiss on each oth¬
characterized by lack of desire for sexual activity and ers body parts, excluding breasts and genitals (this
sexual aversion disorder, characterized by active aver¬ stage is called nongenital sensate focus). In next
sion and avoidance of sexual activity. The only FDA stage, the same activities are done on breasts and
approved drug for treatment of hypoactive sexual genitals (called genital sensate focus). The whole
desire disorder in females is flibanserin, which got purpose is to make the couple aware that pleasure
approval in August 2015. Due to risk of severe hypo¬ can be given and received by methods other than
tension, flibanserin /ÿ\ _ sexual intercourse. The sex therapy is effective not
should not be taken V< DSM-5 Update: In DSM-5, the diag- only for erectile dysfunction but other sexual dis¬
mitantlv with nosis of sexual aversion disorder has
been removed.
orders like premature ejaculation.
alcohol. - Other techniques such as behavioral therapy,
B. Disorders of excitement (arousal) phase: hypnotherapy and psychoanalysis have also been
• Male erectile disorder (erectile dysfunction): It is used.
characterized by recurrent or persistent inability • Female sexual arousal disorder: It is characterized
to attain or to maintain the erection required for by inability to achieve adequate vaginal lubrication
satisfactory sexual intercourse. Erectile dysfunction required for sexual intercourse. The management
is usually caused by psychological factors such as involves use of lubricants during the intercourse.
anxiety and poor marital relation. C. Disorders of orgasm phase:
The presence of early morning erections and • Premature ejaculation: It is characterized by a pattern
erections during REM sleep (nocturnal erections0) of persistent or recurrent ejaculation with minimal
are suggestive of psychogenic erectile dysfunction. sexual stimulation before or immediately after the
Investigation such as penile plethysmography and vaginal penetra¬
nocturnal penile intumescence (NPT)Q are used tion. In DSM-5, the criterion for prema¬
to record nocturnal erections. Hie cause of ture ejaculation has been defined
more clearly, and states that prema¬
The physical causes include vascular and neuro¬ premature ejacu¬ ture ejaculation is a pattern of ejacula¬
logical disorders like arteriolosclerosis and auto¬ lation is usually tion within approximately one minute
following vaginal penetration.
nomic neuropathy. psychogenic.
Sexual Disorders 105

Treatment : Specific techniques have been described • Male orgasmic disorder (retarded ejaculation): It
for the management of premature ejaculation. is characterized by recurrent delay or absence of
These include: orgasm in males. It is less common than premature
a. Squeeze technique:Q: When the man gets the feel¬ ejaculation and is treated with psychotherapy.
ing of impending ejaculation, the female partner D. Other disorders:
(or the man himself) squeezes the coronal ridge • Dyspareunia: It is recurrent or persistent genital
of glans, which results in inhibition of ejaculation. pain in either men or women, before, during or
b. Stop-start technique (Semans technique): Here, after sexual intercourse.
when the man gets the feeling of impending
• Vaginismus: It is involuntary muscle constriction of
ejaculation, the sex is stopped for some time and outer third of vagina which makes penile insertion
once excitement has decreased, it is restarted.
difficult. Vaginismus and dyspareunia frequendy
Apart from these techniques, sex therapy (as
coexist.
described earlier) is also an effective method of
treating premature ejaculation. • NymphomaniaQ:
It is the term used DSM-5 Update: Genito-pelvic pain/
SSRIs (selective serotonin reuptake inhibitors) penetration disorder is new in DSM-5
to describe exces¬
are also frequently used as they can delay the and represents a merging of the
ejaculation. sive sexual desire DSM-4 categories of vaginismus and
dyspareunia, which were highly com-
• Female orgasmic disorder (anorgasmia): It is charac¬ in females. orbid and difficult to distinguish.
terized by recurrent delay or absence of orgasm in • SatyriasisQ: It is
females. It is a common sexual disorder in females the term used to describe excessive sexual desire
and the treatment involves psychotherapy. in males.

QUESTIONS AND ANSWERS

QUESTIONS _ C. Sildenafil induced erection


D. Squeeze technique
1. Most accurate treatment of erectile dysfunction
is: (PGI2002) 5. Squeeze technique is used for: (AI 2000)
A. Sildenafil A. Impotence
B. Master and Johnson technique B. Premature ejaculation
C. (3 -blockers C. Infertility
D. Papaverine D. Priapism
2. Excessive sexual desire in males in known as: 6. A 30-year-old male presents to OPD with erectile
(AIIMS May 2008) dysfunction. Basic screening evaluation is unre¬
A. Nymphomania B. Satyriasis markable. The next step in evaluation/manage¬
C. Tribadism D. Sadism ment should be: (AI2008)
3. A homosexual person feels that he is imposed by A. Oral sildenafil titrate trial
a female body and has persistent discomfort with B. Cavernosometry
his sex. Most likely diagnosis is: (PGI2003) C. Doppler study
A. Gender identity disorder B. Transvestism D. Neurological testing
C. Voyeurism D. Paraphilias 7. A 20-year-old girl Neelu enjoys wearing male
4. How to differentiate between psychological and clothes. Wearing male clothes gives her feeling of
organic erectile dysfunction? (NEET/DNB) more confidence and after these episodes she is
A. Nocturnal penile tumescence an absolutely normal girl. The likely diagnosis is:
B. PIPE test (AIIMS 1997)
106 Review of Psychiatry

A. Transsexualism disorder. Most of the patients with gender iden¬


B. Fetishism tity disorder have homosexual orientation.
C. Dual role transvestism 4. A. Presence of early morningerections and erections
D. Fetishistic transvestism during REM sleep (nocturnal erections) are
8. True about dual sex therapy is: (DNB June 2011) suggestive of psychogenic erectile dysfunction.
A. Patient alone is not treated As during sleep, there is no anxiety, hence a
B. Uses sildenafil patient with psychogenic erectile dysfunction
C. It treats sexual perversion is able to have erections. Whereas, a patient
D. It is used for people with dual gender identities with organic erectile dysfunction (due to vas¬
cular or neurological causes ) won't have erec¬
ANSWERS _ __ tions even during sleep. Investigation such as
penile plethysmography and nocturnal penile
1. A, B. intumescence (NPT) can be used to record
The pharmacological treatment with best nocturnal erections.
evidence in erectile dysfunction is phospho- 5. B. Squeeze technique and stop-start techniques are
diesterase-5 inhibitors like sildenafil. The used for treatment of premature ejaculation.
psychotherapeutic technique which is most 6. A. In a young patient with negative screening,
commonly used is Master's and Johnson's tech¬ the most likely cause of erectile dysfunction is
nique. psychogenic erectile dysfunction. He should be
2. B. Satyriasis is the condition of excessive sexual given a trial of oral sildenafil.
desire in males while the same in females is 7. C. Here the person only enjoys wearing clothes of
known as nymphomania. opposite sex and there is no discomfort with her
3. A. As mentioned in the question the person is own sex and there is no desire to be of other sex.
uncomfortable with his sex and feels that he is Hence, it is a case of dual role transvestism.
imposed by a female body (i.e. he is of another 8. A. Indual sextherapy, the couple is treated and not an
sex), both are characteristics of gender identity individual.
Chapter

Child Psychiatry

ATTENTION DEFICIT HYPERACTIVITY Depending on the


DISORDER (ADHD)/HYPERKINETIC predominant symp¬ DSM-4 required the presence of
symptoms before age of 7 years,
DISORDER_
toms, three subtypes however according to DSM-5 the
have been defined. onset of symptoms should be before
12 yrs of age to make a diagnosis
The DSM-5 uses the diagnosis of ADHD whereas in ICD- A. Combined presen¬ of ADHD.
10, the corresponding diagnosis is hyperkinetic disor¬ tation " '

der. It is a common neuropsychiatry disorder of child¬ B. Predominantly inattentive presentation


hood, which is more prevalent in boys in comparison to C. Predominantly hyperactive/impulsive presentation.
girls. The predominant symptoms and signs in the ADHD
are as follows: Course
A. Inattention: The child has difficulty in giving close
attention to details, makes frequent mistakes in Around 50 percent0 of patients achieve remission before
school work and other activities. The child is distracti- puberty and early adulthood. Others achieve only partial
bleQ and frequently shifts from one activity to another remission and are at risk of developi2ng substance use
as he loses interest in one task quickly. disorders0 (particularly alcoholism), antisocial persona¬
B. Hyperactivity and Impulsivity: The child is hyper¬ lity disorder and mood disorders.
active and appears resdess. Teacher frequendy com¬
plains that child keeps on roaming in the class and is Treatment
excessively talkative and disturbs other studentsQ.
ADHD is a serious medical illness0 and must be promptly
The child is also impulsive0 and often blurts out
treated. The pharmacological treatment is the mainstay of
answer before question has been completed. He also
the treatment. The following medications are used:
has difficulty in waiting for his turn and often inter¬
rupts others or intrudes in others conversation. A. Stimulant medications: CNS stimulants are the
Along with the core symptoms of inattention and first line drugs in the treatment of ADHD. Methyl-
hyperactivity/impulsivity, children with ADHD frequendy phenidate0 is the drug of choice. Dexmethylphenidate
show destructive and aggressive behavior and are irri¬ (containing only the d-enantiomer) has also been
table. used recently. Other stimulant medications used
Neurological examination may reveal soft neurologi¬ are dextroamphetamine, lisdexamphetamine and
cal signs0 (the neurological soft signs are fine abnormali¬ "dextroamphetamine and amphetamine salt" com¬
ties found during detailed neurological examination such binations. Modafinil has also been used with varied
as difficulty in copying age appropriate figures, difficulty success.
in performing rapid alternating movements, difficulty in B. Nonstimulant medications: The nonstimulant medi¬
right left discrimination, etc). cations are used if stimulants are not effective or
I 108 Review of Psychiatry

contraindicated. They include atomoxetine (norepi¬ have development of abnormal language such as
nephrine reuptake inhibitor), clonidine, guanfacine, difficulty in making sentences properly (articula¬
venlafaxine and bupropion. tion difficulties) and pronoun reversals (using "me"
instead of "you").
Apart from medications, psychosocial interventions
such as social skill training, psychoeducation for parents, • Restricted, repetitive and stereotyped behavior: The
behavioral therapy and cognitive behavioral therapy are activities and play of these children tend to be
also effective in the management. repetitive9 and boring. They may show stereotyped
behaviors like hand wringing9, spinning and bang¬
ing. These children are quite resistant to changes
PERVASIVE DEVELOPMENTAL
and may become extremely upset if their routines

DISORDER) _
DISORDERS (AUTISM SPECTRUM are disturbed (e.g. bathing routine is changed or
furnitures are rearranged in the room). These
These are group of neurodevelopmental or neurobeha- restrictive behaviors usually result from a lack of
vioral disorders9, which are characterized by disturbance imagination and creativity.
of social interaction, abnormalities of communication Apart from these three characteristic features, the
and restricted behaviors. The following are the subtypes patients with autism also have abnormal responses
of pervasive developmental disorders: to stimuli. They may have a higher threshold for pain
A. Autism (Childhood autism, autistic disorder): It is and may show intense interest in some sounds (like
a neurodevelopmental disorder (neurobehavioral dis¬ that of a ticking watch) and may totally ignore other
order) with a strong genetic basis. The onset is before sounds. They may also have self destructive behaviors
the age of 3 years. Chromosome 7,2,4,15 and 19 have like head banging, biting , scratching, etc.
been found to contribute to the disorder. Fragile X Precocious skills or islets of precocity: Some indi¬
syndrome9, tuberous sclerosis, congenital rubella and viduals with autism may have skills in certain areas,
phenylketonuria are associated with autism and are which are much higher than their normal peers. For
found with high frequency in children with autistic example, hyperlexia (early ability to ready very well),
disorders. Around 70%9 of children with autism have extremely good rote memory or calculating ability,
comorbid mental retardation. The prevalence of peri¬ etc.
natal insults like birth asphyxia has also been found Treatment: Educational interventions such as a
to be higher in children with autism. Following are the structured classroom teaching along with use of
symptoms: behavioral therapy is the recommended treatment.
• Impairment in social interaction: The patients with The role of medications is limited. Atypical antipsy¬
autism have impaired reciprocal social skills. As chotics such as risperidone and aripiprazole have
infants they have poor eye contact9, lack social been used to reduce aggressive and self injurious
smile9 and anticipatory posture (the posture which behavior.
the kid assumes when he wants to be picked up). B. Rett's disorder (Rett's syndrome): Earlier it was
They may have poor attachment to their parents believed that Rett's disorder occurs exclusively in
and other important persons and may not acknow¬ females9, however of late males with similar presen¬
ledge their presence (e.g. they won't come running tation have been described. It is characterized by nor¬
to meet when the father returns to home after mal development till the age of 5 months. Between
office). However, if the routine of these children is 5-48 months , the child starts to lose acquired hand
disturbed (e.g. if someone rearranges the furniture skills (such as fine motor skills) and there is loss of
in their room), they may show excessive reaction9. acquired speech. Also, there is deceleration of head
When they grow up, they may have difficulty in mak¬ circumference producing microcephaly9. The child
ing friends and getting into a romantic relationship. gradually develops stereotyped hand movements
• Impairment of communication and language: These such as hand wringing, licking or biting of fingers.
children usually have significant delay in lan¬ The language function remains impaired and there
guage milestones9, whereas the motor milestones is also loss of social interaction. The child also deve¬
are usually normally achieved. The patients also lops poorly coordinated gait or trunk movements.
Child Psychiatry 109 H
Along with these symptoms around 75% of children requests and rules, frequent loss of temper and often
have seizures. The disorder is usually progressive and
deliberately annoying adults. However unlike conduct
treatment is symptomatic. disorder, there are no serious violations like theft, fire
C. Childhood disintegrative disorder (Heller's syn-
setting, destruction, etc.
dome): It is characterised by normal development
The management for both involves family interven¬
till the age of 2
years. Between 2-10
@ -tion and behavioural therapy. In some cases, low dose
antipsychotics have been found to be effective.
DSM-5 Update: In DSM-5, the term
years there is loss pervasive developmental disorder has
been replaced by autism spectrum
of acquired motor disorder. Also, all the subtypes, such LEARNING DISORDERS (SPECIFIC
skills, social skills, as autism, Rett's disorder, Aspergers
language skill and
bowel or bladder
syndrome have been removed. Now,
autism spectrum disorder is consi¬
dered as a continuum with difference
in severity, rather than presence of
DEVELOPMENTAL DISORDERS OF
SCHOLASTIC SKILLS) _
control. The child
subtypes. These developmental disorders are characterized by
develops the three
significant impairment in one or more of the scholas¬
core symptoms of impaired communication, impaired
tic skills0 which are out of proportion to the intellectual
social interaction and repetitive, stereotyped behavior.
functioning of the child. For example, a child may pre¬
The course is usually
sent with significant difficulty in reading while having
progressive though DSM-5 Update: In DSM-5, in
some patients may autism spectrum disorders, language normal writing and arithmetic skills and a normal IQ°.
show improvement.
dysfunction has been removed as a Depending on the symptoms, the subtypes have been
core criterion. Only impaired social
The treatment is interaction and repetitive, restrictive described.
symptomatic.
behaviors form the core criterion now. A. Specific reading disorder (Dyslexia): The child's
D. Asperger's syndrome: It is characterized by impair¬ reading performance is significantly impaired and he
ment of social interaction and restricted, repetitive may make errors while reading, may have slow read¬
and stereotyped behavior. However no language delay ing speed or may have difficulty in comprehension.
or disturbance is seen. The treatment is usually sup¬ B. Disorder of written expression (specific spelling dis¬
portive. order): The child may make frequent spelling mis¬
takes, errors in grammar and punctuations and may
CONDUCT DISORDER AND have poor hand writing.
C. Specific disorder of arithmetic skills: The area of
OPPOSITIONAL DEFIANT DISORDER
impairment is arithmetics.
A. Conduct disorder: It is characterized by repetitive D. Mixed disorders of scholastic skills: There is impair¬
and persistent pattern of disregard for rights of ment in reading, writing and arithmetics combined.
others0 and aggressive0 and dissocial behavior0, Apart from the above mentioned symptoms, the child
such as excessive levels of fighting or bullying, cruelty may have associated problems such as inattention, hyper¬
to animals or other people, severe destruction of activity and emotional disturbances.
property, fire setting, stealing , truancy from school,
repeated lying0, frequent running from school and MENTAL RETARDATION_
home, defiance of authority figures and a pattern
of disobedience. Conduct disorder is frequently Mental retardation is a condition characterized by incom¬
associated with unsatisfactory family relationships plete development of intellectual functions and adaptive
and failure at school. These children may later on skills (skills which help an individual live a successful
develop antisocial personality disorder (dissocial life). The intelligence is usually measured by calculating
personality disorder). the Intelligence Quotient (IQ).
B. Oppositional defiant disorder. It is less severe than IQ = Mental age/Chronological age x 100°
conduct disorder and is characterized by persistently In this formula, the maximum denominator is 15, even
negativistic and defiant behavior such as frequent if assessment of an older individual is being performed.
arguing with adults, refusal to comply with adults Mental retardation is diagnosed if the IQ is less than 70.
110 Review of Psychiatry

IQ An earlier classification of retardation used the words


Category
90-109
"idiots, imbecile, and moron"
Normal
Borderline 70-89 Term IQ range
Mild mental retardation 50-69 51-70
Moron
Moderate mental retardation 35-49
26-50
Imbecile
Severe mental retardation 20-34
Idiot 0-25
Profound mental retardation < 20
The level of functioning varies in different severity of men¬ The most common
tal retardation. The following table summarizes the same. chromosomal cause of DSM-5 Update: In DSM-5, the
diagnosis of mental retardation has
Mental mental retardation is been replaced v/ith "intellectual dis¬
age as Educational Down syndromeQ fol¬ ability.
Category Class adults achievement Life Work
lowed by fragile-X syn¬
Mild MR Educable 9-12 yrs Upto 6th Independent Unskilled
class living or semi¬ drome'3.
skilled work
Moderate Trainable 6-8 yrs Upto 2nd Needs some Unskilled Behavioralproblems inmental retardation:The patients
MR class education or semi¬
skilled
with mental retardation may have maladaptive behavior
work such as aggression, self injurious behaviors, hyperacti¬
Severe Dependent 3-6 yrs No formal Needs Simple vity, etc. These behaviors can usually be modified using
MR education attention task-under
supervision behavioral therapy techniques like contingency manage¬
Profound Needs life < 3 yrs No formal Needs None ment13, in which the desired behaviors are rewarded and
MR support education continuous
supervision undesired behaviors are punished.

QUESTIONS AND ANSWERS

QUESTIONS 3. A 10-years-old child presents with hyperactivity


and inattention. Parents are extremely worried,
1. Which of the following is not seen ina hyperkinetic what would you say to the parents?
child? (DNB 1993, AT 1991) (AIIMS Nov 2008)
A. Aggressive outbursts A. It is a normal behavior
B. Decreased attention span B. Child has a behavioral problem and should
C. Left to right disorientation receive behavior therapy
D. Soft neurological signs C. Child has a serious problem and should receive
medical therapy
2. A 9-year-old child disturbs other people, is des¬
D. There should be a change in environment
tructive, interferes when two people are talking,
does not follow instructions and cannot wait for 4. ADHD in childhood can lead to what in future:
his turn while playing a game. He is likely to be (PGI2000)
suffering from: (AIIMS Nov 2005) A. Schizophrenia B. Alcoholism
A. Emotional disorders C. Intellectual changes D. Antisocial behavior
B. Behavioral problems 5. Following drugs are used in the treatment of
C. No disorder ADHD: (DNB NEET 2014-15)
D. Attention deficit hyperactivity disorder A. Amphetamine
Child Psychiatry 111
B. Modafinil C. Impaired folate level
C. Methylphenidate D. A socioeconomic hazard
D. All of the above E. Result of wrong parenting
6. Drug(s) used in treatment of attention-deficit 12. Autism is: (PGI2000)
hyperactivity disorder: (PGIDec 2008) A. Neurodevelopmental disorder
A. Atomoxetine B. Methylphenidate B. Social and language communication problem
C. Dexmethylphenidate D. Quetiapine C. Metabolic disease
E. Dextroamphetamine
D. Mainly due to hypothalamus damage
7. Not an associatedcomorbidcondition inchildren
13. A 6-year-old child with history of birth asphyxia
with hyperkinetic attention deficit disorder is:
does not communicate well, has slow mental and
(DNB Dec 2010)
physical growth, does not mix with people, has
A. Elimination disorder B. Anxiety disorder
limited interests and gets widely agitated if dis¬
C. Sleep disorder D. Language disorder
turbed. Diagnosis is: (AIIMS 2001)
8. A neurodevelopmental disorder which is charac¬ A. Hyperkinetic child
terized by impaired social interaction, impaired B. Autistic disorder
verbal and nonverbal communication, and C. Attention deficit disorder
restricted and repetitive behavior is description D. Mixed receptive-expressive language disorder
for: (DNB NEET 2014-15)
A. Autism 14. A girl with normal milestones spend her time
B. Anxiety disorder seeing her own hand and does not interact with
C. Antisocial personality disorder others. What is the likely diagnosis: (AIIMS 2008)
D. Paranoid schizophrenia A. ADHD B. Autism
C. Asperger's syndrome D. Rett's disorder
9. 10-years-old child presents with impaired social
interaction, impaired communicationand stereo¬ 15. A 2-year-old girl child is brought to the out patient
typed behavior. He has normal IQ and language with features of handwringing stereotyped move¬
skills. What is the most probable diagnosis? ments, impaired language and communication
(DNB NEET 2014-15) development, breath holding spells, poor social
A. Asperger's syndrome skills and deceleration of head growth after six
B. Autism months of age. The most likely diagnosis is:
C. Rett syndrome (AIIMS Nov 2003)
D. Childhood depression A. Asperger syndrome B. Rett's syndrome
10. A 3 -year-old child has normal developmental C. Fragile X-syndrome D. Cotard syndrome
milestones except delayed language development 16. Which of the following is not seen in autism?
(poor speech development). He has difficulty
(AIIMS Nov 2014)
in concentration, communication, and making
A. 2/3rd patients are mentally retarded
friends (i.e. hehas no friends) and spends time see¬
B. Poor eye contact
inghis own hands.The most probable diagnosis is:
C. Language is impaired
(AI2012, AIIMS Nov 2006)
D. Abnormal dermatoglyphics
A. Autism
B. ADHD 17. A child with pervasive developmental disorder
C. Specific learning disability will have all except: (AIIMS Nov 2015)
D. Mental retardation A. Stereotyped behavior
.
11 Infantile autismis characterized by:(PGIDec 2004) B. Reduced social interaction
A. Impaired vision C. Poor language skills
B. Impaired neurobehavioral development D. Impaired cognition
ÿ 112 Review of Psychiatry
18. Which of the following disease is seen only in B. Can study upto 8th standard
females? (DNB Dec 2011) C. Can follow simple verbal commands
A. Autism B. Asperger's syndrome D. Can handle money
C. Rett's disease D. Colard disease E. Recognize family members
19. Rett's syndrome is characterized by all except: 27. All of the following statements about 'Imbecile'
(DNB NEET 2014-15, AIIMS 2013) are true, except: (AT 2011)
A. Regression of acquired skills A. IQ is 50-60
B. Breath holding spells B. Intellectual capacity is equivalent to a child of
C. Autistic behavior 3-7 years of age
D. Macrocephaly C. Impaired self care
20. IQ is: (DNB NEET 2014-15) D. Condition usually congenital or acquired at an
A. Mental age/chronological age x 100 early age
B. Chronological age/mental age x 100
28. X-linked disease leadingto mentalretardationis:
C. Mental age + chronological age x 100 (PGI2000)
D. Mental age - chronological age x 100 X-syndrome
A. Myotonic dystrophy B. Fragile
21. According to Wechsler intelligence scale scoring, C. Tuberous sclerosis D. Phenylketonuria
average IQ of a normalchild is: (AIIMS 2013)
29. Best therapy suited to teach daily life skill to a
A. 50 B. 75
mentally challenged child: (AIIMSMay2011, 2009)
C. 90 D. Ill
A. CBT (Cognitive behavior therapy)
22. A 16-year-old male is found to have a mental age B. Contingency management
of 9 years on IQ testing. He has:(AIIMS May 2005) C. Cognitive reconstruction
A. Mild mental retardation D. Self instruction
B. Moderate mentalretardation
30. A 14-year-old boy is not able to get good grades
C. Severe mental retardation
in 9th standard exam. But he is very sharp and
D. Profound mental retardation
intelligent. Best test to diagnose his problem:
23. Which of the following score is not included in (AIIMS 2012)
mild mental retardation? (PGIMay 2012) A. Child behavior checklist
A. 85 B. 50 B. Bhatia's battery
C. 45 D. 75 C. Specific learning disability test
E. 65 D. Child behavior battery
24. A patient with IQ 30 will be diagnosed with: 31. A child finds difficulty to spell and read, otherwise
(DNB NEET 2014-15)
his IQ is normal, interacts well with parents and
A. Mild mental retardation friends. Visionisnormal. Most probable diagnosis
B. Moderate mental retardation
of the condition is? (DNB June 2011)
C. Severe mentalretardation
A. ADHD B. Dyslexia
D. Profound mental retardation
C. Autism D. Asperger syndrome
25. True about mental retardation: (PGINov 2011)
32. A 14-year-old boy has difficulty in expressing
A. More common in females than males
himself in writing and makes frequent spelling
B. Severe MR is IQ < 20
C. Antenatal factor can cause mental retardation
mistakes. He passes his examination with poor
D. Common cause is down's syndrome marks. However his mathematical ability and
E. Life long inability to learn and progress social adjustment are appropriate for his age.
Which of the following isthe most likely diagnosis?
26. In a child with IQ 50, which of the following is
(AIIMS Nov 2004)
true: (PGI2001) A. Mental retardation
A. Can look after himself independently B. Specific learning disability
Child Psychiatry 113 fl
C. Lack of interest in studies adversely affect his future life. Also, medications
D. Examination anxiety like methylphenidate are the first line treatment.
4. B, D.
33. A boy presents with history of abnormal excessive
Kindly note, that few books have also mentioned
blinking and grunting. He says he has no control
intellectual changes as an answer. This is not
over his symptoms which have risen infrequency
true. Though children with ADHD tend to have
of late. This has started affecting his social life and
lower educational achievements, however it is
is making him depressed. Which of the following
not because of any intellectual impairment but
medications should be used inthe management?
because of poor attention and hyperactivity.
(AIIMS May 2015)
5. D.
A. Carbamazepine B. Imipramine
6. A, B, C, E.
C. Risperidone D. Methylphenidate
See text
34. Appetite for nonnutritive substances is called: 7. A. Elimination disorders are not a common comor¬
(DNBNEET2014-15) bidity inADHD.
A. Pica B. Anorexia 8. A.
C. Bulimia D. Binge 9. B
10. A. The child has all the three core features of autism,
35. Scholastic performance is impaired in all of the
(AI2012) impairment in social interaction (difficulty in
following except:
making friends), impaired communication and
A. Attention deficit hyperactivity disorder
repetitive, stereotyped behavior (spends most
B. Specific learning disorder
time seeing own hands).
C. Anxiety
11. B, D.
D. PICA
Autism is a neurodevelopmental or neurobeha-
36. Conduct disorder in a child manifests with: vioral disorder. It causes socioeconomic pro¬
(PGI2001) blems as a majority of autistic patients remain
A. Disregard for right of others dependent on others however use of term
B. Doesn't care for authority "hazard" is a bit insensitive here.
C. Backward in studies 12. A,B.
D. Decreased head circumference 13. B. The history of poor social interaction and
E. Steals things restricted behaviors alongwith history of agitation
when disturbed supports the diagnosis of autism.
ANSWERS_ _ Around 70% of children with autism have
comorbid mental retardation. The prevalence
1. C. Ihe best answer here would be left to right dis¬
of perinatal insults like birth asphyxia has been
orientation. Please remember, in ADHD, "left to
found to be higher in children with autism.
right discrimination" difficulties can be found.
14. B. The history ofrestricted behaviors and poor social
However, the term "left to right disorientation"
interaction is suggestive of autism.
is used for describing gross inability to distin¬
15. B. The deceleration of head growth after 6 months
guish left from right and is usually a feature of of age followed by repetitive, stereotyped
Gerstmann's syndrome. behavior (wringing hand movements), impaired
2. D. This child has symptoms of hyperactivity and communication and poor social interaction is
impulsivity and the most likely diagnosis would suggestive of Rett's syndrome.
be attention deficit hyperactivity disorder. 16. D.
3. C. The symptoms are suggestive of ADHD. ADHD 17. D.
is a serious medical problem and should be 18. C. Rett's syndrome was earlier believed to occur
treated properly. The symptoms of ADHD inter¬ exclusively in females, however of late males
fere with education of child, and if not treated with similar presentation have been described.
child's education may suffer greatly and will 19. D.
114 Review of Psychiatry
20. A. money, can have an independent living,study till
21. C. According to Wechsler intelligence scale, the 6th class.
following is the classification. 27. A. The IQ of imbeciles is between 26-50. Hence,
IQ range IQ classification most of their features would correspond to that

130 and above Very superior of moderate mental retardation.


120-129 Superior B.
110-119
29 B. See text.
High average
C. The history of poor academic performance
90-109 Average
despite goodintelligence should raise suspicion
80-89 Low average
of learning disorders (specific learning disabi¬
70-79 Borderline
lity). Hence, he should take a specific learning
69 and below Extremely low
disability test to rule out the same.
22. A The formula for IQ is mental age/chronological 31. B. The history of difficulty in reading and spelling
x 100. However, please remember that the maxi¬ mistakes inpresence of normal IQ is suggestive of
mum denominator can be 15. Inthis case 9/15 X learning disorders (specific learning disability).
100 = 60. Hence, it will come under the categoiy 32. B. Scholastic difficulty in a particular skill (written
of mild mental retardation. expression) is suggestive of specific learning dis¬
23. A, C, D. ability.
The range for mild mental retardation is IQ from 33 C. The history of motor tics (abnormal excessive
50-69.
blinking) and vocal tics (grunting) is suggestive
24. C.
of tics disorder (possibly tourette syndrome).
25. C, D.
Antipsychotics like haloperidol and risperidone
Mental retardation is more common in boys,
are the drugs of choice.
severe MR is IQ < 35 and patients with mild and
moderate MR can learn.
34, A.
26. A, C, D, E. 35- D.
IQ of 50 corresponds to mild mental retardation. 36- A, B, C, E.
People with mildmentalretardation can handle See text.
12 Psychoanalysis

The term "psychoanalysis" was coined by Sigmund Freud started treating hysterical patients, wherein he
FreudQ who is also known as "father of psychoanalysis"9. would try to retrieve the unconscious memories during
Freud (1856-1939)Q was born in Freiburg, Moravia (now the treatment procedure. Freud developed a technique
in Czech republic) and lived most of his life in Vienna9. called "free association" in which the patient was asked
He died in London in 1939. to say whatever came into their minds without censoring
Psychoanalysis is a theory which states that the child¬ their thoughts. With the help of this technique, Freud
hood experiences and memories and unconscious men¬ was able to gain access to unconscious memories, which
tal activity (activity of mind which we are not aware of) would come out as patient would start saying all that came
plays an important role in determining human behavior into their minds and would not try to stop any thought.
and emotions and also in the development of psychiatric Freud also gave a lot of importance to slips of the tongue
disorders. The term "psychoanalysis" is used not only to (which he called parapraxis9). Freud believed that these
refer to this theory but also for the treatment method "slips of tongues" were not simple mistakes, and that
which is based on this theory. these slips actually conveyed important information9
The theory of psychoanalysis was developed by about what was going on in the unconscious mind.
Freud while working with patients of hysteria (the term The psychoanalytic treatment provided by Freud also
hysteria is no longer used, these patients will get a used the principles of transference and countertransfe-
diagnosis of "dissociative disorder" according to cur¬ rence.
rent classification). In particular Freud came to know Transference9 is the feeling that the patient develops
about a patient Anna O, who had developed multiple for the doctor. This feeling is a combination of the feelings
unexplained neurological symptoms including paralysis patient had for figures from the past and the real feeling
of limbs, after the death of her father. Whenever she was for the clinician. For example, if the doctor reminds the
able to recall how a particular symptom originated, that patient of his dominating and insensitive father, the patient
symptom would improve. For example, once she was able will develop a negative feeling for the doctor, despite the
to recall that on one occasion while she was sitting at her fact that doctor has not done anything to offend him.
sick father's bedside, she had a daydream that a snake Countertransference9 is the feeling that the clinician
was crawling towards her father and while she wanted develops for the patient.
to ward off the snake she couldn't do it as her arm had
gone into sleep. As soon as Anna O, was able to recall Topographical Theory of Mind
this event, the paralysis of her arm improved. This case In 19009, Freud published a book called "The interpre¬
provided Freud a strong demonstration, that unconscious tation of dreams"9. In this book, Freud said that dreams
memories (memories which an individual has forgotten, were meaningful and by understanding dreams, one can
but which are still present in the unconscious mind) can understand about the unconscious mind of an individual.
result in development of symptoms. In this book, Freud proposed a theory of mind, called the
116 Review of Psychiatry

topographical theory of mind. According to this theory consequences). The primary process thinking is illogical
the mind can be divided into three regions: and contradictory.
A. Ihe conscious
B. The preconscious Structural Theory of Mind
C. The unconscious Later in his life, Freud replaced the topographical theory
A. The conscious: It is the part of mind which is acces¬ of mind with a newer theory, called the structural theory
sible to us. We are aware of the contents of conscious of mind. According to this theory, there are three com¬
mind. Everything you know about yourself is a part of ponents of mind: id, ego and superego.
conscious mind. A. Id: It is the most primitive part of mind with which
B. The preconscious: The content of preconscious an infant is born. Id consists of the instinctual drives.
mind are not normally available to us, but they can It is that part of mind which wants to have pleasure
be recalled or brought into awareness by focusing and that too immediately. Id doesn't care about the
attention. For example, you may not be aware of the external word or any consequences. Id hence works
appearance of your 5th class teacher, however if you on "pleasure principal". Id uses the primary process
try to focus and remember hard, you might be able thinking. Id is completely in the unconscious domain
to recall her appearance. The preconscious separates of mind.
the conscious and unconscious mind. The precon¬ B. Ego: Ego is that part of mind which deals with the
scious mind has a barrier, called repression, which external world. The part of your mind which is reading
normally doesn't allow the contents of unconscious this book is "ego" Apart from dealing with the external
mind to reach the conscious mind. If any uncon¬ world, another important function of ego is to deal
scious memory has to reach the conscious awareness, with the "id" and "superego" and maintain a balance
it must find a way to overcome die force of "repres¬ between the two and the external word. Since, the ego
sion" Freud reported that during sleep, the repression maintains a balance and helps in dealing with the rea¬
force becomes lax, and many unconscious memories lities of the outside world, it is said to work on "reality
and desires are able to reach the conscious in the principal". Ego is said to be the "executive organ" of
form of dreams. That's why Freud believed that the the mind. Ego has both conscious and unconscious
interpretation of dreams can reveal the contents of components. The "defense mechanisms" reside in the
unconscious memories and desires. Further, when a unconscious component of ego.
person indulges in "free association" few unconscious C. Superego: It is that part of our mind, which wants to
contents are able to cross the barrier of repression and follow the moral principles and do the right thing.
are able to come out in the form of "slips of tongue" The voice of conscience, which scolds you, when you
C. The unconscious: The unconscious mind is not acces¬ are not studying, comes from superego. Superego is
sible to an individual. The unconscious mind contains, mostly unconscious, but also has a conscious com¬
the instinctual drives (i.e. the drives and desires one is ponent.
born with) such as sexual instinct and aggressive instinct. To understand how these components work, an exam¬
Further, distressing childhood memories and distressing ple can be illustrated. While you are studying, your id
desires are also buried inside the unconscious. These wants you to throw away the books and instead go out

contents are not available to the conscious mind due to and have fun and indulge in some pleasurable activity.
the barrier of "repression" Freud believed that by not On the other hand, your superego wants you to study very
allowing these memories to reach conscious, repression hard without taking many breaks and stay away from all
causes development of psychiatric symptoms and disor¬ distractions. Finally, your ego does a balancing act and
ders. you decide that you will study for two hours and after
The unconscious mind is characterized by "primary that you will take a break and will watch a movie. This is
process thinking"Q. This is primitive way of thinking in how, ego always keeps a balance.
which the mind wants immediate 'wish fulfillment" and As mentioned in this example, conflicts keep on going
instinctual discharge (wants all desires and instincts in the mind (between id, ego and superego) and these
to be fulfilled immediately without considering the unconscious conflictsÿ in the mind are believed to cause

;
Psychoanalysis 117 1
psychiatric disorders according to the psychodynamic (or dropped the glass. Here, the child was able to express
psychoanalytic) theories. his anger indirectly by dropping the glass.
E. Regression: Attempt to return to an earlier phase of
Defense Mechanisms development (i.e. childhood) to avoid the tensions
and conflicts of present phase of development (i.e
An important function
adulthood). For example, extremely stressed because
of ego is to prevent a
build up of excessive
—/Ml the defense mechanisms oper-
ate at an unconscious level (except,
of an upcoming entrance examination, a medical
, , ,, . suppression which is a conscious and students goes to a park and starts playing cricket
and unbearable anxi- voluntary defense mechanism). along with the children. Regression is involved in
ety. Many unaccep¬
development of neurosisQ.
table urges, if they reach the conscious awareness, can F. Projective identification: In this defense mechanism,
produce excessive anxiety. Defense mechanisms are the intolerable aspects of self are projected on to another
tools used by the "ego" to prevent the development of person, that person is induced to play the projected
excessive anxiety. The defense mechanisms have been part and the two persons than act in unison. For
divided into four groups: narcissistic, immature, neurotic example, a wife who has lots of aggression can
and mature defense mechanisms. Following are the project her aggression on to the husband, and make
important defense mechanisms: him behave in an aggressive manner and finally
a system develops where the husband indulges in
Narcissistic Defenses
A. Denial: It is refusal to acknowledge the reality. The
aggression and wife is the recipient of aggression.
Please remember all of this happens unconsciously
person continues to behave as if nothing has hap¬
without entering into awareness of either the wife
pened. For example, a mother refused to accept that
or the husband. Projective identification is seen in
her seven year old son died in an accident and insists
patients with borderline personality disorder.
that he will be back for dinner.
B. Projection: Projecting "own" unacceptable feeling Neurotic Defenses
about others, on to others. For example, a husband G. Displacement: Shifting emotions about one object/
with an unacceptable wish of indulging in infidelity, individual onto another object/individual. For exam¬
starts accusing his wife of indulging in infidelity. ple, after being scolded by his consultant, a senior
Here, the husband has "projected" his own wish on resident comes to the ward and started shouting at
to the wife. This defense mechanism is responsible for the intern. Here, actually the senior resident is angiy
development of delusions and hallucinations. at die consultant but he is displacing his anger on the
intern. Displacement is involved in the development
Immature Defenses of phobiasQ.
C .Acting out Acting on unconscious desires without H. Intellectualization: Excessive use of intellectual
becoming aware of them. For example, a person process to avoid the painful emotions. For example,
suddenly steals an item from a shop without any prior a doctor who was diagnosed with pancreatic cancer
planning. In this case, this person had an unconscious has a long discussion about the pathophysiology of
desire of indulging in stealing. His mind however did the cancers with his treating physician. Here, the
not allow this feeling to enter his conscious, as that doctor is trying to avoid the painful emotion of being
would result in this person feeling bad about himself. diagnosed with the cancer by discussing excessively
Hence, this person resorts to straight away acting on about the pathophysiology of cancers.
the unconscious desire without even becoming aware I. Isolation of affect: Removing the feelings associated
of the same. This defense mechanism is involved in with a stressful life event. For example, without
development of impulse control disorders. showing any emotions, a woman tells her family
D. Passive-aggressive behavior: Indirectly expressing the members that she has been diagnosed with advanced
anger towards others. For example, a young boy was stage cholangiocarcinoma.
forced to bring a glass of water for the father, while J. Repression: It is one of the most important defense
bringing the water, the child accidentally tripped and mechanism, often referred to as the "primary" defense
I 118 Review of Psychiatry

mechanism. It is unconsciously forgetting something, students joked and laughed at themselves after getting
which can not be retrieved later. For example, a young humiliated by the examiner during the viva.
girl who was sexually abused by her father, "forgets" S. Sublimation: Expression of unacceptable feelings in
this incidence of sexual abuse. Now, even if she wants a socially acceptable manner. For example, a middle
to recall it, she can't do it in normal circumstances. aged man with unacceptable sexual desire becomes
K. Rationalization: Offering rational explanations to a painter and starts making nude paintings. Here, the
justify own unacceptable behavior. For example, an sexual desires are getting an outlet and its socially
alcoholic blamed his family environment for his habit acceptable since painting nudes is considered an art.
of excessive drinking. It is a commonly used defense T. Suppression: It is the only voluntary or conscious
mechanism in substance use disorders. defense mechanism. It involves a voluntary decision
L. Dissociation: Splitting of a single (e.g. memory, to not think about an event for some time and hence
identity) or group of mental functions from the avoid the accompanying emotions. For example, a
remaining mental functions. It is seen in disorders like medical student who is extremely stressed out because
dissociative identity disorder, where for example, of an upcoming entrance exam decides to take a one
the identity of an individual gets split from rest of the day break during which he doesn't think at all about
mental functions. the exam.
M. Reaction formation: Transformation of feelings into Defense mechanisms in psychiatric disorder: All the
exact opposite. For example, a man who is actually defense mechanisms are used at times by all of us.
infatuated by an office colleague tells his friend that However when used excessively, they can result in deve¬
he really hates her. Here, the actual feeling is that lopment of psychiatric disorders. Following is a list of few
of infatuation but that is being transformed into the defense mechanisms and associated disorders:
feeling of "hatred". A. Obsessive compulsive disorder: Reaction formation0,
N. Undoing: An act which is done to nullify a previous displacement0, undoing and inhibition0
act. For example, a husband brings gifts for wife next B. Phobia: Displacement and inhibition0
day after having a fight with her the previous day. The C. Dissociative disorder: Dissociation0
defense mechanism of undoing is used in obsessive D. Neurosis: Regression0.
compulsive disorder0.
O. Aim inhibition: Placing a limitation upon instinctual Psychosexual Stages of Development
demands, accepting partial or modified fulfillment
Sigmund Freud0 proposed that the sexuality develops
of desires. For example, a student who wanted to
in multiple stages. Freud used the term "sexuality" in a
became a doctor but who was not able to clear the pre broader concept that included others forms of pleasure
medical tests takes admission in a veterinary course also and not only genital sexuality. He proposed five
and becomes a veterinary doctor.
stages of development. Freud further proposed that the
Mature Defenses development may get arrested at a particular stage (called
P. Altruism: Satisfying internal needs by helping others. "fixation") and may result in development of psychiatric
For example, while driving in a drunk state, a man disorders:
met an accident and lost his son who was travelling A. Oral stage (0-1.5 years): This is the first stage of deve¬
alongside him. Later, he started a campaign against lopment where in the pleasure is derived from the oral
drunk driving and started educating people about ills cavity. The child derives pleasure in cutting, biting,
of drunk driving. chewing, etc.
Q. Anticipation: Planning in advance to deal with an B.Anal stage (1.5-3 years): The site of pleasure is anal
uncomfortable event. For example, a student plans all region. The child gets a sense of achievement by get¬
his arguments comprehensively before going to home ting toilet trained. If the psychosexual development
after a bad exam result. gets arrested at this stage (called fixation at anal
R. Humor: Using comedy to deal with unpleasant stage), it can result in development of obsessive com¬
feeling and situations. For example, two medical pulsive disorder0.
Psychoanalysis 119 ]

C. Phallic stage (3-5 years): The site of pleasure is the doesn't have a penis and desires to get one (known as
genital area. According to Freud, penis becomes the "penis envy"). The female child believes that she was
organ of principal interest to children of both sexes. castrated and that's why does not have a penis and
The male child develops what is known as oedipus holds her mother responsible for it, developing anger
complexQ in which he starts developing sexual feeling against the mother. The stage gets resolved when
towards the mother and wants to replace the father. the female child starts identifying with the mother.
However , the male child also becomes fearful, that if Failure to resolve the oedipus and electra complex
father finds it out, his father might castrate him (and can result in development of neurotic illnesses (like
hence the child develops castration anxiety*3). The hysteria). Hence, the neurotic illness develops due
oedipus complex in male child gets resolved once the to fixation at phallic stageQ.
child shifts his affection away from mother to some other D. Latent stage (5-12 years): During this stage, there is
female and starts identifying (starts imitating father and relative quiescence or inactivity of sexual drive and
trying to become like him) with the father. child focuses on learning and gaining skills.
In females, the oedipus stage unfolds differently E. Genital stage (12 years onward till young adulthood):
(at times the term used for female child is "electra This stage is characterized by maturation of genital
complex"). The girl child develops sexual desire for the functioning and gradual achievement of a mature
father. At the same time, she becomes aware that she sexual and adult identity.

QUESTIONS AND ANSWERS

QUESTIONS 6. Counter transference is: (AIIMS Nov 2011)


A. Type of defense mechanism
.
1 The term 'id' and "superego" were coined by: B. Psychic connection between patient and disease
(DNB 2003, DNB 1994, WB 2001) with transfer of psychic energy from body parts
A. Freud B. Skinner to brain
C. ErikErikson D. Bleuler C. Implies doctor's feelings towards patient
D. Patient's feelings towards doctor during psycho¬
2. That part of mindwhich works on reality principle
therapy
is: (DNB 2004, Karnataka 2001)
A. ID B. Ego .
7 Accordingto SigmundFreud,primaryprocessthink¬
C. Superego D. Ego-ideal ing is: (JIPMER 2011)
A. Illogical and bizarre
3. The term 'free association' which is a fundamental
B. Rational
technique of psychoanalysis was coined by:
(DNB 2006, JIPMER 2001) C. Absent during sleep
A. Freud B. Adler D. Logical and unconscious
C. Erikson D. Jung 8. Psychodynamic theory of mental illness is based
4. Theory of "Psychosexual development" was given on: (AIIMS Nov 2007)
by: (DNB Dec 2010) A. Unconscious internal conflict
A. Anna Freud B. Sigmund Freud B. Maladjusted reinforcement
C. Jean Piaget D. Skinner C. Organic neurological problem
D. Focuses on teaching patients to restrain absurd
5. Interpretation of dreams by Freudwas published
in: (UP 2001, KA 2002, DNB 1999) thoughts
A. 1990 B. 1900 9. Wrong statement about psychoanalysis is:
C. 1956 D. 1919 (DNB 2007, J&K2008; TN2006)
120 Review of Psychiatry

A. Parapraxis has meaning 18. Which of the followingexcludes painfulstimulifrom


B. Transference is patient's feeling for therapist awareness? (AIIMS 1998)
C. Counter transference is clinician's feelings for A. Repression B. Reaction formation
patient C. Projection D. Rationalization
D. Unguided communication has no meaning 19. Avoiding awareness of pain of realitybynegative
10. Oedipus complex (given by Sigmund Freud) is sensory data is seen in which of the following
seen in: (PGI 1998, Delhi 1998, DNB 2004) defense mechanisms? (MH2011)
A. Boys of 1-3 years of age A. Distortion B. Denial
B. Girls of 1-3 years of age C. Displacement D. Dissociation
C. Boys of 3-5 years of age 20. Postponing paying attention to a "conscious
D. Girls of 3-5 years of age impulse" or "conflict" is a mature defense mecha¬
11. In psychoanalytic terms, obsessive compulsive nism known as:
A. Sublimation B. Suppression
disorder is fixed at:
C. Humor D. Anticipation
(DNB 1998, Delhi 1998, TN2002, Mah. 2003)
A. Oedipal stage B. Genital stage .
21 A reluctant child forced to bringsugar from a shop
C. Oral stage D. Anal stage spills half of it on the way. This is an example of:
(JIPMER 1997, Delhi 2002, DNB 2004)
12. Fixation of hysteria is:
A. Hysteria B. Passive aggression
(DNB 1999, WB 2002, J&K2004, PGI2005)
C. Disobedience D. Active aggression
A. Genital B. Anal
C. Oral D. Phallic 22. A chronic alcoholic blames the family environ¬
ment as a cause of his alcoholism. This is pheno¬
13. Followingname(s) is/are associated with psycho-
menon of: (AIIMS 2000)
dynamic theory: (PGINov 2009)
A. Projection B. Denial
A. Carl Jung B. Sigmund Freud
C. Rationalization D. Sublimation
C. Emil Kraepelin D. EugenBleuler
E. Kurt Schneider 23. Ego's defense mechanism "Undoing" is typically
seen in: (PGI2001, AIIMS 1993, 1995)
Defense Mechanisms A. Depression
B. Schizophrenia
14. Not a defense mechanism: (PGI1998)
C. Obsessive compulsive neurosis
A. Derailment B. Repression
D. Hysteria
C. Distortion D. Undoing
24. Most important cause of neurotic reaction is the
15. Which of the following is a mature defense mecha¬
excessive use of:
nism? (DNB 2002, JIPMER 1991, UP 2007) (DNB 2005, PGI 1998, Nimhans 2001, Mah. 2004)
A. Projection B. Reaction formation
A. Projection B. Regression
C. Anticipation D. Denial C. Suppression D. Sublimation
16. Which of the following is not a neurotic defense 25. Displacement reaction is characteristically seen
mechanism? (DNB NEET 2014-15) in: (DNB 1998, MP 1998)
A. Isolation B. Regression A. Mania B. Phobia
C. Reaction formation D. Undoing C. Conversion disorder D. Depression
17. Which of the following is a neurotic defense 26. Defense mechanism in phobia is:
mechanism? (DNB NEET 2014-15) (DNB NEET2014-15)
A. Repression B. Anticipation A. Inhibition B. Dissociation
C. Projection D. Undoing C. Distorsion D. Conversion
Psychoanalysis 121 1
27. Defense mechanisms involved in OCD are: 11. D. See text
(PGI2012, PGI2007) 12. D. See text
A. Repression B. Undoing 13. A, B.
C. Rationalization D. Sublimation Apart from Sigmund Freud, other big names
E. Reaction formation associated with psychoanalysis include Carl lung
and Alfred Adler. Initially Jung and Adler worked
ANSWERS _ along with Freud, however later they separated
1. A. and gave their own theories.
2. B. 14. A. Derailment is a formal thought disorder and not
3. A. a defense mechanism.
4. B. 15. C.
5. B. 16. B. Regression is an immature defense mechanism.
6. C. Rest all are neurotic defense mechanism.
7. A. The primary process thinking is a characteristic 17. A,D.
of unconscious mind. It is illogical and aims for However, if you have to chose, go for repression.
immediate wish fulfilment. It is one of the most important neurotic defense
8. A. Psychodynamic (or psychoanalytic) theory mechanism.
stresses that unconscious memories and con¬ 18. A. Repression is the defense mechanism which
flicts are responsible for development of psychi¬ removes painful memories or unacceptable desi¬
atric disorders. The "conflict" may be between res away from the consciousness or awareness.
different parts of mindsuch as id and ego or ego 19. B. Denial is the defense mechanism which helps
and superego. a person to avoid (or refuse to accept) the rea¬
9. D. According to psychoanalytic theory, "parapraxis" lity. Don't get confused by the phrase "negative
or "slips of tongue" are believed to reveal uncon¬ sensory data".
scious content and hence are believed to have 20. B. Postponing or delaying action on a conscious
meaning. The description of transference and impulse (a conscious wish) and its accompany¬
countertransference given inthis question is also ing emotions is known as suppression.
correct. The last statement is wrong. In psycho¬
21. B. See text.
analysis, unguided communication is believed
22. C. See text.
to have meaning. Unguided communication
23. C. Undoingis typically seen inobsessive compulsive
here refers to the technique of "free association"
disorder.
in which patient speaks all that comes into his
mind, without any censoring. The "free associa¬ 24. B. Excessive use of regression causes neurotic ill¬
tion" helps in understanding the unconscious nesses.
contents of mind and hence is meaningful. 25. B. Displacement and Inhibition are the defense
10. C. Sigmund Freud described oedipus complex for mechanisms involved in phobia.
both sexes, however that term is mostly associ¬ 26. A.
ated with male sex now a days. 27. B, E.
Chapter

13 Miscellaneous

ELECTROCONVULSIVE THERAPY (ECT) B. Unilateral ECTS: In an attempt to decrease the side


effects of ECTs, the unilateral electrode placements
The convulsive therapies have long been used for have been introduced. The right unilateral ECT has
treatment of psychiatric disorders. Initially, intramuscular
been found to have better side effect profile in compari¬
injections of camphor were used to produce convulsions son to the bilateral ECTs and is being increasingly used.
in patients with psychosis, with good therapeutic results.
Later, electricity was used as an agent to induce convul¬ Mechanism of Action
sions and it was called "electroconvulsive therapy."
The induction of a bilateral generalized seizure is consi¬
Types dered necessary for the beneficial effect of ECTs. Earlier
it was considered that the response to ECTs was an "all
A. Direct ECT: In this technique, anesthetic agents and or none" phenomenon, however of late it has been found
muscle relaxants are not used. The generalized con¬ that at least in right unilateral ECTs, a dose response rela¬
vulsions produced can result in fracturesQ or teeth tion is present. The mechanism of action of ECTs is still
dislocations. Due to higher incidence of side effects not completely understood. Various hypothesis include
this technique is rarely used now. changes in the neurotransmitters (especially down regu¬
B. Modified ECT (Indirect ECT): Here, anesthetic lation of postsynaptic |3-adrenergic receptors), changes
agents and muscle relaxants are administered before in growth factors and molecular mechanisms (latest
giving ECT. As muscles are relaxed, the risk of bone research suggests increase in brain derived neurotrophic
fractures and other injures from the motor activity factor, BDNFQ as an important mechanism) and neuro¬
during the seizures gets minimized. genesis in areas like hippocampus.

Electrode Placement Indications


Various configurations have been developed for electrode A. Depression (Major depressive disorder): The ECT was
placement. These include: initially invented for the treatment of schizophre¬
A. Bilateral ECT: This is used most commonly and it nia and other psychotic illnesses, however currently
involves placement of electrodes on both sides of it is mostly used for treatment of depression0. ECT
the skull. In bilateral ECTs, various configurations of is effective for depression in both major depressive
electrode placement have been devised. The bifronto- disorder as well as bipolar disorder. The clearest
temporal electrode placement is deployed most indication for ECT is depression with suicide risk0.
commonly. Other commonly used configuration uses The indications of ECT in depression include the
bifrontal electrode placement. following:
Miscellaneous 123 1
• Depression with suicide risk (ECT is treatment of A. Raised intracranial tension0 (space occupying
choice in acutely suicidal patientsQ due to imme¬ lesion in CNS°)
diate onset of action) B. Recent myocardial infarction
• Depression with stuporQ C. Severe hypertension
• Depression with psychotic symptoms (psychotic D. Cerebrovascular disease
depression or delusional depression) E. Severe pulmonary disease
• In case of failed medication trials or intolerance to F. Retinal detachment.
medications.
B. Manic episode: Electroconvulsive therapy can be used COGNITIVE DEVELOPMENT STAGES
in the treatment of acute mania, however since effec¬
tive pharmacotherapy is available for mania, ECT is The thinking process undergoes a series of changes as the
not the first line treatment. The ECT is used in only
child grows up into an adult. Jean Piaget0, described four
those patients who are either intolerant/unrespon¬ stages of development of thinking processes, also known
as cognitive developmental stages. These are described
sive to pharmacotherapy or when mania is so severe
that there is a risk of homicide/suicide or danger of below:
A. Sensorimotor stage (Birth to 2 years): This is the first
physical violence and immediate control of symptoms
is required. stage. During this stage, child learns through sensory
observations and gradually gains control of his motor
C. Schizophrenia: Electroconvulsive therapy is the first
functions. Initially, the child thinks that if he cannot
line treatment in catatonic schizophreniaQ. It is also
see an object, it means that the object has ceased
effective in other types of schizophrenia however
to exist. For example, if a rattle with which child is
since the advent of antipsychotics, is used only if
playing, is taken away from the child and is covered,
patient is unresponsive/intolerant to medications.
so that the child can no longer see it, the child will
Electroconvulsive therapy is not effective in chronic
think that the rattle no longer exists and will not try
schizophrenia0.
to look for it. This type of thinking is also described
D. Other indications where ECT is occasionally used
as "out of sight, out of mind"0 and "here and now''0
include intractable seizures0, neuroleptic malig¬
type of thinking. In the end of the sensorimotor stage
nant syndrome0, delirium, on-off phenomenon of
the child develops "object permanence", which is the
Parkinson's disease, etc.
development of the concept that object continue to
exist even if they are not visible currently. In the above
Adverse Effects
example, once the child develops object permanence,
A. Memory disturbances: It is the most common side he will try to search for the rattle by removing the
effect of ECT. Both retrograde and anterograde amne¬ covering cloth as he now knows that the rattle con¬
sia is seen, however retrograde amnesia0 is much tinues to exist though he is not able to see it. Another
more common. It is however mild and recovery occurs important development at around 18 months, is a
usually within 1-6 months after treatment. process known as "symbolization". It means that
B. Other side effects include delirium, headache, muscle the infants now start developing mental symbols and
aches, fractures (very rare with modified ECT), nausea using words for objects. For example, they make a
and vomiting. mental symbol to represent a ball and use a word for
it. The development of "object permanence" indicates
Contraindications the transition to the next stage of development i.e.
There are no absolute contraindications0 of ECT. Earlier stage of preoperational thought.
raised intracranial tension was considered as an absolute B. Stage of preoperational thought (2-7 years): In this
contraindication, however it is now regarded as a relative stage, use of symbols and language becomes more
contraindication. Pregnancy is not a contraindication for extensive. The thinking process is characterised by
ECT. The following are the relative contraindications of "intuitive thought",0 which refers to thinking with¬
ECT: out use of reasoning and an inability to use logicality.
| 124 Review of Psychiatry

The children are also "egocentric" in this stage which LEARNING THEORY __
means that they are only concerned about their own
Learning is acquiring of new behavioral patterns. The two
needs and cannot think from others perspective.
types of learning are:
C. Stage of concrete operations (7-11 years): In this stage,
A. Classical conditioning
the egocentric thought is replaced by "operational B. Operant conditioning
thought" and hence the children start to see things
A. Classical conditioning: Classical conditioning (also
from others perspective also. The thinking is concrete
called respondent conditioning) results from the
(concrete thinking is the literal thinking. For example,
repeated pairing of a neutral stimulus with one that
when asked, the meaning of proverb "people who live
naturally produces a response. The concepts of clas¬
in glasshouses should not throw stones" the child will
sical conditioning emerged from the experiments
say that "if my house is of glass, Ishould not throw of Russian physiologist, Ivan Pavlov. The Pavlovian
stones as it will break my house". Thq child is not able to experiment included the following:
understand the deeper meaning. The logical thinking Under normal circumstances, a dog would salivate
starts to develop and children are able to understand to the smell of food. The ringing of bell would not
and follow rules and regulations. Two important produce any salivation response. In the experiment,
developments in this stage are attainment of "con¬ a bell was rung everytime before the presentation
servation" and "reversibility". Conservation is the of food. The dog ultimately paired the bell with the
ability to understand that despite changes in shape, food. Eventually the ringing of bell alone started to
the object remains the same. For example, water may produce salivation, even if no food was presented to
be transferred from a cup to a glass, and may appear the dog. The following are the elements of classical
different in shape, however the amount will remain conditioning:
the same. Reversibility is the capacity to understand • Unconditioned stimulus: It is a stimulus that natu¬
that one thing can turn into another and back again, rally without any learning, produces a response. For
e.g. water and ice. example, smell of food, which produces a response
D. Stage offormal operations (11 to end of adolescence): of salivation.
This stage is characterized by development of abstract • Unconditioned response: It is the natural response
to an unconditioned stimulus. For example, sali¬
thinkingQ, which is ability to understand the deeper
vation is the unconditioned response to smell of
meaning and deduce the larger meanings. For exam¬
food.
ple, when asked to explain the meaning of phrase
"pen is mightier than sword" a child with concrete
• Conditioned stimulus: It is a stimulus which when
paired with unconditioned stimulus, starts produc¬
thinking will say that the pen is heavier and stronger
ing a response. For example, ringing of bell usually
than the sword, whereas a child who has achieved
doesn't produce any response. However, when it is
abstract thinking will say that "power of knowledge
repeatedly paired with food (unconditioned stimu¬
is stronger than power of brute force" The thinking lus), it also starts to produce a response.
becomes logical, the child understands the concept
• Conditioned response: The response which results
of permutation and combination and probability. from pairing of conditioned stimulus to the uncon¬
There is development of "hypothetico deductive ditioned stimulus. For example, the salivation
thinking". Hypothetico deductive thinking is ability to which results secondary to ringing of bell is a con¬
make hypothesis and use deductive reasoning (ability ditioned response.
to deduce, e.g. a child while playing a video game
• Extinction: If the conditioned stimulus (ringing of
observes that whenever he breaks a banana, apple or bell) is presented repeatedly without the uncondi¬
cherry, he loses point, and hence is able to deduce tioned stimulus (smell of food), the response (sali¬
that in this game to win he should avoid breaking the vation) will decrease and eventually disappear. This
fruits). is called extinction.
Miscellaneous 125 j

• Stimulus generalizationQ: Here, a conditioned Behavior Therapy


response gets transferred from one stimulus to
According to learning theory, the maladaptive behaviors
other. For example, apart from the bell, ringing of
are learned by either classical conditioning or operant
a tuning fork also starts resulting in salivation.
conditioning and hence can be unlearnt. A large number
B. Operant conditioning (Instrumental conditioning):
The principles of operant conditioning were given of psychiatric disorders can be treated, if the psychiatric
by BF Skinner. According to this theory, a behavior symptoms are considered as learned maladaptive beha¬
is determined by its consequences'3 for the indi¬ viors. Behavior therapy is a psychological treatment in
vidual. Hence, according to this theory any behavior which the maladaptive behaviors of patients are changed
can be learned or unlearned and its frequency can to improve the quality of life. Behavior therapy is a generic
be changed by modifying the consequences of that term and is used to describe a variety of specific tech¬
behavior. If a behavior is followed by pleasant con¬ niques which intend to remove maladaptive behaviors.
sequence (called reward), that behavior will get rein¬ The techniques of behavior therapy include.
forced i.e. its frequency will increase. For example, A. Systematic desensitisation: This technique was deve¬
if a child is given a chocolate on studying for a par¬ loped according to the principle of "reciprocal
ticular amount of time, the frequency of studying will inhibition'"3. According to this principle if an anxiety
increase. Similarly if the consequence is negative, the provoking stimulus is provided while a person is in a
frequency of behavior will decrease. For example, if a relaxed state, the anxiety gets inhibited. For example,
child is slapped on using a bad word, the frequency if a person who is phobic to spiders is first made to
of using bad words will decrease. relax and then is exposed to a spider, he may develop
Types: The frequency of a behavior is increased by posi¬ much lesser anxiety. In systematic desensitisation, the
tive or negative reinforcement and decreased by punish¬ patient is first taught relaxation techniques (usually
ment or extinction. progressive muscle relaxation) and then a hierarchy is
made of anxiety provoking stimuli. For example, if a
Table 1: Types of operant conditioning. ÿ
person is afraid of heights, the list may have "standing
Type Effect Example
at the roof of a ten storey building" at the topTÿ'stand-
Positive Behavior is A child increases his study
hours as every study
ing on the balcony at second floor" in the middle
reinforcement0 increased by
a positive session is rewarded with a and "standing on third stair" at the bottom of list. The
consequence chocolate patient is then exposed (or asked to imagine that expo¬
(reward) sure) to a series of anxiety provoking stimuli, start¬
Negative Behavior is A child increases cleaning ing with the least anxiety provoking stimulus while
reinforcement0 increased of his room to avoid he is also using relaxation techniques. As the patient
to avoid a scolding by the mother
masters the technique of relaxation in the presence
negative
consequence of an anxiety provoking stimuli, he moves up to the
Punishment0 Behavior is A child stops using foul next stimulus.
decreased by language after getting Systematic desensitization is used in the treatment
a negative slapped for the same of phobias'3, obsessive compulsive disorders'3 and
consequence certain sexual disorders.
Extinction Behavior is An intern who used to B. Therapeutic graded exposure or in vivo exposure (or
decreased work very hard in the
due to lack of ward, becomes inefficient exposure and response prevention): It is similar to
reinforcement as he was never praised systematic desensitisation except that no relaxation
by his seniors. techniques are used and that real life situations are
used. For example, if a patient is afraid of dogs, the
PSYCHOTHERAPY exposure will start with looking at a picture of dog,
Psychotherapy is treatment of psychiatric disorders by then looking at a video of dog, followed by looking at a
using psychological methods. The following are impor¬ dog from a distance and finally holding a dog in arms.
tant kinds of psychotherapy: The patient learns to get habituated to anxiety (i.e. he
126 Review of Psychiatry

learns that anxiety gradually decreases by itself). It is disorders which are caused by dysfunction in autonomic
used in phobiasQ and obsessive-compulsive disorder. control such as asthma, tension headaches, arrhythmias,
C. Flooding (Implosion): Here, the patient is made to etc. The technique uses a feedback instrument, the choice
confront the feared situation directly, without any of which depends on the patients problem. This instru¬
hierarchy, as in systematic desensitisation or graded ment gives patient a feedback about the current status of
exposure. No relaxation exercises are used either. The a specific autonomic function. For example, an electro-
patient is exposed to the feared situation, experiences myogram (EMG) may be used to give patient feedback
fear and anxiety which gradually subsides, and the about muscle tension in a particular muscle group. When
patient is not allowed to escape. the muscle tension is high, the EMG will emit a higher
D. Modeling (Participant modeling): Here, therapist tone and when muscle tension is low (i.e. when muscle
himself makes the contact with phobic stimulus and is relaxed), the EMG will emit a lower tone. Using feed¬
demonstrates this to the patient. Patient learns by back, patient learns to control his muscle tone and hence
imitation and observation. For example, a therapist is able to control symptoms caused by increased muscle
himself took a dog in his arms while a patient who tone (e.g. bruxism).

had phobia of dogs observed him. This technique is


used in phobias as well as obsessive compulsive dis¬ Cognitive Therapy
orders. The cognitive theory assumes that the cognitions
E. Assertiveness training: Here a person is taught to be (thoughts) are at the core of psychiatric symptoms. On
assertive while asking for his rights and while refusing the basis of early experiences, an individual may develop
unjust demands of others. wrong patterns of thinking, known as cognitive distor¬
F. Social skills training: Usually used in patients with tions (or maladaptive assumptions). For example, a child
schizophrenia, it involves imparting skills required who was praised when he came first and was scolded
for dealing with others and living a social life. when he got second rank, may develop a cognitive dis¬
G.Aversive conditioning (Aversion therapy): It is the tortion that "To be successful it is necessary to get first
clinical use of principles of classical conditioning. It rank, otherwise Iwould be considered as a failure" These
is used for treatment of unwanted behaviors (such as cognitive distortions (or maladaptive assumptions) give
paraphilias0). Here, the patient is asked to imagine rise to "negative automatic thoughts" which are thoughts
that he is indulging into an unwanted behavior and with a negative connotation and appear automatically.
immediately a painful stimulus (such as an electric For example, in the above example, when the child with
shock) is given. An association gets created between the above mentioned cognitive distortion has a below
the unwanted behavior and painful stimuli0 and the expectation performance in the exam, he may start hav¬
unwanted behavior ceases. It is now rarely used due ing "negative automatic thoughts" like "lama failure" "I
to ethical considerations. performed badly in exams, Iwill perform badly in every
other exam" "I will never get a post graduation seat" etc.
Uses: The various technique of behavior therapy are used
The cognitive therapy aims to correct these "negative
primarily in treatment of anxiety disorders (like phobia,
automatic thoughts" and "cognitive distortions" When
panic disorders). Behavior therapy can also be used in
along with these, behavioral techniques are also used, the
depression, dissociative disorders, eating disorders, sexual
therapy method is known as "cognitive behavioral the¬
disorders, personality disorders, substance used disor¬ rapy" Cognitive therapy and cognitive behavioral therapy
ders and schizophrenia. are used in the treatment of depression, panic disorder,
obsessive compulsive disorder, personality disorder and
Biofeedback somatoform disorder.
It is a treatment technique that uses the principles of ope¬ Cognitive Distortions: Following is the list of common
rant conditioning. The biofeedback is based on the idea cognitive distortions0 (maladaptive assumptions):
that autonomic nervous system (which is usually invol¬ A. All or nothing thinking: Seeing things in black and
untary) can be brought under voluntary control with the white. For example, if Ifailed to get a particular job,
help of operant conditioning. It is used for treatment of it means that Iwould never ever get any job.
Miscellaneous 127 1
B. Approval seeking: Belief that you should always be K. Personalization: Blaming yourself for event, which you
liked and loved by others, otherwise life would be are not responsible for. For example, a wife blames
terrible. herself for her husbands extramarital affair.
C. Disqualifying positive: It is a tendency of refusal to L. Should statements: Having a lots of rules about how
acknowledge the positive events in life and insisting should you and others behave. For example, Ishould
that they "don't count". For example, a housewife was exercise daily, Ishouldn't be lazy.
praised by her husband, however she thought that "he
is praising me just to make me feel better, in reality I Substance Use Disorder: Psychosocial
don't deserve to be praised". Treatment
D. Emotional reasoning: Belief that your emotions reflect
The patients with substance use disorders (and other
the reality. For example, if Iam having a bad feeling
problematic behaviors) go through a series of changes
about a person, it means that the person in reality is a before quitting the substance use. Various models of
bad human being even if Ihave no evidences for the these changes have been described, the most acceptable
same.
model is known as transtheoretical model of change.
E. Fallacy offairness: Tendency to judge a random nega¬ According to this model, the following are the stages of
tive event as an issue of justice. For example, you
change:
missed the flight due to heavy traffic and you believe A. Precontemplation: In this stage, the substance user
"life is always unfair to me" doesn't see any problem in his behavior and doesn't
F. Jumping to conclusions: Making an interpretation with think about quitting.
minimal evidence. For example, a friend did not reply B. Contemplation: In this stage, the substance user starts
to your message and you made a conclusion that the realising that he has a problem and that he is taking
friend hates you. substance excessively. He considers about the pros
G. Labelling mislabelling: Giving labels to self or others. and consQ of stopping substance use. However, he is
For example, if your roommate didn't clean room yet to make any decision.
once, you label him as a "lazy slob" C. Preparation: In this stage, the substance user decides
H. Magnification (catastrophizing) and minimization: to quit the substance and starts making a plan to quit.
Focussing on worst possible outcome is maximization D. Action: In this stage, the substance user actually stops
and in its extreme form, it is called catastrophizing. taking the substance and makes changes in his beha¬
For example, if you lose a hundred rupee note and viors (e.g. he stops meeting with the friends who use
you say that its one of the biggest losses Iever had, its drugs in an attempt to keep away himself from drugs),
maximization. If you say that now there is nothing left starts taking treatment.
in my life, its catastrophization. Minimization is trying E. Maintenance: In this stage, the patient continues to
to minimise the importance of events. For example, stay away from substances (drugs) and continues with
an alcoholic when criticised about his heavy drinking
the treatment and other behaviors to prevent relapse.
A patient may remain in maintenance stage or may
says that "I don't really drink much, just a peg here
relapse if he starts taking substance again. Usually, a
and there"
patient has few relapses before attaining complete absti¬
I. Mental filtering/selective perception: Picking a single
nence (freedom) from substance.
negative detail while ignoring the rest. For example,
Various psychological treatment methods have been
in a party, everybody gave you a complement for
devised to help patient quit substance use and move
your looks, however a single person said that "have from stages of precontemplation to maintenance. One
you gained weight" and you give all the importance of the most commonly used technique which focuses on
to that one person's remark and ignore all the praise.
increasing the motivation of the patient to quit substance
J. Overgeneralization: Considering a single negative is known as motivation enhancement therapy or moti¬
event and making a general rule out of it. For exam¬ vational interviewing.
ple, you made a mistake at work and then you start Once the patient has reached maintenance stage,
thinking "I always mess up everything" Labelling is an relapse prevention techniques are used to prevent any
extreme form of overgeneralisation. relapses (return to previous pattern of substance intake).
HI 128 Review of Psychiatry

PSYCHOSURGERY _ c. Bhatia's battery of performance tests of intelli¬


gence.
The surgical techniques for treatment of psychiatric Personality assessment The personality assessment

disorder are rarely used and are reserved for only the can be done using two types of test:
chronic and severe cases which have not responded to all a. Objective test: These are standardized tests which
other methods of treatment. The psychosurgeries involve give numerical scores and can be analyzed using
creating a lesion in the limbic system or its connecting standard result tables. For example, Minnesota
fibres (limbic system is considered to be responsible for Multiphasic Personality Inventory (MMPI).
normal and abnormal emotional reactions). The lesions b. Projective tests: In these tests, patients are pro¬
are now a days produced with precision using stereotactic vided with ambiguous stimuli (unclear stimuli)
methods. The following are the commonly used psycho¬ and it is believed that the patient's response
surgeries. to such unclear stimulus reflects his internal
A. Stereotactic subcaudate tractotomy: It produces a thought processes and emotional factors. The
subcaudate lesion and is used in chronic, severe and patient "projects" his internal situation on to
intractable cases of depression, obsessive compulsive the test question and finally an expert analyses
disorder and schizoaffective disorder. the patients answers and deduces the aspects of
B. Stereotactic limbic leucotomy: Small lesion is made patients personality. The projective tests include:
in subcaudate and also a lesion is made in cingulate - Rorschach test?: The patient is shown ten
bundle. It is used in treatment of chronic, severe and cards which have inkblots and is asked
intractable obsessive compulsive disorder and schizo¬ what he sees in the card.
phrenia. - Thematic apperception test (TAT)Q: Here
C .Amygdalotomy: A lesion is made in amygdala in patients are shown certain pictures and
patients with severe, uncontrolled aggression. asked to make stories about them
- Sentence completion test?: Here patients are
NEUROPSYCHOLOGICAL TESTS _ given incomplete sentences and are asked
to complete them. For example, a sentence
Neuropsychology is a brach of psychology which exami¬ may be like "I wish I ."
nes the relationship between the behavior and brain - Word association technique: Here the
functioning. It tries to locate the areas of disturbances examiner says a word and patient has to
in brain, on the basis of behavioral symptoms (includ¬ respond with the first word that comes in
ing cognitive, sensory, motor and emotional symptoms). to his mind.
Neuropsychological tests are used extensively for various - Draw a person test (DAPT): Here patient
purposes. Few of them have been discussed below: is asked to draw a person and then specific
A. Neuropsychological assessment of intelligence and questions are asked about what he drew.
personality: B.Neuropsychological assessment for brain disorders
• Intelligence testing: The simplest way of measuring or organic mental disorders: Several tests have been
intelligence is in terms of Intelligence Quotient, IQ devised which extensively measure a wide range of
IQ = MA/CA x 100, MA is the mental age and CA cognitive functions like memory, motor functions,
is the chronological age, In this formula, the maxi¬ sensory functions, problem solving, reading, writing,
mum chronological age can be 15. arithmetic, etc. Few such important tests include:
Now, much better and precise tests have been • Luria Nebraska Neuropsychological battery
devised that measure the intelligence, few com¬ • Halstead Reitan battery of neuropsychological
monly used tests include: testsQ
a. Wechsler adult intelligence scaleQ • Bender Gestalt Test? (Bender visual motor gestalt
b. Malin's intelligence scale for Indian children test): This test is used mostly as a screening tool for
(MISIC) organic brain disorders.
Miscellaneous 129 (

QUESTIONS

QUESTIONS _ C. Neuroleptic malignant syndrome


D. Acute anxiety
ECT
8. Absolute contraindication to ECT is:(AIIMS 1995)
1. Indications for ECT is/are: (PGIMay 2010) A. Glaucoma B. Brain tumor
A. Psychotic depression C. Aortic aneurism D. MI
B. Catatonic schizophrenia 9. ECT is absolutely contraindicated in:
C. Cyclothymia A. Pregnancy (AI1992, DNB 1995)
D. Dysthymia B. Very ill patient
E. Post traumatic stress disorder C. Raised intracranial tension
2. Best marker for electroconvulsive therapy: D. Severe heart disease
A. CSF5HIAA (AIIMS Nov 2008) 10. Most common complication of ECT is:
B. CSF serotonin A. Anterograde amnesia (AIIMS 1996)
C. Brain derived growth factor B. Retrograde amnesia
D. CSF dopamine C. Psychosis
3. ECT is currently indicated as a line of treatment D. Depression
in the following conditions except: (UPSC 2008) .
11 Memory disturbance of ECT recovers in:
A. Catatonic schizophrenia A. Few days to few weeks (AIIMS 1996)
B. Severe depression with psychosis B. Few weeks to few months
C. Manic-depressive psychosis C. Few months to few year
D. Obsessive compulsive disorder D. Permanent
4. ECT is indicated in: (AIIMS 1998) 12. Most common complication of modified ECT:
A. Neurotic depression (AIIMS 1991, AI2, DNB 1997)
B. Auditory hallucination A. Intracerebral bleed
C. Chronic Schizophrenia B. Fracture spine
D. Delusional depression C. Body ache
5. ECT is not useful in treatment of: D. Amnesia
A. Chronic schizophrenia (AI 1993, DNB 1994) 13. True about ECT is: (PGIMay 2012, AIIMS 2011)
B. Catatonic schizophrenia A. It is not a treatment for dysthymic disorder
C. Endogenous depression B. Used to treat complex partial seizures
D. Acute psychosis C. Used for those major depressive patients not
6. ECT indepressive phase of MDP is usefulbecause responding to medication
it: (PGI1999) D. Memory impairment is a side effect
A. Produces recurrence E. Effective in OCD
B. Reduces recurrence
C. Shortens duration Names
D. Increases drug effects
14. Who introducedcocaine in psychiatry:
7. All of the following are indications for ECT except: (Kerala 1998, DNB 1992)
A. Intractable seizures (DNB NEET 2014-15) A. Freud B. Jung
B. Depressive stupor C. Miller D. Stanley
| 130 Review of Psychiatry

15. Moral treatment of mentally ill-patient was first 22. 'Reinforcement' is used in: (AIIMS 1994, 1999)
stressed by: A. Psychoanalysis
(AIIMS 1995, CMC 1998, DNB 2001, TN 2004) B. Hypnosis
A. Pinel B. Morel C. Abreaction
C. Kraepelin D. Sigmund Freud D. Conditioned learning

16. The eight stage classification of human life is pro- 23. Behavior therapy to change maladaptive behavior
posed by: (DNB 2K, WB 2004, UP 2005) using response as reinforcer uses the principles
A. Sigmund Freud B. Pavel of: (AI2003)
C. Strauss D. Erikson A. Classical conditioning
B. Modeling
17. Which of the following scientist propagated 'thera¬
C. Social learning
peutic community concept:
D. Operant conditioning
(Karnataka 2K, DNB 2003)
A. JB Watson B. Maxwell Jones 24. Many of our bad habits of day to day life can be
C. Freud D. Adler removed by: (AIIMS Nov 2004)
A. Positive conditioning
Cognitive Development Stages B. Negative conditioning
C. Bio feed back
18. Which of the following is a stage of intuitive
D. Generalization
thought appearance in Jean-Piaget scheme:
(PGI1999) 25. Operant conditioning in which pain stimulus are
A. Sensorimotor given to a child for decreasing a certain undesired
B. Concrete behavior can be classified as: (AI2010, 1997)
C. Preoperational stage A. Positive reinforcement
D. Formal operations stage B. Negative reinforcement
C. Punishment
19. Ability to form a concept and generalize is known
D. Negotiation
as: (JIPMER 2011)
A. Concrete thinking 26. A child is not eating vegetables. His mother starts
B. Abstract thinking giving a chocolate each time he finishes vegetables
C. Intellectual thinking in the diet. The condition is: (AIIMS Nov 2012)
D. Delusional thinking A. Operant conditioning
B. Classical conditioning
20. In Piaget's theory of cognitive development 'out
C. Social training
of sight, out of mind' and 'here and now' is seen
D. Negative reinforcement
in the stage of: (AIIMS 2013)
A. Sensorimotor stage 27. Patient of contamination phobia was asked by
B. Preoperational stage therapist to follow behind him and touch every¬
C. Concrete operational stage thing he touches. During process therapist kept
D. Formal operational stage talking quietly and calmly to the patients. The
patient was asked to repeat the procedure twice
Learning Theory and Psychotherapy daily. The procedure is: (AIIMS May 2010)
21. Pavlov's experiment is an example of: (AI2006) A. Flooding B. Modeling
A. Operant conditioning C. Positive reinforcement D. Aversion therapy
B. Classical conditioning 28. Therapeutic exposure is a form of: (MH2011)
C. Learned helplessness A. Behavior therapy B. Psychoanalysis
D. Modeling C. Cognitive therapy D. Supportive therapy
Miscellaneous 131

29. Reciprocal inhibition is done by: (SGPGI2000) about quittingbut is reluctant to do so because he
A. Systematic desensitisation is worried that quitting will make him irritable.
B. Flooding Which of the following option best describes the
C. Exposure and response prevention stage of behavior change: (AI2011)
D. Psychoanalysis A. Precontemplation and preparation
B. Contemplation and cost factor
30. Along a pleasant stimulus,a noxious stimuli is
C. Contemplation and sickness susceptibility
given in treatment of alcohol dependence and
D. Belief
paraphilias. This is an example for which kind of
behavior therapy: (MH2008) 38. A smoker isworried about the side effects of smok¬
A. Negative reinforcement B. Aversive therapy ing. But he does not stop smoking thinking that
C. Punishment D. Fooding he smokes less as compared to others andtakes a
good diet. This thinking is called as:
.
3 1 Behavior therapy is useful in: (PGIJune 2008)
(AIIMS May 2015)
A. Psychosis B. OCD
A. Self-exemption B. Cognitive error
C. Personality disorder D. Panic attack
C. Self-protection D. Distortion
E. Anxiety disorders
32. A patient can be taught to control his involuntary Neuropsychological Tests
physiological responses by which of the following
39. A Study comparing the behavioral and develop¬
therapies: (MH2009)
mental changes ina normalbrainwith a damaged
A. Breathing exercise
brain is: (AIIMS 2013)
B. Stress modification
A. Neuropsychology
C. Biofeedback
B. Neurodevelopmental psychology
D. Rational emotive therapy
C. Child psychology
33. Tics, hair pulling, nail biting can be treated by: D. Criminal psychology
(DNB December 2011)
40. Rorschachinkblot test is: (BIHAR 2003)
A. Mind fullness
A. Projective B. Subjective
B. Social habit training
C. Both D. None of the above
C. Habit reversal training
D. No intervention required 41. Best test for diagnosis of organic mentaldisorder:
A. Sentence completion test (AI2000)
34. Which of the following is not a cognitive error/
(AI2010) B. Bender gestalt test
dysfunction?
C. Rorschach test
A. Catastrophic thinking B. Arbitrary inference
D. Thought block D. Thematic appreciation test
C. Overgeneralization
42. Rorschach test measures: (PGI 1999)
35. Typically changes inproblembehavior shows how
(DNB NEET 2014-15) A. Intelligence B. Creativity
many stages:
A. 2 B. 3 C. Personality D. Neuroticism
C. 4 D. 5 43. Signs of organic brain damage are evident on:
36. All of the following are parts of cognitive behavior A. Bender-Gestalt test (AI2004)
change technique except: (AI2010) B. Rorschach test
A. Precontemplation B. Consolidation C. Sentence completion test
C. Action D. Contemplation D. Thematic apperception test

37. A chronic smoker taking 20 cigarettes per day has 44. Halstead Reitan battery involves all except:
developed chronic cough. His family suggested A. Finger oscillation
quitting cigarettes. He is ready to quit and thinks B. Constructional praxis
ÿ 132 Review of Psychiatry
C. Rhytm A. Severe depression
D. Tactual performance B. Conversion disorder
C. Personality disorder
Miscellaneous D. Somatization disorder

45. A personlaughs to a joke, and then suddenly loses 53. Serial 7 subtraction is used to test:
tone of all his muscles. Most probable diagnosis (DNB NEET2014-15)
of this condition is: (DNB Dec 2009) A. Working memory B. Long-term memory
A. Cataplexy B. Catalepsy C. Mathematical ability D. Recall power
C. Cathexis D. Cachexia (DNB NEET2014-15)
54. Erotomania is seen in:
46. Hypomimia is: (DNB NEET 2014-15) A. Schizophrenia B. Mania
A. Decreased ability to copy C. Neurosis D. OCD
B. Decreased execution
55. Highest level of insight is: (DNB NEET 2014-15)
C. Deficit of expression by gesture B. Emotional
A. Intellectual
D. Deficit of fluent speech D. Affective
C. Psychological
47. Deja vu is seen in: (Kerala 1994)
56. Which category of ICD is associated with mood
A. Temporal lobe epilepsy (DNB NEET 2014-15)
disorders:
B. Normal person B. F 20-29
A. F 10-19
C. Psychosis
C. F 30-39 D. F 40-49
D. All of the above
57. Which category of ICD is associated with schizo¬
48. Unfamiliarity of familiar things is seen in:
phrenia? (DNB NEET2014-15)
(Kerala 1999, JIPMER 2002) (Karnataka 1994)
A. F 10-19 B. F 20-29
A. Dejavu B. Jamais vu
C. F 30-39 D. F 40-49
C. Dejaentendu D. Deja pence
58. DSM-IV classification of psychiatric disorder as
49. Patient wanting to scratch for itching in his
proposed by American Psychiatric Association
amputated limb is an example of: classifies and helps indiagnosing patients on mul¬
A. Illusion (DNB NEET 2014-15)
tiple axes. Of these, axis V represents the degree
B. Pseudohallucination of: (MH2009)
C. Phantom limb hallucination A. Present state of symptoms
D. Autoscopic hallucination B. Comorbid medical condition
50. All of the following are true about pseudohalluci- C. Global assessment of function
nations except: (DNB NEET 2014-15) D. Comorbid psychological problem
A. It arises in inner subjective self 59. When informationmemorized afterwards isinter¬
B. Patient describes that the sensations are being fered by the information learnt earlier, it is called:
perceived by "mind's eye" A. Retroactive inhibition (AIIMS May 2004)
C. They are under voluntary control B. Proactive inhibition
D. Distressing flashbacks of PTSD is an example C. Simple inhibition
51. Catatonia is most commonly seen with: D. Inhibition
A. Schizophrenia (DNB NEET 2014-15) 60. Methods of learning in psychiatry are all except:
B. Depression (AIIMS Nov 2007)
C. Anxiety disorder A. Modelling B. Catharsis
D. Obsessive compulsive disorder C. Exposure D. Responseprevention
52. Catatonic features are seen inschizophrenia, they .
61 According to DisabilitiesAct, 1995, the seventh dis¬
are also seen in: (PGIJun 2008) ability is usually referred to as? (AIIMS Nov 2008)
Miscellaneous 133
A. Neurological abnormality 10. B. Amnesia is the most common side effect of ECT.
B. Mental illness Both retrograde and anterograde amnesia are
C. Substance abuse seen, however retrograde amnesia is much more
D. Disability due to road traffic accident common.
11. B. Amnesia caused by ECT is mild and recovery
62. Patients suffering from which of the following
occurs usually within 1-6 months after treatment.
disease as per ICD/DSM criteria are eligible for
12. D.
disability benefit as per National Trust Act?
13. A,C,D.
(AI2009)
A. Schizophrenia B. Bipolar disorder
Names
C. Dementia D. Mentalretardation
14. A. Sigmund Freud studied about the effects of

63. Consultation liaison (C-L) psychiatry involves
cocaine. It is also believed that he was addicted
diagnosing: (MAHE 2006, SGPGI 2004) to cocaine for a long period.
A. Psychiatric illness in medically ill 15. A. Moral treatment of mentally ill patients using
B. Medicalillness in psychiatric patients humane methods was first stressed by Pinel.
C. Suicidal tendency in psychiatric patients 16. D. Erik Erikson divided the human life into eight
D. Suicidal tendency in medically ill stages, known as Erikson's psychosocial stages.
17. B. Therapeutic community is a group based
ANSWERS_ approach for treatment of substance use
disorders and other psychiatric disorders. It is a
ECT residential approach where in patients live in a
l. A,B. house for long-term andhave defined roles dur¬
See text. ing the stay. The term "therapeutic community"
was given by Thomas Main and the concept was
2. C. Latest research suggests that increase in brain
developed by Maxwell Jones.
derived neurotrophic factor, BDNF mediates the
response to ECT and is the best marker for the
same. Cognitive Development Stages
3. D. ECT is rarely used in the treatment of OCD. 18. C. Intuitive thinking is seen in stage of preopera¬
4. D. Delusional depression or psychotic depression tional thought.
is an indication for ECT. 19. B. Abstract thinking is the ability to make concepts
5. A. Electroconvulsive therapy is not effective in (i.e. ability to grasp essential of whole) and to
chronic schizophrenia. generalise.
6. C. ECT shortens the duration of depressive episode. 20. A. See text.
It doesn't prevent the recurrence unless given as
a maintenance treatment. Learning Theory and Psychotherapy
7. D. ECT is occasionally used in intractable seizures, 21. B.
neuroleptic malignant syndrome, delirium, on- 22. D.
off phenomenon of Parkinson's disease. Acute 23. D. Use of rewards as a reinforcer (in positive rein¬
anxiety is not an indication. forcement) is a technique of operant condition¬
8. B. There are no absolute contraindications for ECT. ing.
Earlier, raised intracranial tension and space 24. B. Negative conditioning is used to decrease the
occupying lesions were considered as absolute frequency of a particular behavior.
contraindications, hence the best answer here is 25. C. Punishment is decrease in frequency of a beha¬
brain tumor. vior due to unpleasant consequences.
9. C. Again, the best answer is raised intracranial ten¬ 26. A. This is an example of positive reinforcement, a
sion. type of operant conditioning.
I 134 Review of Psychiatry

27. B. This is an example of participant modeling in 47. D. Deja vu refers to the feeling that an event which
which patient learns by observation and imita¬ is being currently experienced has also happened
tion of therapist. in the past. It can be seen in normal persons
28. A. and also in certain disorders like temporal lobe
29. A. epilepsy.
the technique of systematic desensitisation. 48. B. Jamais vu refers to the feeling of unfamiliarity for
30. B. Aversive therapy. familiar things.
31. A,B.,C,D,E. 49. C. Phantomlimbisthe experiencingofsensationsinan
Behavioraltherapy is primarily used intreatment amputated limb.
of anxiety disorders (including panic disorder), 50. C. Pseudohallucinations are not under voluntary
obsessive compulsive disorder. It is also useful control.
in personality disorders. Though, in psychotic 51. B. Catatonia is most commonly seen in mania fol¬
disorders like schizophrenia, behavioral therapy lowed by depression and than schizophrenia.
is not the first line treatment, however it can be 52. A.
used. 53. A. Serial 7 subtraction test, in which the patient is
32. C. asked to serially subtract 7 from 100 is a test for
33. C Habitreversaltrainingisakindofbehavioraltherapy working memory.
which is used in the management of tics, tri¬ 54. A. Erotomania or delusion of love is most commonly
chotillomania, nail biting, skin picking and seen in schizophrenia and delusional disorder.
other similar disorders. The technique involves 55. B. Emotional insight is the highest level of insight. In
getting aware of the urge that precedes tics and emotional insight, the patient is aware of the ill¬
other impulsive behaviors and developing an ness and also changes his behavior accordingly.
alternative response. Intellectual insight is next to emotional insight
34. D. Thought block is not a cognitive error. in the hierarchy of insight. Inintellectual insight,
35. D According to the transtheoretical model, there the patient is aware that he has illness, however
are 5 stages of change in substance use and other he doesn't change his behavior in any manner
problem behaviors. based on this knowledge.
36. B. Consolidation is not a stage of change. 56. C. The fifth chapter ofICD-10 classifies psychiatric
37. C. In this question, patient is considering quitting disorders. The chapter has been further sub
and thinking about the pros and cons of it. This divided into blocks as described below:
is characteristic of stage of contemplation. F00-F09: Organic, includingsymptomatic, mental
38. A. Self-exemption refers to the beliefs that give disorders
smokers false reassurances and allow them to F10-F19: Mentaland behavioral disorders due to
avoid thinking deeply about the importance of psychoactive substance use
quitting. F20-F29: Schizophrenia, schizotypal and delu¬
sional disorders
Neuropsychological Tests F30-F39: Mood (affective) disorders
39. A. See text. F40-F48: Neurotic, stress-related and somato¬
40. A. Rorschachinkblot test is a projective test. form disorders
41. B. See text. F50-F59: Behavioral syndromes associated with
42. C. Personality. physio-logical disturbances and physical factors
43. A. F60-F69: Disorders of adult personality and
44. B. Constructionalpraxis isnot a part ofhalsteadreitan behavior
battery. F70-F79: Mental retardation
F80-F89: Disorders of psychological development
Miscellaneous F90-F98: Behavioral and emotional disorders
45. A. with onset usually occurring in childhood and
46. C. Hypomimia refers to decrease in facial expres¬ adolescence
sions, usually seen in parkinsonism. F99-F99: Unspecified mental disorder.
Miscellaneous 135 I
57. B. 59. B. The tendency of previously learned information
58. C. In DSM-IV, a multiaxial system was used to hinder subsequent learning is known as pro¬
while making the diagnosis. The diagnosis was active inhibition,
described in the following five axes: 60. B. Catharsis is not a method of learning. The term
Axis I:Clinical syndromes/Disorders (psychiatric "catharsis" is used to denote the process of
disorder) release of pent-up emotions (emotional outlet),
Axis II:Personality disorders/Mental retardation 61. B. According to persons with disability Act, 1995; the
Axis III: Medical conditions sixth disability is mental retardation and seventh
Axis IV: Psychosocial and environmental stress¬ disability is mental illnesses.
ors 62. D. The National Trust Act is applicable for autism,
Axis V: Global assessment of functioning cerebral palsy, mental retardation and multiple
In DSM-5, the multiaxial system has been disabilities.
removed.The former axis I, IIand IIIhave been 63. A. Consultation liaison psychiatry is the speciality
combined and for the last two, separate notations of psychiatry which deals with the psychiatric
are being used. illnesses in medically ill patients.

You might also like

pFad - Phonifier reborn

Pfad - The Proxy pFad of © 2024 Garber Painting. All rights reserved.

Note: This service is not intended for secure transactions such as banking, social media, email, or purchasing. Use at your own risk. We assume no liability whatsoever for broken pages.


Alternative Proxies:

Alternative Proxy

pFad Proxy

pFad v3 Proxy

pFad v4 Proxy